Success Physics SPM Free Chapter PDF

You might also like

You are on page 1of 130

Notes:

1. Click on the green buttons to view the multimedia learning resources (internet
connection required)
2. E-Reader functions (e.g.: highlight, notes) will not be available in this PDF view.
3. Desktop and laptop users may open the file within Adobe Digital Editions (ADE) to view the
e-book sample chapter with all the e-Reader functions. Please refer to these links for a step-
by-step guide to install (ADE): Windows Mac OS
FORM 4

2
CHAPTER

Forces and Motion

2 Forces and Motion SPM Topical Analysis


Year 2007 2008 2009 2010 2011
Paper 1 2 3 1 2 3 1 2 3 1 2 3 1 2 3
Section A B C A B A B C A B A B C A B A B C A B A B C A B
Number of questions 8 1 1 1 7 1 1 1 8 1 1 9 1 1 1 9 1 1

ONCEPT MAP

FORCES AND MOTION

Kinematics Dynamics

Linear Motion graphs Inertia Momentum Elasticity


motion s t graph
v t graph
Mass Hookes
Impulse, Conservation law
Ft = mv mu of F = kx
Distance Speed momentum
Newtons first
Displacement = Distance law of motion
Time Impulsive force,
Velocity mv mu Elastic potential
F= energy,
Displacement t
= 1 2
Time Effects of a force E= kx
Acceleration 2
Deceleration
vu
a= t Newtons second Forces in Newtons third
law of motion equilibrium law of motion
F = ma
Acceleration due
to gravity, g Work, Energy
W = Fs
Equations of
motion
v = u + at Gravity Potential Kinetic energy,
s = 1 (u + v)t energy, 1
2 Ep = mgh Ek = 2 mv2
s = ut + 1 at 2 Safety features
2 Weight, in the design of
v2 = u2 + 2as W = mg vehicles Power

COMPANION WEBSITE
Learning Objectives 37
2.1 Analysing Linear Motion

2.1 Analysing Linear Motion

Linear Motion

1 Linear motion is motion in a straight line. 3 Examples of non-linear motion:


2 Examples of linear motion: Earth

Sun


F
O (a) A spinning top (b)The earth orbiting
2

R the sun
CHAPTER

M
(a)A passenger on a moving escalator Figure 2.2
4 4 The study of the motion of an object without
considering the forces acting on it is called
kinematics.
5 The study of the motion of an object and
(b) An athlete running a 100 m race
F
the forces acting on the object is called
Figure 2.1 O dynamics.
2

R
CHAPTER

M
Distance, Displacement, Speed and Velocity
4

1 The physical quantities of motion are distance, displacement, speed, velocity, time and
acceleration.

Motion
An object in motion changes position.

Distance Displacement
The total length of the path an object travels from The distance an object travels in a specific
one location to another. direction.
Distance is a scalar quantity. The magnitude of displacement is equal to the
shortest distance between two points.
Thus, displacement is a vector quantity.

Speed and velocity both describe how fast an object is moving but
there is an important difference between these two quantities,
i.e, velocity is related to direction.

Velocity
Speed Velocity is the speed of an object in a specified
Speed is the distance covered per unit time, that is, direction, that is, the rate of change in
the rate of change in distance. displacement.
Total distance travelled, s (m)
_________________________ Distance moved in a specific direction
______________________________________
Speed, v = Velocity, v =
Time taken, t (s) Time taken
Speed is a scalar quantity. ____________________
Displacement, s (m)
=
Time taken, t (s)
Velocity is a vector quantity.

Forces and Motion 38


2 Both distance and displacement have the same SI units.
They are measured in metres (m).
3 Both speed and velocity have the same SI units of metre per An object moves from position
second (m s1). A to B. The figure below
4 However, since displacement and velocity are vector quantities, the shows five possible paths
direction of motion must be stated together with the magnitude. taken by the object. Which is
the shortest path?
Q

R B

1
P X
A
Figure 2.3 shows the location of two towns, P and Q.
Y F
N
lake O

2
The path with the shortest R

CHAPTER
Jamil's length is APB (the straight line M
Town Q
car that joins A and B) and APB is
Raven's 4
helicopter known as the displacement
Town P from A to B. Q
The magnitude of the
mountain displacement is the shortest R
Figure 2.3 distance that links the initial
and final positions of an
Jamil drives a car along the road from town P to town Q, which is 300 km away. object that has moved.
P
The journey takes 5 hours. Raven flies a helicopter due east from town P to Q A
The other paths, AQB, ARB,
for a distance of l00 km in half an hour. AXB and AYB are known as
Analyse, in terms of speed and velocity, Jamil's and Raven's journeys. the distances travelled from A
to B.
Solution
Jamils journey Ravens journey
Distance travelled = 300 km Displacement = 100 km due east
Time taken = 5 hours Time taken = 0.5 hour
Distance travelled Distance in a
________________
Speed, v = Displacement
____________ specific direction.
Velocity, v =
Time taken Time taken
_______ 300 km _______ 100 km
= =
5h 0.5 h
= 60 km h1 = 200 km h1
The speed of the car is 60 km h1. The velocity of the helicopter is 200 km h1 due east.

Note: SI unit is not used in this example.

Average Speed and Average Velocity


2
1 Refer to Example 2.
An athlete runs 100 m in 10 s.
2 The term average velocity is used because the
What is his average velocity?
athlete is not running at a constant velocity of
10 m s1 in the whole race. Solution
3 Similarly, the speed of the car (in Example 1) is s

Average velocity, v = __
an average speed since the speed of the car is
t
not the same throughout the whole journey.
______ 100 m
4 Table 2.1 shows the difference between average =
speed/average velocity and constant speed/ 10 s
constant velocity. = 10 m s1

39 Forces and Motion


Table 2.1

Average speed / Average velocity Constant speed / Constant velocity


A car moves at an average speed/velocity of A car moves at a constant or uniform speed / velocity
20 m s1 (equivalent to 72 km h1). of 10 m s1.
This means the car may move 15 m in the first This means the car moves 10 m in the first second,
second, 25 m in the next second and 20 m in the 10 m in the next second, 10 m in the third second,
third second. and so on.
On average, the car moves a distance/displacement The car always covers a distance or displacement
of 20 m in 1 second for the whole journey. of 10 m in 1 second for the whole journey.
The magnitude of speed/velocity remains the same.
15 m 25 m 20 m
F t=0 t=1s t=2s t=3s 10 m 10 m 10 m
O
2

R t=0 t=1s t=2s t=3s


CHAPTER

An object is said to be moving at a constant or uniform speed if it moves


equal distances in equal successive time intervals, no matter how small the
F
time interval is. O
2

For example, an object moving at Ra constant speed of 10 m s1 covers a


distance of 10 m every second, or 1 M
m every 0.1 s.
CHAPTER

SPM
3 07/P1

In an activity for a Physics lesson, a student was (a) Total distance travelled = AB + BC
instructed by his teacher to run due north for = 12 + 16
a distance of 12 m before moving east for another = 28 m
16m. The time taken was 20 s. _____________________ Total distance travelled
Average speed, v =
What was the students Time taken
(a) average speed, and 28 m
_____ =
(b) average velocity?
20 s
= 1.4 m s1
Solution (b) Displacement
= The distance travelled in the direction of AC
16 m
B C = 122 + 162 Apply Pythagoras theorem
= 20 m
__
tan = 16 To determine the
direction of AC
12
= 1.333
12 m
tan = 53.1
N Average velocity, v
Displacement
=
Time taken
20 m
A =
20 s
= 1 m s1 in the direction N53.1 E

Forces and Motion 40


4
Figure 2.4 shows a car moving round a roundabout. Solution
VR
VP (a) The speed of the car is 3 m s1.
P (b) (i) At P:
R
The velocity, vP = 3 m s1 due east
(ii) At Q:
The velocity, vQ = 3 m s1 due west
N Q (iii) At R:
VQ The velocity, vR = 3 m s1 due N 45 W

Figure 2.4 Note: F


If the car covers a distance of 3 m each second, find The speed remains constant throughout while the O

2
VR V
(a) the speed of the car, F4/2/3 velocity is changing (i.e. change in direction, but not
P
R
P

CHAPTER
(b) the velocity of the car at in magnitude, bearing in mind that velocity is a vector M
R
(i) P(ii)Q(iii)R quantity which has both magnitude and direction.) 4

Acceleration and Deceleration N Q


VQ
1 When the velocity of an object changes with 3 Since velocity is a vector quantity (i.e., it has
time, the object is said to be accelerating. magnitude as well as direction), acceleration
2 Acceleration is defined as the rate of change is, thus, a vector quantity.
in velocity with time. F4/2/3
4 The SI unit for acceleration is metre per
Acceleration, a Initial velocity = u second per second or m s2 (read as metre per
Change in velocity Final velocity = v second squared).
= Time taken = t
Time taken
Final velocity Initial velocity
=
Time taken

a = v u
t
vu
v>u Acceleration, a = v<u
t

Acceleration Deceleration/Retardation

When v > u, a is positive, the velocity is increasing. When v < u, a is negative, the velocity is decreasing.
Thus, the speed of the object increases and is said Thus, the speed of the object slows down and is
to be accelerating. said to be decelerating.
For example, For example,
u=0 v = 30 m s1 u = 30 m s1 v=0
30 35 30 35 30 35 30 35
25 40 25 40 25 40 25 40
20 20 20 20
45 45 45 45
15 15 15 15
50 50 50 50
10 10 10 10
5 m s1 55 5 m s1 55 5 m s1 55 5 m s1 55
0 60 0 60 0 60 0 60

60 60 60 60
55 27
30
3
5 55 27
30
3
5 55 27
30
3
5 55 27
30
3
5
6 6 6 6

50
24

50
24

50
24

50
24

21 9 10 21 9 10 21 9 10 21 9 10
18 12 18 12 18 12 18 12
15 15 15 15

45 15 45 15 45 15 45 15

40 20 40 20 40 20 40 20
35 25 35 25 35 25 35 25
30 30 30 30

t=0 t=5s t=0 t=5s

The driver steps on the accelerator when the traffic The driver applies the brakes when he sees the
light turns green. The car increases its speed with traffic lights turn red to reduce its velocity with a
F4/2/4 F4/2/5
an acceleration of 6 m s2. deceleration or retardation of 6 m s2 until it stops.
Mathematically, we write as: a = 6 m s2 Mathematically, we write as: a = 6 m s2

41 Forces and Motion


5
From a constant velocity of 2000 m s1, the velocity of Acceleration, a
a rocket increases to 3000 m s1 in 5 s when the rate of vu
=
combustion in the combustion chamber is increased. t
What is the acceleration of the rocket? 3000 m s1 2000 m s 1
=
5s
Solution
1000 m s 1
= 200 metres per second per
5s second, i.e., a gain in velocity
200 m s 1 of 200 m s1 in each second.
=
1s
u = 2000 m s1 v = 3000 m s1
F t=0 t=5s = 200 m s2 Also read as 200 metres per
O second squared.
2

R
CHAPTER

The acceleration of the rocket in Example 5 is 200 m s2. This means that its velocity increases by 200 m s1
for every second, as illustrated in the following diagram.

F
t=0 t=1s Ot = 2 s t=3s t=4s t=5s
2

v = 2000 m s1 v = 2200 m s1 v = 2400 m s1


R v = 2600 m s1 v = 2800 m s1 v = 3000 m s1
CHAPTER

4 F4/2/9
6
Azmi cycles at a uniform speed of 20 m s1. He then Solution
stops pedalling and his bicycle comes to a stop after u = 20 m s1, v = 0 m s1 and t = 8 s
8 s. What is his average deceleration? v u
____
Acceleration, a =
u = 20 m s1 v=0 t
_____ 0 20
=
8 Negative means
= 2.5 m s2 deceleration.
t=8s
stop pedalling bicycle stops Deceleration = 2.5 m s2
Figure 2.5

change in displacement change in velocity


Displacement Velocity Acceleration
causes causes

Study of Linear Motion SPM


03/P3/(B)

Ticker-timer

1 A ticker-timer is a device used in the laboratory to study the motion of a moving object,
usually a trolley.

Forces and Motion 42


2 The ticker-timer can be used to determine the (c) The velocity of the object
following variables. (d) The acceleration of the object
(a) The time interval of the motion (e) The type of motion of the object
(b) The displacement of the object

2 1
A vibrating metal strip with a pin is set to vibrate A ticker-timer consists of an electrical
up and down 50 times per second (i.e., at 50hertz, vibrator, which is connected to an alternating
which is the frequency of the a.c. supply). current (a.c.) power supply (12 or 6 V).

1 vibrator
2 metal strip
F
50 dots are O

2
punched on the R
tape in one second

CHAPTER
M
3
ticker-tape 4
inclined runway
4 trolley
Figure 2.6
3 4
Each time the metal pin moves down, The ticker tape is attached to a trolley which moves on a
it makes a dot on the carbonised ticker F4/2/6
bench or runway. As the trolley moves, it pulls the tape
tape which passes underneath it. through the ticker-timer. A trail of dots is punched on
the ticker tape at equal time intervals. Thus, the dots on
the tape form a complete record of the motion of
the trolley.

Analysing Motion on a Single Strip

The type of motion of the trolley can be inferred from the distance between the dots on the ticker
tape pulled by the trolley.
direction of motion

The distance between two neighbouring dots is equal. The object is moving at a constant/uniform velocity.
F4/2/14(a)

direction of motion direction of motion

The distance F4/2/14(b)


between two neighbouring The distance between
F4/2/14(c)two neighbouring dots is
dots is small. greater.
The object is moving slowly. The object is moving at a greater velocity.

direction of motion direction of motion

The distance between two neighbouring The distance between two neighbouring dots
F4/2/14(d) F4/2/14(e)
dots increases. decreases.
The velocity increases. The velocity decreases.
The object is accelerating. The object is decelerating.

43 Forces and Motion


To Find the Time Interval of Motion and Velocity of an Object SPM
05/P3/(B)

1 The time for 1 dot-space, or 1 tick of time is the time interval between one carbon dot and
the next one on the ticker tape.
dot
number 0 1 2 3 4 5 678 910 20 30 40 50

direction
of one tentick
motion of time
First 10-tick Second 10-tick Third 10-tick Fourth 10-tick Fifth 10-tick
strip strip strip strip strip

Figure 2.7
F
O Since the vibrating pin makes 50 dots in 1 4 Therefore, one 10-tick of time
2

R second, therefore: = 10 0.02 s = 0.2 s


10-tick = 0.2 s
CHAPTER

M Time for moving 50 dot-spaces = 50 ticks = 1 s


1 5 Similarly, one 5-tick of time and one 2-tick of
4 1 tick = s = 0.02 s
50 time can be calculated as shown in Table 2.2.
2 A 10-tick of time is the time interval from dot 6 With the quantities of time interval and
number 0 to dot number 10 on the tape. displacement, we can calculate the velocity of
3 The next 10-tick of time is the time taken to an object.
move from dot number 10 to Fdot number 20.
O
2

R Table 2.2
CHAPTER

M
10-tick strip Time taken to move from A to B Constant velocity,
A 4 = 10 0.02 s s 8 cm
direction of motion B v = =
1 2 3 4 5 6 7 8 9 10 = 0.2 s t 0.2 s
10-tick = 0.2 s = 40 cm s1
8 cm
5-tick strip Time taken to move from P to Q Average velocity,
P direction of motion = 5 0.02 s s 6 cm
Q v = =
1 2 3 4 5 = 0.1 s t 0.1 s
6 cm
5-tick = 0.1 s = 60 cm s1

2-tick strip Time taken to move from R to S Average velocity,


R = 2 0.02 s s 5 cm
direction of motion S v = =
1 2
= 0.04 s t 0.04 s
2-tick = 0.04 s = 125 cm s1
5 cm

To Find the Uniform Acceleration or Deceleration of an Object

7
A trolley is moving down a runway. A strip with six The ticker-timer vibrates at a frequency of 50 Hz.
dot-spaces as shown in Figure 2.8 is obtained. Find the acceleration of the trolley.
Solution
1 2 3 4 5
direction
Frequency, f = 50 Hz
of motion A B P Q 1 tick = 0.02 s
0.5 cm
Average velocity from A to B: u = = 25 cm s1
1.5 cm 0.02 s
0.5 cm
1.5 cm
Average velocity from P to Q: v = = 75 cm s1
Figure 2.8 0.02 s

Forces and Motion 44


Time taken (from the midpoint of AB to the Change in velocity
________________
Acceleration, a =
midpoint of PQ) to produce the change in velocity Time taken
= 5 0.02 s ____ v u
= 0.1 s =
t
Watch out! It is 5-tick, not 6-tick. The average
velocity happens somewhere at the midpoints (75 25) cm s1
______________
=
of AB and PQ respectively. Therefore, the time 0.1 s
taken for the change in velocity is from the
midpoint of AB to the midpoint of PQ. ________ 50 cm s1
=
0.1 s
= 500 cm s2
= 5 m s2
Alternative method F
O

2
The time taken, t to produce the change in velocity can also be found as shown below. R

CHAPTER
First, label the time as an interval of 0.02 s at the dots as shown in the figure below. M

direction
of motion
0 0.02 s 0.04 s 0.06 s 0.08 s 0.10 s 0.12 s

t
0.01 s 0.11 s

t = 0.11 s 0.01 s = 0.1 s

Making a Tape Chart 2 On the tape, lines are drawn across dots
number 0, 10, 20 and so on, from the start of
1 Figure 2.9 shows a ticker tape obtained in an the first clear dot, to mark off in sections 10
experiment. dot-spaces long.
dot 3 The 10 dot-spaces are labelled in order. The tape
number 01 2 3 4 5 6 7 8 9 10 20 30
is then cut at the lines to form 10-tick strips.
1 2 3
direction 4 The 10-tick strips are pasted in order side by
of motion one tentick
of time side on paper, preferably a graph paper, for
easy measurement, to form a tape chart as
Figure 2.9
shown in Figure 2.10.
Velocity, v (or distance moved per tentick) v

8
7
6
5
4
3
2
1

Time in ticks t
0 10 20 30 40 50 60 70 80
0 0.2 0.4 0.6 0.8 1.0 1.2 1.4 1.6 Time in seconds

Figure 2.10
5 The length of each 10-tick strip is the distance the time taken for each strip is the same, i.e.
moved in one 10-tick of time, that is, 0.2 s. 0.2s.
6 The length of the strip represents velocity. The 7 As such, the tape chart is practically a velocity-
longer the strip, the greater the velocity since time graph where the vertical axis is the

45 Forces and Motion


velocity (distance moved per 10-tick), while length actually represents the velocity since the
the horizontal axis indicates the time since time taken for each strip is the same.
each strip starts 0.2 s after the one before. 9 Tape charts can also be made of strips with
8 Some may prefer to label the vertical axis as 5 dot-spaces or 2 dot-spaces.
length of the strips, but remember that this

Activ To determine displacement, velocity and acceleration of a trolley


ity 2.1
Apparatus/Materials Calculation
Trolley, runway, ticker-timer, 12 V power supply, (a) Displacement = x1 + x2 + x3 + x4 + x5 + x6
F ticker tape, cellophane tape and wooden block.
O where x is the length of the strip.
(b) Time taken = 6 10-tick
2

R Arrangement of apparatus
= 6 0.2 s
CHAPTER

M
ticker tape ticker-timer
= 1.2 s
4 trolley
Displacement
12 V a.c. inclined Average velocity = ____________
power runway
Time taken
supply wooden
(x1 + x2 + x3 + x4 + x5 + x6) cm
= _________________________
block

Figure 2.11 1.2 s
F
Procedure O (c) Average velocity for the 1st strip:
2

R x1
___
1 The apparatus is set up as shown
M in Figure 2.11. u= cm s1
CHAPTER

0.2
2 The inclination of the runway is set so that the
4 it is released.
trolley will roll down freely after Average velocity for the 6th strip:
3 A length of ticker tape is passed through the x6
v = ___ cm s1
ticker-timer and attached to the trolley. 0.2
4 The ticker-timer is switched on and the trolley is
released. ime taken for the change in velocity, t
T
5 The ticker tape obtained is then cut into 6 pieces = (6 1) 0.2 s
of 10-tick strips. = 5 0.2 s
6 The strips are pasted side by side on a graph = 1.0 s
paper to form a tape chart.
Results Acceleration of the trolley is calculated from the
velocity (distance moved per tentick) formula:
vu
5 a=
x6
t
6
4
x5
5
Discussion
3
The length of strip increases uniformly.
x4
2
4
Thus, the trolley moves down the runway at a
x3 constant or uniform acceleration.
3
1
x2
2 Conclusion
x1 1
The average velocity and acceleration of a trolley are
thus determined.
Activity 2.1

0 10 20 30 40 50 60 Time in ticks
0 0.2 0.4 0.6 0.8 1.0 1.2 Time in seconds
t

Figure 2.12
F4/2/10

Forces and Motion 46


To prepare a friction-compensated runway
In Activity 2.1, there are two main forces acting on the trolley along the runway, i.e., the component weight of the trolley
down the runway and the frictional force of the runway on the trolley (please refer to Section 2.9 on page 118).
(a) If the slope is not steep enough, (b) If the slope is too steep, the trolley (c) The slope is adjusted until the trolley
after a slight push, the trolley moves moves down the runway by itself. goes down at uniform velocity, after
a short distance and then stops. The dots get farther apart. a slight push. The dots on the tape
The dots on the tape get closer. are equally-spaced. This is a
friction-compensated runway.
Physics Blog
direction of motion
direction of motion
direction of motion
trolley given a slight push
stop trolley goes down on its own F
trolley given a slight push
velo O

2
city cons
Key : incr tant v R
eas elocit
ing y

CHAPTER
component weight friction M

8
F4/2/11b F4/2/11c
A student carried out an experiment using a trolley Average velocity
and a ticker-timer that vibrates at a frequency of ____________________
Total distance travelled
=
50Hz. Figure 2.13 shows a tape chart consisting of Time taken
10-tick strips that he obtained. 30 cm
______
= = 30 cm s1
velocity (distance moved per tentick)
1.0 s
4 = 0.30 m s2
10 (c) Average velocity for the 1st strip:
3 Key :
____ 2 cm
start u = = 10 cm s1
component
weight
8 0.2 s
2 trolley given a slight push friction
6
Average velocity for the 5th strip:
1 ______
stop 10 cm
v = = 50 cm s1 Watch out, not 5 10-tick!
4 0.2 s The time taken for the
2
Time taken, t change in velocity is from
=4 10-tick the midpoint of the 1st strip
to the midpoint of the 5th
0 =4 0.2 s = 0.8 s strip.
0 10 20 30 40 50 Time in ticks
0 0.2 0.4 0.6 0.8 1.0 Time in seconds OR:
t Time taken, t From the time axis.
0.1 0.9
= (0.9 0.1) s = 0.8 s
Figure 2.13 Change in velocity
_________________
Acceleration, a =
Find F4/2/12 Time taken
(a) the total distance travelled,

____ v u
(b) the average velocity, =
(c) the acceleration of the trolley. t

______________ (50 10) cm s1
Solution =
0.8 s
(a) The total distance travelled = 50 cm s2 = 0.5 m s2
(from dot number 0 to number 50)
Note:Since the motion is of uniform acceleration, the
= (2 + 4 + 6 + 8 + 10) cm
average velocity in (b) can also be calculated
= 30 cm u+v
(b) Total time taken from the formula v = .
= 5 10-tick
The total time taken 2
is 1 s if you label the 1
= 5 0.2 s time axis in seconds. v = (10 + 50) cm s =30cm s1
= 1.0 s 2

47 Forces and Motion


In the questions, the vertical axis may not be labelled as As explained earlier, the distance between dots, or the
velocity or distance moved per 10-tick (or 5-tick or length of a strip, represents the velocity of a moving
2-tick). However, the way to solve the problems is still object. Therefore, the increase in distance between dots
the same. and the increase in length between successive strips
represent the increase in velocity. If the increment is
uniform, then the increase in velocity is uniform, that is,
the object is moving with uniform acceleration.
(a) In the figure below, the increase in distance
9 between the successive dots is equal (i.e., 0.2 cm).
Therefore, the acceleration is uniform.
Figure 2.14 shows a tape chart for a trolley which
F
O
moves up on an inclined plane. Determine the direction of motion
2

R deceleration of the trolley.


1.0 cm 1.2 cm 1.4 cm 1.6 cm 1.8 cm 2.0 cm
CHAPTER

M Length of strips (cm)


1
4 2 (b) In the figure below, the increase in length between
6 3 the successive strips is equal (i.e., 1 cm). Therefore,
5 4 the acceleration is uniform. (Alternatively, if a
4 5 straight line can be drawn across all the top
3 midpoints of the strips, the acceleration is uniform)
2 F
Velocity (cm per tentick) Velocity
1
O
2

R 8
7
CHAPTER

0 M Time (s)
0.04 0.08 0.12 0.16 0.20 0.24 6
t 5
4 4
0.02 0.22
3
Figure 2.14 2
1
Time (s)
Solution Time
0 0.2 0.4 0.6 0.8 1.0 1.2
The time of each 2-tick strip = 2 0.02 s
= 0.04 s (c) In the figure below, there is no increase in length
6 cm between the successive strips. Therefore, the
u= = 150 cm s1 acceleration is zero (the object is moving with
0.04 s
constant or uniform velocity).
1 cm
v = = 25 cm s1
0.04 s Velocity (cm per tentick) Velocity
Time taken to produce the change in velocity, t 6
= 5 0.04 s
= 0.20 s = 5 2-tick
Time(s)
OR: t = (0.22 0.02) s Time
If the time axis is 0 0.2 0.4 0.6 0.8 1.0
= 0.20 s labelled.
Acceleration, a
(d) In the figure below, the decrease in length between
v u
____ the successive strips is equalVelocity
(i.e., 2(cm
cm). Therefore,
= per tentick)
t the deceleration is uniform.
(25 150) cm s1 8
= 0.2 s Velocity (cm per 5-tick) 7
Velocity
16 6
125 cm s1 14
= 0.2 s 12 Velocity (cm per tentick) 5
Veloc
10
8 4
= 625 cm s2 6
3
= 6.25 m s2 4
2
6
Time(s) 2
Deceleration = 6.25 m s2 0 Time
0.1 0.2 0.3 0.4 1
Time (
0 0.2 0.4 0.6 0.8 1.0 1.2
0 0.2 0.4 0.6 0.8 1.0 Time
Forces and Motion 48
F4/2/34
The Equations of Linear Motion

For linear motion with uniform acceleration, a, the displacement or Summary of the equations for
distance travelled, s, in a particular direction is given by: linear motion with uniform
s = Average velocity Time taken acceleration:
v = u + at
s = 1 (u + v)t 1
s = (u + v) t
2 2
1 2
s = ut + at
2
rearrange rearrange
v = u + at a = v u t = v u v 2 = u2 + 2as
t a
substitute substitute s = displacement F
u = initial velocity O

2
v = final velocity R
1 1 a = uniform acceleration
( )

CHAPTER
s = (u + v)t 1 s = (u + v) t M
2 s = u + v t 2 t = time interval
1 2
= (u + u + at)t 1 (v u) 4
2 = (u + v) a
2
1
= (2ut + at 2) (v 2 u2)
2 = 1
2 a
1 2 2 2
s = ut + at 2as = v u
2
v 2 = u 2 + 2as

There are 5 physical quantities in the equations of 1


linear motion with uniform acceleration. (II)For s = (u + v) t:
2
Each of the four equations of motion involves 4 physical
quantities. The velocity of a rocket travelling at 2000 m s1
To solve numerical problems, you need to know 3 increases to 6000 m s1 after moves through a
quantities before you can find the value of the fourth distance of 80km. Calculate the time for the rocket
quantity. In the beginning, you may be unsure of which to reach this velocity.
equation to choose to solve the problem. You may start 2000 m s1 6000 m s1
by trial and error. After some practice, you will be able to
select the required equation easily.
80 km
The table below shows some examples.

(I)For v = u + at: Solution


A car accelerates from 20 m s1 with an acceleration u = 2000 m s1, v = 6000 m s1,
of 2 m s2. s = 80 km = 80 000 m, t = ?
What is the velocity after 8 seconds? Unable to
? ? calculate t
Solution 1 because only
First trial: s = ut + at2 2 quantities
2
u = 20 m s1, a = 2 m s2, t = 8 s, v = ? are known.

Unable to ?
? 1 3 quantities
calculate v, Second trial: s = (u + v) t
First trial: v 2 = u 2 + 2as because only 2 are known.
2 quantities are Can be used
(2000 + 6000) to find t.
? known. 80 000 = t
2
Second trial: v = u + at
v = 20 + 2(8) t = 20 s
3 quantities are
= 36 m s1 known. Can be
used to find v.

49 Forces and Motion


10 12
Starting from rest, a sprinter reaches his top velocity By applying the brakes, a driver reduces the
in 3 seconds. He runs a distance of 24 m in the velocity of his car from 20 m s1 to 10 m s1 after
3 seconds. What is his acceleration? a distance of 30 m. Calculate the deceleration of
(Assume his acceleration is uniform.) the car.
Solution
u = 0, s = 24 m, t = 3 s, a = ? Solution
1 u = 20 m s1, v = 10 m s1, s = 30 m, a = ?
Applying s = ut + at 2:
2 Applying v 2 = u2 + 2as:
1 102 = 202 + 2a(30)
F 24 = 0(3) + a(3)2
O 2 100 400
a =
2

R 2(30)
a = 2 24
CHAPTER

M = 5 m s2
9
Deceleration = 5 m s2
4 = 5.3 m s2

F 1 SPM
Clone
07
O
2

11 R The figure below shows the route taken by a van


CHAPTER

M
from town P to town S.
Salina is driving at a velocity of 10 m s1.
Seeing a cow in front, she brakes 4 to stop her car. 3 km
Q R
If the deceleration of the car is 2 m s2, what is 1 km
the distance the car covers before it comes to a S
halt?
5 km

Solution
P
u = 10 m s1 v=0

What is the displacement of the van?


F4/2/14a
A 4.0 km C 7.5 km
s=?
B 5.0 km D 9.0 km
Comments
u = 10 m s1, v = 0, a = 2F4/2/13
m s2, s = ?
2 2 Displacement is a vector quantity. It is the shortest
Applying v = u + 2as:
distance that links the initial position of an object
0 = 102 + 2(2)s
to its final position. In the above question, the
4s = 100
100 displacement is the distance PS. Join P to S, and use
s = Pythagoras theorem to calculate the displacement.
4
= 25 m Q 3 km R
1 km 3 km 1 km
S

4 km
PS = 3 2 + 4 2
= 5 km

P
Dont forget to insert the negative sign for deceleration
when doing the calculation. Answer B
F4/2/14b

Forces and Motion 50


2.1

1 During a class activity, Hashim walks 20 m due east. 6 When a dart is blown from a blowpipe of 1.2m
He reverses his direction and walks 12 m. After that, length, it travels at a speed of 15ms1. Find the
he reverses his direction again and walk for another time taken for the dart to travel in the barrel.
10m. If the total time taken is 30 s, what is his
7 Daniel drives his car at a constant velocity of
speed and velocity?
20ms1. He steps on his brakes to reduce the
2 A speedboat moves due north for 12 km before velocity of the car to 10ms1 after travelling 30 m.
turning east for 8 km. Later, the speedboat moves Calculate the deceleration and the further distance
south for 6 km. What is the displacement of covered before the car stops.
the speedboat from its original position? Find its
average velocity, in m s1, if the total time taken is 8 An aeroplane needs a velocity of 33ms1 for
30 minutes. take-off. F
3 Find the acceleration or deceleration for the tape O

2
v = 33 m s1
R
charts below. The ticker-timer in use vibrates at a

CHAPTER
M
frequency of 50 Hz.
u=0
(a) Velocity (cm per 10 ticks) 4

s
10

If the plane accelerates at 3ms2, calculate the


minimum length of the runway needed by the plane
2 to reach the speed for take-off.
Time
9 Velocity
A baseball pitcher
(cm per holds a baseball in his hand for
10 ticks)
3 m before the baseball is thrown at a velocity of
(b) Length of strips (cm) 39ms1.
10
12
3.0 m
10
8
6
4 39 m s1
2
Time
2
Time

4 A sports car accelerates from rest and covers a


distance of 90 m in 6 s. What is its acceleration?
5 A cyclist starts from rest and reaches a velocity of Find the acceleration of the baseball before it is
20ms1 in 8 s. Calculate his acceleration. released.

2.2 Analysing Motion Graphs

2.2 Analysing Motion Graphs

Motion Graphs

1 Graphs can be useful in studying motion. They show the changes in


the motion of an object with time.
2 There are two main types of linear motion graphs:
(a) the displacement-time graph
(b) the velocity-time graph

51 Forces and Motion


Displacement-time Graphs

1 A displacement-time graph (s-t graph) is a graph that shows how the


Relationship between the
displacement of an object varies with time.
position of an object,
2 Figure 2.15 shows a student cycles at a constant velocity from position
displacement, s, and velocity, v:
A to reach position B, which is 300m away, in 200 seconds. He rests
s = (+)
for 100 seconds at position B and then cycles back to position A using
v = (+)
the same straight path. He reaches position A after another 200
(object on the
seconds. right of O and
N
zero velocity at rest moving
displacement (m)
due east)
positive velocity O
F moving in a fixed direction
negative velocity
O moving in opposite direction s=0 v=0
B B
2

300 and stops at


R s = () s = (+)
the starting point
CHAPTER

M v = () (object on the
(object right with
4 y
on the respect to O)
I II III left of
A x A O and v = ()
time (s) moving (object moving
O 100 200 300 400 500
due due west)
F
Figure 2.15 west)
O
F4/2/15
2

R
CHAPTER

4 In Section III of the graph:


From conclusion 1:
In Section I of the graph: Velocity = Gradient
To find the velocity, use the formula: (0 300) m
Change in displacement =
Velocity = (500 300) s
Time taken
= 1.5 m s1
300 m
Velocity, v = The negative sign shows that the direction of
200 s motion is opposite to its original direction.
= 1.5 m s1 Take note that velocity is a vector quantity.
Gradient of the graph At t = 500 s, the graph intersects the t-axis.
y The displacement at this moment is zero, that is,
= the student has returned to the original position.
x
(300 0) m
=
(200 0) s
= 1.5 m s1 In Section II of the graph, a horizontal line is drawn from
t = 200 s to t = 300 s. During this period, the student
remained at position B, which is 300 m away from position
A, the origin, with a velocity of zero.

Conclusion 1 Conclusion 2
On a displacement-time On a displacement-time graph,
graph, the gradient of the a horizontal line (gradient = 0)
graph is equal to the shows that an object is
velocity of the object. stationary, i.e., not in motion.

Forces and Motion 52


Velocity-time Graphs SPM
03/P1
SPM
04/P1
SPM
05/P1
SPM
07/P1
SPM
08/P1
SPM
09/P1

1 A velocity-time graph (v-t graph) is a graph that shows the variance in the velocity of an object against
time.
2 A car starts from rest and accelerates for 20 seconds until it reaches a velocity of 30 m s1. The driver
maintains this velocity for 20 seconds. The velocity of the car is then reduced until it stops at t = 60 seconds.
3 The graph in Figure 2.16 shows how the velocity of the car changes against time.
v = 0 m s-1 v = 30 m s-1 v = 30 m s-1 v = 0 m s-1

t=0 t = 20 s t = 40 s t = 60 s
starts stops
v (m s-1)

F
constant velocity O

2
30 To determine the acceleration of the car R

CHAPTER
From conclusion 1: M
acceleration deceleration Acceleration, a
(positive y y (negative 4
I II III = Gradient
gradient) gradient)
(0 30) m s1
= (60 40) s
x x
t (s) The negative
0 20 40 60
= 1.5 m s2 sign indicates
The deceleration of deceleration.
FigureF4/2/17
2.16 the car is 1.5 m s2.

To determine the acceleration of the car


Using the formula:
Change in velocity
Acceleration= Conclusion 3
Time taken
vu A negative gradient indicates deceleration.
Acceleration, a =
t
(30 0) m s1
= 20 s Distance travelled from t = 20 s to t = 40 s:
Distance = Velocity Time
= 1.5 m s2 = 30 m s1 20 s
y = 600 m
Gradient of the graph =
x Area under the graph (for section II)
(30 0) m s1 = 30 20
= = 600 unit2
(20 0) s
1.5 m s2
=

Conclusion 4
Conclusion 1
On a velocity-time graph, the area under
On a velocity-time graph, the gradient of the the graph is numerically equal to the distance
graph represents the acceleration of the object. travelled.

The car travels at a constant velocity of 60 m s1


from t = 20 s to t = 40 s. A horizontal line is
shown in the graph. This method of calculating the distance travelled
can be applied to any velocity-time graph,
whether the velocity is constant or not.
For example, in Section I of the graph, the area
1
Conclusion 2 of the shaded triangle (= base height)
2
On a velocity-time graph, a horizontal line equals 300. So, the car travelled a distance of
(gradient = 0) represents a constant velocity. 300 metres in the first 20 second of its motion.

53 Forces and Motion


13
Figure 2.17 shows the velocity-time graph of a Solution
motorcycle travelling along a straight road between (a) The line OA shows that the motorcycle accelerates
two traffic lights. uniformly from 0 m s1 to 12 ms1.
The line BC shows that the motorcycle decelerates
uniformly before coming to a rest.
velocity (m s1)
(b) The motorcycle is moving at a constant velocity
for 10 s (line AB of the graph).
(c) Distance between the two traffic lights
A B = Area under the graph
12
1
F = (10 + 25) 12 = 210 m
O 2
2

R (d) From 0 s to 5 s:
CHAPTER

M
12 0
C Acceleration = = 2.4 m s2
4 time (s) 50
O 5 10 15 20 25
From 5 s to 15 s:
Figure 2.17
Acceleration = 0 because the
object is moving
(m From
velocity s1) 15 s to 25 s: at constant
(a) Explain the motion of the F motorcycle as
Acceleration velocity
represented by the lines OAO and BC on the A
0B 12
12 = = 1.2 m s2
2

graph. R
25 15
CHAPTER

(b) What is the time interval Mduring which the Thus, the acceleration-time graph is as shown below.
motorcycle is moving at a constant velocity?
4 C 2
(c) What is the distance between the two traffic O 5 10 acceleration 15 20 25
(m s time
) (s)

lights? 2.4
(d) Sketch an acceleration-time graph to represent
time (s)
the motion of the motorcycle between the two
1.2
traffic lights.

Non-uniform Velocity

displacement, s (m) 1 Figures 2.18(a) and (b) show the


positions of a ball falling from rest
50
55
24
27
60
30
3

6
5
10
and its corresponding displacement-
t=0 time graph.
21 9

18 12
15

45 15

40 20
35 25
30

s1 2 The ball covers a longer distance in


55
60
5
gradient is steeper the second second as compared with
greater velocity
30
27 3

50
24

10

t=1s the first second. The ball is moving


21 9

18 12
15

45 15

40
35
30
25
20
gradient
s2
= velocity with non-uniform velocity.
(at t = 1 s)
3 To determine the balls instantaneous
velocity, for example, at t = 1 s or
s t = 2 s, a tangent must be drawn at the
50
55
24
27
60
30
3

6
5
10
t
respective points on the graph, as
t=2s
21 9

shown in Figure 2.18(b).


18 12
15

45 15

time, t (s)
40 20
35 25
30

O 1 2
4 The gradient of the tangent is equal
(a) (b) to the velocity.
5 However, the stone is falling with
Figure 2.18
F4/2/18 uniform acceleration.

Forces and Motion 54


Non-uniform Acceleration

acceleration constant velocity


1 Figure 2.19 shows an
athlete running a 100 m
track and the corresponding
velocity-time graph of his
motion.
2 The athlete increases his
velocity, v (m s-1)
constant velocity velocity (accelerates) until
the maximum velocity.
smaller gradient gradient = 0 He maintains the
smaller acceleration zero acceleration maximum velocity to
F
finish the race. O

2
gradient of tangent
3 The acceleration of the R
athlete can be determined

CHAPTER
v = acceleration at this instant M
by drawing tangents at the
t 4
respective points and
time, t (s) determining the gradients
O t1 t2
Figure 2.19
of the tangents.

F4/2/19

A comparison between the displacement-time graph and the velocity-time graph:

Displacement-time graph Velocity-time graph


displacement
velocity

A
time time

The object moves at a Non-horizontal


The object moves at a constant acceleration.
constant velocity. straight line

Represents the velocity of


Gradient Represents the acceleration of the object.
the object.

The object is stationary. Horizontal line The object moves at a constant velocity.

The object returns to its Intersection on


The object stops.
original position. the time-axis

Positive The object moves


in a specific Sign of the Positive Acceleration
direction. gradient Negative Deceleration
Negative The object moves (positive or
in the opposite negative)
direction.

Area under the Numerically equals the distance travelled by the


No significance
graph object

55 Forces and Motion


14
The velocity-time graph in Figure 2.20 shows a particle (a) 10 s (from t = 0 s to t = 10 s, the velocity is
starting from rest and travelling east. positive).
v ( m s 1 )
(b) 4 s (from t = 16 s to t = 20 s, the velocity is
negative).
B
20 (c) Distance travelled while moving towards the
east, S1 = Area of triangle ABC
10
1
C P R = 10 20
A t ( s) 2
5 10 16 18 20
= 100 m
10
F Q Distance travelled while moving towards the west,
O S2 = Area of triangle PQR
2

R
Figure 2.20 1 = 4 10
CHAPTER

M 2
(a) How long does the particle travel towards the east?
4
= 20 m
(b) How long does the particle travel towards the west?
(c) Find the average speed and the average velocity. Total distance
Average speed =
Solution Time taken
100 + 20
To understand the above situation better, please refer =
F
to the figure below which demonstrates the positions 20
O
6 m s1
=
2

of the particle that vary with time. R


CHAPTER

100 m M Final displacement


v=0 v = 20 m s1 v=0 Average velocity =
A B 4 C Time taken
t=0s t=5s t = 10 s
100 20
=
N
v=0 v = 10 v=0 20
R Q P = 4 m s1 to the east
t = 20 s t = 18 s t = 16 s
(Please note that in the situation discussed, the
20 m
particle is always to the east of its original
Note: The particle is at rest from t = 10 s to t = 16 s. position.)
F4/2/21a

Area under velocity-time graph


v
An object moving at a decreasing acceleration is
moving at a decreasing velocity.

A
t
B
The velocity of an object with a decreasing acceleration
is always increasing. However, the rate of increase is
Area A (above the t-axis) getting smaller.
= Distance travelled by the moving object in its original
direction
Area B (below the t-axis)
=Distance travelled by the moving object in the
opposite direction
Total distance travelled = Area A + Area B
Final displacement = Area A Area B

Forces and Motion 56


2 SPM
Clone
11

The diagram shows the velocity-time graph of a toy A 0 m C 9m


car in motion. B 6 m D 15 m
velocity(m s-1) Comments
6 Displacement = Area above the time-axis Area below
4 the time-
2 axis
0 time (s)
1 1
= (6)(2 + 3) (2)(6)
-2 2 2
-4
= 15 m 6 m F
= 9 m O

2
-6
R

CHAPTER
M
What is the displacement of the toy car in 6 s? Answer: C
4

2.2
s (m)
1 The figure 3 The figure below shows the velocity-time graph of a
shows the 20
motorcycle starting from rest and travelling north.
displacement-
15
time graph v (m s1)
10
of a moving
5 12
particle.
t (s)
O 3 10 15

15 20
(a) What is the velocity of the particle in the initial O t (s)
6 10 13
period of 3 seconds?
(b) How long is the particle stationary?
(c) At what point in time does the particle return to 10
its original position?
(d) Calculate (a) What is the deceleration from t = 10 s to t = 13 s?
(i) the average speed, and (b) What is the displacement of the motorcycle
(ii) the average velocity of the moving particle. during the first 13 s?
(c) For how long is the motorcycle travelling towards
2 The the south?
v (m s1)
velocity- (d) What is the final displacement of the motorcycle
time graph at t = 20 s?
shows the 12 (e) What is the average velocity of the motorcycle for
movement the whole journey?
of a particle. 4 A sports car starting from rest, accelerates uniformly
t (s)
to 30 m s1 over a period of 20 s. The car maintains
O 3 6 10 the velocity for 30 s. The velocity is then reduced
(a) What is the total distance, in m, travelled by the uniformly to 20 m s1 in 10 s and then brought to
particle in 10 seconds? rest after another further 10 s.
(b) For how long is the particle moving with constant Draw a velocity-time graph to represent the journey
velocity? as described above. From the graph, find
(c) Calculate the ratio of acceleration : deceleration. (a) the acceleration of the car for the first 20 s,
(d) Calculate the average velocity of the particle. (b) the distance travelled, and
(c) the average velocity over the time described.

57 Forces and Motion


2.3 Understanding Inertia

2.3 Understanding Inertia


SPM SPM
Concept of Inertia 05/P1 09/P1

1 All objects tend to continue with what they are doing.


2 Newton's first law of motion (also known as the Law of Inertia)
states that:

Every object continues in its state of rest or uniform speed in


a straight line unless acted upon by an external force.

F 3 The tendency of an object to maintain its state of rest or uniform


O motion in a straight line is called inertia.
2

R 4 Only an external force (or a non-zero net force) can cause a change to
CHAPTER

M
the state of motion of an object (either at rest or moving at the same
4 speed in a straight line).
5 The external force is a non-zero net force, if more than one external
force act on the object.
SPM
Situations Involving Inertia 09/P1

F
O
1 When a boy is riding a bicycle that runs over a stone, he is
2

R
thrown forward and off the bicycle.
CHAPTER

M
Explanation
4
The stone abruptly stops the motion of the bicycle, but the
inertia of the boy keeps him in the forward motion and
throws him forwards.

F4/2/23
A cardboard is placed on the rim of a glass and a coin is
2
placed on top of the centre of the glass. When the
cardboard
cardboard is pulled away quickly, the coin resting on the
coin
cardboard drops straight into the glass.
Explanation
The inertia of the coin maintains its state of rest. When the
card is pulled away, the coin falls into the glass due to gravity.
Note:
If the card is pulled away slowly, the frictional force
between the coin and the card causes the coin to accelerate
so that it moves together with the card. However, if the card
is pulled quickly, the time is too short for the friction to
cause any appreciable movement of the coin.
3 If thread Y is pulled slowly, thread X will snap. If thread Y is
thread X pulled suddenly, then thread Y will snap.
(can withstand 10.5 N)
Explanation
When thread Y is pulled slowly, the additional force to the
weight (10 N)
weight causes thread X to snap when the tension exceeds
thread Y snaps
the breaking force. When thread Y is pulled abruptly, the
weight maintains its state at rest due to inertia. Thread Y
thus stretches and snaps. (If the weight does move, it will
stretch and snap thread X instead.)

Forces and Motion F4/2/24 58


4 When a stationary bus starts to move forward with an
acceleration, the passengers are thrown backwards.
Explanation
The passengers in the bus are originally in a stationary
state. When the bus starts to move forward with an
acceleration, the inertia of the passengers keeps them in
their position. Thus, the passengers are thrown backwards.

F
O

2
R

CHAPTER
M

5 When a moving bus stops suddenly, the passengers lurch 4


forwards.
Explanation
The passengers are in a state of motion when the bus is
moving. When the bus stops suddenly, the inertia of the
passengers keeps them in motion. Thus, the passengers
lurch forwards.

6 A weight is suspended by a fine thread and rests at point P.


When the weight is raised and then released, the thread
fine
thread
snaps when it passes point P.
thread
snaps Explanation
The fine thread is able to support the weight when it is
stationary. When the weight is allowed to fall, the weight
P
maintains its inertia of moving downwards when it passes
point P. The inertia of the weight causes an additional force
weight to exert on the thread, thus causing it to snap.

F4/2/25
7 If a book is pulled out very quickly from the middle of a
BO
OK

BO

pile of books, the books above it will drop instead of


OK

BO
OK

moving along with it.


Explanation
The inertia of the books above keeps them in their original
position.
F4/2/26
Again, if the book is pulled out slowly, the books above it will
move together with the book as explained in situation 1 .

59 Forces and Motion


8 The decorative item hanging from the rear-view mirror
swings backwards when a stationary car starts to move or
accelerates.
Explanation
The inertia of the decorative item keeps it in its original
position while the car moves forwards.

9 The decorative item swings forwards when the moving car


comes to a sudden halt.
Explanation
F
O
The inertia of the decorative item keeps it in motion when
2

R the car stops suddenly.


CHAPTER

F object will not change its state of motion unless forced to do so.
Inertia = Laziness literally, such that an
O
2

R
CHAPTER

4 I am in
I am at rest, motion.
I will always I will always
stay at rest. move at
constant
speed in a
straight line.


Force is needed to change its velocity, i.e.,
(a) to get the object to start moving, or
(b) to make the object move faster, slower or change the direction of motion.

Hard-boiled or raw?
On a table are two eggs, one raw and one hard-boiled.
How can the two eggs be distinguished?

Solution
The answer is to spin the egg.
Spin each egg on its side. It is much easier to spin the hard-boiled egg.
A hard-boiled egg is solid whereas a raw yolk floats in the liquid white
in a raw egg. When a hard-boiled egg is spun, the solid contents turn
together with the shell. When a raw egg is spun, the yolk and the liquid
white tend to remain at rest because of inertia. The reluctance to spin
along with the shell makes the raw egg harder to spin.

Forces and Motion 60


SPM SPM
Relationship between Mass and Inertia 04/P1 07/P1, P2

1 A shopper in a supermarket observes that it is always easier to start


moving an empty trolley than a full trolley.

Supertankers: Large crude oil F


carriers O

2
R

CHAPTER
Massive oil tankers carry M

(a)Empty trolley (b)Full trolley crude oil ranging from 150 to


4
300 thousands tons. With full
Figure 2.21
power on, a tanker with a
very large inertia takes a long
2 Similarly, it is easier to stop an empty trolley than a full one if both are time to accelerate to its
moving at the same speed towards the shopper. maximum speed. The tanker
3 The more mass an object has, the harder it is to change its state of might need to travel a
motion. distance of 10km before
For example, coming to a stop even with
(a) It is more difficult to start moving a bucket filled with sand. the engine set into reverse
(b) When both the buckets are swinging and an attempt is made to stop condition (propeller in
them at the lowest point of the swing, it is more difficult to stop the reverse rotation).
bucket filled with sand. It is not an easy job to turn
the tanker around since the
inertia of the crude oil is
ceiling
enormous. Supertanker
rope of officers need special training
the same in the handling of heavily
length It is harder
to start it
loaded ships.
sand moving from
rest

It is harder
to stop it
empty bucket filled here with
bucket with sand your hand

Figure 2.22
F4/2/28

(c) This shows that the bucket with more mass offers a greater resistance
to change from its state of rest or from its state of motion.
4 By the same reasoning, it is harder to start a bowling ball moving and
harder to stop it than a hollow rubber ball of the same size.
5 Thus, an object with a larger mass has a larger inertia.
Mass is a measure of the inertia of a body.
6 However, inertia is a phenomenon. It has no unit even though it is
closely related to mass.

61 Forces and Motion


2.1
SPM SPM
04/P3(A) 06/P3(B)
Relationship between inertia and mass
Situation Procedure
Mrs Tan and her son Siao Yang are sitting on two 1 One end of the hacksaw blade is clamped by a
similar swings. G-clamp to a leg of a table as shown in Figure 2.24.
2 A 50 g plasticine ball is fixed to the free end of
the blade.
3 The free end of the blade is displaced horizontally
and released so that it oscillates. The time for 20
F complete oscillations, t20, is measured using a
O stopwatch. This step is then repeated. The average
2

R of t20 is calculated. Then, the period of oscillation,


CHAPTER

M
Figure 2.23 t
T is determined by using T = 20 .
4 Mr Tan gives each of them a push while they are at 20
rest and later tries to stop them in the same position. 4 Steps 2 and 3 are repeated with plasticine balls of
Mr Tan finds that it is harder to push as well as to masses 75 g, 100 g, 125 g and 150 g.
stop Mrs Tans motion. 5 A graph of T against m is drawn.
Inference F
Tabulation of data
The inertia of an object depends onO its mass.
2

R
Hypothesis M Table 2.3
CHAPTER

An object with a larger mass has a larger inertia Mass of Time of 20 oscillations, Period of
(represented by a longer period of 4oscillation). load, t20 (s) oscillation,
Aim t
m (g) t t Average T = 20 (s)
To investigate the relationship between mass and 1 2 20
inertia using an inertia balance 50
Variables 75
(a) Manipulated : mass of plasticine, m 100
(b) Responding : period, T
(c) Fixed : the stiffness of a hacksaw blade 125
and the distance of the centre of 150
the plasticine from the clamp
Graph
Notes
A larger inertia means it is more difficult for the T(s)

load to start or stop, thus making the time for one


oscillation, T longer. Therefore, the period of
oscillation represents the inertia.
Apparatus/Materials
Hacksaw blade, G-clamp, stopwatch, and plasticine
balls of mass 50 g, 75 g, 100 g, 125 g and 150 g. m (g)
Arrangement of apparatus Figure 2.25
Experiment 2.1

Conclusion
G-clamp
The graph of T against m in Figure 2.25 shows that
hacksaw blade the period increases with the mass of the load i.e. an
plasticine object with a larger mass has a larger inertia.
The hypothesis is valid.
Figure 2.24

Forces and Motion 62


Effects of inertia

Animals such as dogs and If the bottom of the handle of a hammer is knocked
cats shake their bodies downwards against a hard surface, the loose head of the
vigorously to dry their hammer tightens in its wooden handle. The hammer head
wet fur. The droplets of continues with its downward motion after the handle has
water on the fur tend to come to a stop. This causes the upper part of the wooden
continue in motion when handle to slot deeper into the
the fur are reversed in hammer head. The chilli sauce in a bottle
direction during shaking. can be forced out easily by
As a result, water droplets turning the bottle upside
are separated from the fur down and giving it a quick
and fall off. downward shake, followed
by a sudden stop. The sauce
continues its downward
movement due to its inertia
when the bottle is stopped
Applications causing the sauce to be forced

63
of the out of the bottle.
concept of
inertia
When chased by a bull, run in a zigzag pattern. Droplets of water on
The larger inertia of the bull makes it more a wet umbrella can be
difficult for the bull to turn around spun off if the
continuously. umbrella is rotated
vigorously and
stopped it abruptly.
Owing to inertia,
water droplets on the
Sumo wrestlers are heavy. umbrella continue to
This is an advantage because the When the branch of an apple tree is shaken, the move even though the
larger inertia forms a greater apples fall to the ground. The apples which are umbrella has stopped
resistance to his opponent who is stationary tend to remain at rest when the branch spinning.
trying to topple him. is shaken. As a result, the stalks are strained and
the apples break away from the branch.

Forces and Motion


CHAPTER 2
F

4
R
O

M
Ways of Reducing the Negative Effect of Inertia SPM
07/P2

1 If a car crashes while travelling at a fixed velocity of 30 m s1, for


example, the inertia of the passengers causes them to continue moving
at 30 m s1 until a force acts to change this speed. This is a
dangerous situation. Upon impact, the passengers will crash into
the parts of the car immediately in front of them and suffer injuries
(Figure 2.26).
2 Ways to reduce the negative effect of inertia in a car:

(a) Safety belt


Seat belts secure the driver
F and passengers to their seats.
O When the car stops suddenly,
2

R
the seat belt provides the
CHAPTER

M
external force that prevents Figure 2.26
F4/2/30
4 the driver or passengers from
being thrown forwards.

(b) Airbag system


Airbags mounted on the
F dashboard or steering wheel
O inflate automatically when a
2

R
collision occurs. This prevents
CHAPTER

M
the driver or passengers from
4 crashing into the dashboard.

3 Lorries that carry heavy loads utilise the following features to reduce the negative effects of inertia:

(a) Strong structure behind the drivers cabin If a loaded lorry stops abruptly, its heavy load, for
strong iron structure
example, timber logs, will continue to move
forward towards the drivers cabin because of its
massive inertia. A strong iron structure between
the drivers cabin and the load ensures the drivers
safety.

(b) Subdivision of the mass


F4/2/31 to reduce its inertia The figure shows a liquid product being carried in
three separate compartments instead of one. This
reduces the effect of inertia of the liquid on the
walls of each container if the lorry stops suddenly.

(c) Fastening of the objects to the carrier Big items such as furniture or large electrical
appliances (the objects) must be securely fastened
to the lorry (the carrier) so that the objects become
part of the carrier. This will ensure that the objects
and the carrier always move and stop together.
The objects will not fall off the lorry when the lorry
starts moving, or move forwards when the lorry
stops suddenly.

Forces and Motion 64


3 SPM
Clone
10

The diagram shows two trolleys P and Q with two R S


identical wooden blocks R and S placed on them.
A Pushed to the right Pushed to the left
Trolley P moves and collides with the stationary
trolley Q. B Pushed to the right Pushed to the right
C Pushed to the left Pushed to the right
R S
D Pushed to the left Pushed to the left

P Q
Comments
The collision of the trolleys causes trolley P to slow
Direction of movement of P Stationary down while trolley Q to start moving. Due to F
inertia, R continues to move fast to the right, and O

2
R
Key : Right appeared to be pushed to the right while S, reluctant

CHAPTER
Left M
to move along with trolley Q, appeared to be
pushed to the left. 4
What happens to wooden block R and S?
Answer A

2.3

1 In a bus moving with a uniform speed in a straight 4 A circus strongman slams a hammer and breaks a
line, a boy drops a steel sphere from rest outside the brick over the hand of a clown. The clown feels no
window. He observes that the steel sphere drops pain. Why?
vertically downwards. Explain. (Caution: Please do not attempt this at home.)
2 A durian is originally placed in a lorry as shown in
Figure (a). When the lorry starts to move with forward
acceleration, the durian rolls backwards. Later, when
the lorry stops suddenly, the durian rolls forwards as
shown in Figure (b). Explain the observations.
hammer

hand brick
on floor
(a)
(a)
(a)
5 Suria runs along a track from P to R through Q while
holding a pail full of water. At which points of the
track will more water likely to be spilt? Give your
explanation.

(b)
(b)
(b) Q
3 The figure below shows two blocks of the same
dimensions but of different materials. One is heavier R
F4/2/32
than the other. WithoutF4/2/32
lifting the two blocks, explain
one way to identify the heavier block.

F4/2/34

65 Forces and Motion


2.4 Analysing Momentum

2.4 Analysing Momentum

What is Momentum?
1 If a loaded lorry and a car are moving at v

the same speed, it is more difficult for


the lorry to stop (Figure 2.27).
2 This is because the lorry possesses a
physical quantity, momentum, more v
than the car.
3 All moving objects possess momentum. Figure 2.27
F 4 Activity 2.2 will help you to gain the idea
O of momentum by comparing the effects
2

R of stopping two objects.


CHAPTER

4
Activ To compare the effects of stopping two objects in motion
ity 2.2

Apparatus/Materials One steel ball and one wooden ball of the same diameter, 2 slabs of plasticine.
Arrangement of apparatus F
steel
O
2

R Note
steel
CHAPTER

M steel
Note
Although both balls
4 100 cm wood
The steel ball have the same
released from a diameter, the steel
greater height strikes 50 cm 50 cm ball has a greater
the plasticine at a plasticine mass than the
greater velocity. wooden ball.
(a) (b)
Figure 2.28

(b) Two objects of different masses moving at


(a) Two objects of the same mass moving atF4/2/96
different velocities the same velocity
Procedure Procedure
1 A steel ball is first released from a height of 1 A steel ball and a wooden ball of the same
50cm and then from 100 cm above a slab of diameter are released from a height of 50 cm
plasticine as shown in Figure 2.28(a). above a slab of plasticine as shown in Figure
2 The depths and sizes of the cavities caused 2.28(b).
by the steel ball on the slab are observed and 2 The depths and sizes of the cavities formed
compared. are observed and compared.
Observations Observations
The depth and size of the cavity caused by the The depth and size of the cavity formed by the
steel ball released from a greater height is deeper steel ball is deeper and larger.
and larger.
Activity 2.2

Conclusion
The moving balls produce an effect on the plasticine which is there to stop the motion. The greater the mass
or the velocity of the moving object is, the greater is the effect (the depth and size of the cavity), the greater is
the momentum.

Forces and Motion 66


Linear Momentum

1 Activity 2.2 also shows that it is always harder to stop a massive object Momentum and inertia are
moving at a high velocity. not the same.
2 The above activity serves to explain a concept in physics called
Momentum Inertia
momentum.
3 The linear momentum, p, of a m Inertia depends solely on
mass, m, moving at a velocity, mass while momentum as
v, is defined as the product of v
mass in motion depends on
mass and velocity. both mass and velocity.
Figure 2.29
F4/2/36 F
Momentum =Mass Velocity O
Unit=kilogram metre per second

2
p =mv =kg m s1
R

CHAPTER
M
4 The unit of momentum is kg m s1.
5 Momentum is a vector quantity with the same direction as velocity. 4

6 If the direction to the right is denoted as positive, an object moving to


the right possesses a positive momentum while an object moving to
the left will have a negative momentum.

15
A ball of mass 0.8 kg strikes a wall at a velocity of (a) Take the direction to the right as positive.
10 m s1 and rebounds at 6 m s1. Momentum of the ball before striking the wall, p1
What is its momentum =mv
(a) before it strikes the wall, and =0.8 10
(b) after the rebound? =8 kg m s1
Solution 10 m s1 (b) Momentum of the ball after rebound, p2
=mv
=0.8 ( 6)
= 4.8 kg m s1
Since direction to the
right is positive, direction
6m s1 to the left is negative.

Conservation of Momentum 4 To be precise, the Principle of conservation of


momentum is true for a closed system.
A closed system is one where the sum of
1 The term conservation is used if the total
external forces acting on the system is zero.
amount of matter or quantity remains the same
5 The principle shall be discussed in two
before and after the occurence of an event.
situations as shown in Table 2.4.
2 The Principle of conservation of momentum
states that: Table 2.4
A collision An explosion
The total momentum of a system The total momentum of the The sum of the
is constant, if no external force objects before a collision momentums
acts on the system. equals that after the remains as
collision. zero after an
3 An example of an external force is friction. explosion.

67 Forces and Motion


SPM SPM
Collisions 06/P1 08/P1

There are two types of collisions.

Collisions

Elastic collisions Inelastic collisions

Two objects collide and move apart after a Two objects combine and stop, or move together
collision. with a common velocity after a collision.
F m1 m2 m1 m2 m1 m2 m1 m2
O u1 u2 v
u1 u2 v1 v2
2

R
CHAPTER

M
Momentum is conserved. Momentum is conserved.
4 Total energy is conserved. Total energy is conserved.
Kinetic energy is conserved. Kinetic energy is not conserved:
The total kinetic energy after the collision is less
than the total kinetic energy before the collision.

Formula: F Formula:
m1 u1 + m2 u2 = m1 v1 +Om2 v2 m1 u1 + m2 u2 = (m1 + m2) v
2

R
CHAPTER

Most collisions are inelastic as a significant amount of kinetic energy is converted to other forms of energy
during the collision. However, the collisions involving two objects that bounce off one another with little
deformation during collision are approximate elastic collisions. Collisions between air molecules are elastic.
Examples of approximate elastic collisions are as follows:

Approximate elastic collisions

PhysicsBlog
Collision of steel balls of equal mass Collision of two snooker balls of equal mass
Before collision After collision Before collision After collision
stationary stationary

P
QR S T P QP
RQS RS
T T

Ball P is pulled to the side and then released so as to Ball P is hit and moves with velocity u. It is observed
fall back and strike ball Q. It is observed that ball P that ball P stops when it collides with ball Q which
stops, but ball T swings out to the same height from moves away with velocity u.
which the ball P was released. This shows that ball Q, after the collision, has the
This shows that ball T possesses the same amount of same amount of momentum and kinetic energy as
momentum and kinetic energy as P before it struck ball P before the collision.
ball Q.

Forces and Motion 68


Activ To verify the principle of conservation of momentum in
ity 2.3 (a) elastic collisions, and (b) inelastic collisions

Apparatus/Materials
Ticker-timer, 12 V a.c. power supply, runway, 4 trolleys, wooden block, ticker tape, cellophane tape,
and plasticine.

(A) Elastic collision


Arrangement of apparatus
spring-loaded piston
ticker-timer trolley A
ticker tape trolley B
mA
mB
F
friction-compensated
runway
O

2
R

CHAPTER
M
12 V a.c. wooden
power supply block 4

Figure 2.30
Procedure
1 The apparatus is set up as shown in Figure 2.30.
2 The runway is adjusted so that it is friction-compensated.
3 Two trolleys of equal mass are used. Trolley A with a spring-loaded piston is placed at the higher end of the
runway while trolley B is placed halfway down the runway.
4 A ticker tape is attached to trolley A and another to trolley B. Both ticker tapes are allowed to pass through the
ticker-timer.
5 The ticker-timer is switched on and trolley A is given a slight push so that it moves down the runway at
a uniform velocity and collides with trolley B.
6 After the collision, the two trolleys move separately.
7 From the ticker tapes, the velocities of trolleys A and B before and after the collision are calculated.
8 Assuming that the mass of each trolley is 1 unit, the momentum before and after the collision is calculated
and recorded in a table.
9 The experiment is repeated using:
(a) 1 trolley to collide with 2 stationary stacked trolleys,
(b) 2 stacked trolleys to collide with 1 stationary trolley.
Results
1 Ticker tapes obtained:
(a) Trolley A
after during before
collision collision collision

direction
of motion
x2 x1
x2 x1
vA = uA =
0.2 0.2

(b) Trolley B
after during before
collision collision collision
Activity 2.3

direction
of motion
x3 uB = 0
x
vB = 3
0.2

Figure 2.31

69 Forces and Motion


Tabulation of data

Table 2.5

Before collision After collision


Initial total Final total
mA mB uA momentum, vA vB momentum,
mA uA mA vA + mB vB
1 1
1 2
2 1
F
O
2

R
CHAPTER

M Disscussion
4 1 The spring-loaded piston acts as a springy buffer in the collision in order to make the trolley bounce off the
other one.
2 Strictly speaking, this collision is not a perfect elastic collision as part of the kinetic energy of the colliding
trolley changes to sound or heat energy during the collision.

F
Conclusion O
2

From Table 2.5, it is found that: R


CHAPTER

Total momentum
4
before collision = Total momentum after collision

The principle of conservation of momentum is verified.

(B) Inelastic collision


Arrangement of apparatus

ticker-timer
ticker plasticine
tape
trolley A
trolley B
12 V friction-compensated
a.c. power runway
supply

wooden block

Figure 2.32

Procedure
1 Some plasticine is pasted onto trolleys A and B (both without a spring-loaded piston) as shown in Figure 2.32.
2 A ticker tape is attached to trolley A only.
3 The ticker-timer is switched on. Trolley A is given a gentle push so that it moves down the runway to collide
with trolley B which is stationary halfway down the runway.
4 After the collision, trolley A attaches itself to trolley B and they move together.
Activity 2.3

5 From the ticker tape obtained, the velocity of trolley A before the collision, and the common velocity of
trolleys A and B after the collision are determined. The initial velocity of trolley B is zero.
6 The experiment is repeated using:
(a) 1 trolley to collide with 2 stationary stacked trolleys,
(b) 2 stacked trolleys to collide with 1 stationary trolley.

Forces and Motion 70


Results
Ticker tape obtained: during
collision
before
after
collision
collision
direction
of motion
x2 x1
x2 x1
v = u =
0 .2 0.2

Figure 2.33
Tabulation of data
Table 2.6 F
O
Before collision After collision

2
R
Initial Initial Initial total Final Final Final total

CHAPTER
M
mass, velocity, momentum, mass, velocity, momentum,
mA u (mA u) (mA + mB) v (mA + mB) v 4

1 1+1=2
1 1+2=3
2 2+1=3

Conclusion
From Table 2.6, it is found that:
Total momentum before collision = Total momentum after collision
The principle of conservation of momentum is verified.

16
An astronaut of mass 90 kg moves at a velocity of 6 m s1 and bumps into a stationary astronaut
of mass 100kg. How fast do the two astronauts move together after collision?
90 kg 100 kg v
6 m s1 at rest 100 kg
90 kg

(a) Before collision (b) After collision



Figure 2.34

Solution
This is an inelastic collision.
Total momentum before collision = Total momentum after collision
m1 u1 + m2 u2 = (m1 + m2)v
(90 6) + (100 0) = (90 + 100)v
540
Activity 2.3

v is the common velocity


v = of the two astronauts.
190
= 2.8ms1
The two astronauts move at the speed of 2.8 m s1 after the collision.

71 Forces and Motion


1 1
3ms 7ms

17
A 50 kg skater is moving due east at a speed of 3ms1 before colliding into another skater of mass
60kg moving in the opposite direction at a speed of 7 m s1. After the collision, the two skaters hold
on to each other. In which direction will they move? What is the speed of the two skaters?
1 1
3ms 7ms

v=?

F
O
2

R
(a) Before collision (b) After collision
CHAPTER

M v=?
Figure 2.35
4
Solution
Total momentum before collision = Total momentum after collision
m1 u1 + m2 u2 = (m1 + m2)v
50 3 + 60 (7) = (50 + 60) v
F 150 420 = 110v
v = 2.5ms O 1
v is the common velocity
2

R
of the two skaters.
CHAPTER

Momentum to the east takenMas positive, so


momentum to the west is negative.
4
The two skaters will move to the west at a speed of 2.5m s1.

18
A trolley of mass 3 kg moving at a velocity of 2ms1 collides with another trolley of mass 0.5 kg
which is moving at a velocity of 1 m s1 in the same direction. If the 0.5 kg trolley moves at a
velocity of 2.5 m s1 in the same direction after the collision, what is the velocity of the 3 kg trolley?
Solution

2 m s1 1 m s1 v=? 2.5 m s1

3 kg 0.5 kg 3 kg 0.5 kg


(a) Before collision (b) After collision
The collision is elastic.
Total momentum before collision = Total momentum after collision
m1 u1 + m2 u2 = m1 v1 + m2 v2
3 2 + 0.5 1 = 3 v + 0.5 2.5
6.5 = 3v + 1.25
3v = 6.5 1.25
5.25
v =
3
= 1.75ms1
The 3 kg trolley moves at a velocity of 1.75ms1 in its original direction.

Forces and Motion 72


SPM
Momentum and Explosions 04/P2

1 Rifle
2
The explosion creates a backward momentum on
the rifle. This causes the rifle to recoil backwards.
Total momentum of the rifle
and the bullet is zero as they ard
kw tum
are stationary. bac men v1
mo riflev 2 forward momentum
on on bullet
m1
m2

1 F
When the rifle is fired, the explosion of
O
the gunpowder forces the bullet out of

2
the barrel. A momentum in the forward R

CHAPTER
direction is created. M

4
(a) Before explosion (b) After explosion
Figure 2.36
F4/2/38

2 Air escapes from a deflating balloon


upward
momentum
m2
balloon v2 2
Total momentum The balloon shoots upwards,
stationary of the balloon is moving with an upward momentum.
(u = 0) zero as it is
stationary. 1
v1 Air has mass and moves with a velocity.
This creates a momentum in the
m1 downward direction.
downward
momentum

(a) Before explosion


(a) Before explosion (b) After explosion
(b) After explosion
Figure 2.37

3 An explosion is a closed system which does not involve any external forcethat is, the total
momentum is conserved in an explosion.
Total momentum before explosion = Total momentum after explosion
0 = m1v1 + m2v2
Rearranging the formula:
m1v1 = m2v2
where v1 and v2 are of opposite directions.
If we ignore the direction,
m1v1 = m2v2
as the two momenta have the same magnitude.

Momentum to the left = Momentum to the right


Bear in mind that the two velocities
Momentum upwards = Momentum downwards are in opposite directions.
Momentum forwards = Momentum backwards

73 Forces and Motion


Activ To verify the principle of conservation of momentum in an
ity 2.4 explosion
Apparatus/Materials
4 trolleys, 2 wooden blocks, a hammer, and a metre rule.
Notes
The positions of the wooden blocks are adjusted so that each trolley collides with the corresponding
wooden blocks at the same time, t. From the equation d = vt, the magnitude of the velocity v is
directly proportional to the distance d, i.e., v d (if t is constant). Thus, the distance d travelled by the
trolley represents the velocity of the trolley.
Arrangement of apparatus Procedure
F
O (a) Before explosion 1 The apparatus is arranged as shown in Figure
2

R 2.38 (a).
release pin
CHAPTER

M wooden
trolley B trolley A block 2 Two trolleys A and B of equal mass are placed in
4 contact with each other on a smooth surface. The
mB mA spring-loaded piston in trolley B is compressed.
3 The release pin on trolley B is given a light tap to
dB dA release the spring-loaded piston which then
pushes the trolleys apart. The trolleys collide with
(b) After explosion F the wooden blocks.
O
spring- 4 The experiment is repeated and the positions
2

vB loaded R v
piston A of the wooden blocks are adjusted so that both
CHAPTER

M
trolleys collide with them at the same time.
4 5 The distances dA and dB are measured and
recorded.
Figure 2.38 6 The experiment is repeated using
(a) 1 trolley with 2 stacked trolleys,
(b) 3 stacked trolleys with 1 trolley.

Tabulation of data
The results of the experiment are recorded in Table 2.7.

Table 2.7
Before explosion After explosion
Initial total Mass of Mass of Velocity of Velocity of Final total
momentum trolley A, trolley B, trolley A, trolley B, momentum,
mA mB dA (dB) mA dA + mB (dB)
0 1 1 0
0 1 2 0
0 3 1 0
Discussion
1 Total momentum before explosion = 0 (because both trolleys are stationary)
Total momentum after explosion = mAdA + mB (dB) as d represents v,
Activity 2.4

2 Table 2.7 shows that mA dA + mB (dB) = 0 and v is a vector.


Total momentum after explosion = Total momentum before explosion
Conclusion
Momentum is conserved in an explosion.

Forces and Motion 74


19
Jane and John go ice skating. With their skates on,
Jane and John push against each other on level ice.
Jane, of mass 50 kg, moves away at a velocity of
3 m s1 to the right. What is Johns velocity if he is
75 kg?
Solution
This is a closed system since the external force, i.e.,
friction, is negligible.
Let the velocity of John be v.
F
Total momentum = Total momentum
Figure 2.39 O
after explosion before explosion

2
R
m1 v1 + m2 v2 = 0

CHAPTER
M
50 3 + 75v = 0
150 4
v = _____ The minus sign indicates that
75 John moves to the left,
= 2 m s1 opposite to Janes motion.

Alternative Method
omentum to the right = Momentum to the left
M
m1 v1 = m2 v2
Just ignore the sign as the two
50 3 = 75 v skaters are moving in opposite
v = 2 m s1 directions.

John moves to the left at a velocity of 2m s1.

20
Figure 2.40 shows trolley A, with a weight attached, placed in contact with trolley B on a smooth surface.

weight attached to trolley A


release pin

A B

d1 = 1 d2 d2
3

Figure 2.40

When the release pin of trolley B is tapped lightly, the Solution


spring-loaded piston pushes the two trolleys to move Applying m1 d1 = m2 d2:
in opposite directions. The two trolleys touch the 1 m1 is the total
wooden blocks simultaneously. Trolley A moves one- m1 d2 = m d2 mass of trolley A
3 and the weight.
third the distance moved by trolley B. Given that the
two trolleys are of equal mass, m kg, find the mass of m1 = 3m
the weight, in terms of m, that is attached to trolley A. The mass of the weight, m
= 3m m
= 2m

75 Forces and Motion


21
Harfeez alights a boat at a v m s1
velocity of v m s1 and lands
on the dock. v' = ?
The boat bounces backwards
with a speed of v.
If the masses of Harfeez and
the boat are m and 3m
respectively, find the speed
of the boat (ignoring friction Figure 2.41
F due to water).
O
2

R Solution
CHAPTER

M
m1 v1 + m2 v2 = 0
4 (m v) + (3m v) = 0
3mv = mv
1
v = v
3
1 1
The speed of the boat = v m s F
3
O
2

R
CHAPTER

4
Application of the Conservation of Momentum

1 Rocket

1 A rocket carries liquid 5

hydrogen and liquid


oxygen. 5 In accordance with the
principle of conservation of
momentum, the rocket gains
a forward momentum and
2 The mixture of hydrogen 1 moves forwards at high
fuel and oxygen burns liquid velocity.
vigorously in the hydrogen
(as fuel)
combustion chamber.
1
liquid
oxygen

3 The gases formed


expand rapidly and are
forced to discharge 2
through the exhaust combustion 4 A backward momentum
chamber
nozzle at a high is created.
velocity.
3
hot exhaust
gases at
high velocity
4

Figure 2.42
F4/2/41
Forces and Motion 76
2 Jet engine
1 2 3 4
Air from the atmosphere is drawn into In the combustion chamber, The hot gases formed expand rapidly
the engine and compressed by a kerosene fuel burns vigorously and are forced out of the nozzle at
compressor before it is forced into the with the compressed air. high speed through the turbine which
combustion chamber at high pressure. rotates the compressor.

3 combustion 4 turbine blade


2 compressor
chamber

5 F
1
jet of
O

2
air exhaust R

CHAPTER
intake gases M

nozzle

6 moves forwards fuel injector

6
In accordance with the principle of the conservation of 5
momentum, a forward momentum for the engine is produced. The ejected high-speed exhaust gases create
The plane thus flies forwards. a backward momentum.

Figure 2.43

Demonstrations to Show the Principle of Propulsion in Jet fuel


Engines
injector and Rockets
6
The following demonstrates the principle in rockets and jet engines. moves
forward

Jet engine Rocket


2 5 1 5 upward
thread
jet of cylindrical 1
1
3 exhaust weight plastic air
front gases air bottle intake

water
3 plastic tube
bicycle rubber stopper 2
pump
nozzle
elastic back tripod stand
band downward
4 turbine blade 3 combustion 4 2 compressor
chamber
1 A weight is placed on the trolley. 1 One-third of a plastic bottle is filled of water.
F4/2/43b
2 As the thread is cut, the weight is
F4/2/43a 2 The bottle is then plugged
F4/2/42tightly with a rubber stopper with a
thrown to the back with a plastic tube.
momentum. 3 Air is pumped into the bottle.
3 This in turn creates a forward 4 When the pressure of the air in the bottle is sufficient, it will
momentum which causes the push the stopper out together with the water. A downward
trolley to move forwards. momentum is produced.
5 This in turn creates an upward momentum which moves the
plastic bottle upwards.

77 Forces and Motion


The principle of the conservation of momentum occurs in nature the squid uses it to propel itself in the water.
The squid moves forward by discharging a jet of water from its body. An equal and opposite momentum created thus
propels the squid in the opposite direction.

F The shower of burning fragments from an exploding fireworks launched into the sky is governed by the principle of the
O F4/2/44athat the total momentum is conserved.
conservation of momentum. The symmetrical pattern indicates
2

R
CHAPTER

F
The large volume of water that rushes out from a water hose with a very high speed has a large momentum. In
O
accordance with the principle of theR conservation of momentum, an equal and opposite momentum is created causing
2

the fireman to fall backwards. Thus,M


several firemen are needed to hold the water hose.
CHAPTER

4 SPM
Clone
11

The diagram shows three identical theme park coins P Q R


on a straight line, with Q and R touching each other.
A Moves Stationary Stationary
P is given a velocity and collides Q.
B Stationary Stationary Moves
Q R C Moves Moves Stationary
token
P token
D Moves Stationary Moves
token

Comments
The momentum of P is totally transferred to R
What will be observed?
through Q.
Therefore, P stops and R moves with the velocity of
P before collision while Q remains stationary.
Answer B

Forces and Motion 78


2.4

1 A pigeon of mass 120 g is flying at a velocity of 5 A butterfly rests on a leaf floating on the surface of a
2ms1. What is its momentum? pond.

5 cm s1
2 A bull of mass 250 kg is moving at a momentum of
750 kg m s1. Find its velocity.

3 During training, Othman fires a pistol of 1.5 kg mass. 3 cm s1

The butterfly then starts moving to the tip of the leaf


at a speed of 5 cm s1 relative to the water. The leaf, F
stationary in accordance with the principle of the conservation O

2
of momentum, moves at 3 cm s1 relative to the R

CHAPTER
water in the opposite direction. If the mass of the leaf M
(a) Before is 8 g, determine the mass of the butterfly.
(a) Beforeshooting
shooting 4
6 Boat A and boat B are moving at a speed of 2 m s1
and 1 m s1 respectively before the two collide head
on. The masses of boats A and B (including the
passengers) are 150 kg and 250 kg respectively.
30 g

1.5 kg

(b) After shooting


(b) After shooting

A bullet with a mass of 30 g is released at a (a) Before collision


velocity of 300 m s1. What is the recoil velocity of
the pistol? 0.5 m s1 v=?

4 Hizam and his son Jamal are at an ice rink.

A B

(b) After collision


If boat A bounces back with a velocity of 0.5 m s1,
what is the velocity of boat B?
7 Sau Fei and Siew Ling, each with a mass of 60 kg and
49.5kg respectively, are standing at rest on an ice
rink. Sau Fei throws a ball of mass 0.5 kg towards Siew
Ling.

Jamal with a mass of 20 kg is moving at a velocity of


2ms1 while Hizam with a mass of 60 kg, is directly
behind Jamal and moving at 6 m s1. Hizam decides
to pick Jamal up and continues moving without
stopping. Determine the final velocity of Hizam and What is the recoil velocity of Sau Fei if the velocity of
Jamal. the ball is 8 m s1? What is the velocity of Siew Ling
after she receives the ball?

79 Forces and Motion


2.5 Understanding the Effects of a Force

2.5 Understanding the Effects of a Force

What is Force?

F
F A force is a push
or a pull.
Pressing a switch

F
Lifting objects
F
F F
O
2

R F
CHAPTER

M
COKE
4 Pulling off the ring of a soft drink tin Stretching a chest expander Kicking a football

1 When you push or pull on an object, you need to know


(a) the strength or magnitude of your force, and
F4/2/45
(b) the direction in which you are pushing or pulling.
F
2 Therefore, force is a vector quantity since it has both magnitude and
O

10
5
2
0

9
3

6
direction.
2

spring balance
R
3 A spring balance which isM used to measure the weight (or the
CHAPTER

gravitational pull on the object) can be used to measure the magnitude


of a force. For example, the4 force pulling a wooden block can be
measured as shown in Figure 2.44. Figure 2.44

The Effects of a Force

The shape of a tube of toothpaste A spring lengthens or compresses A plastic ruler can be bent when
changes when you press on it. when you stretch or compress it. a force is exerted on it.


1 A force can change the shape of an object (deformation of an object). F4/2/46c

F4/2/46a F4/2/46b

Effects of force

2 A force can change the original state of motion (either at rest or in motion) of an object.
(a) To move a stationary object (b) To stop a moving object (c)To change the direction of
motion of an object


A pushing force is required to F4/2/46e
Friction will stop the bicycle A tennis player hits a fast-moving
F4/2/46f
move a F4/2/46d
stalled car. when the boy stops pedalling. tennis ball to return it to his opponent.

Forces and Motion 80


The effects of force are applicable in the games below.
(a) Football
1
3 Ben kicks the football to Sani from a corner
Hazuri manages to catch the ball. during a football match. He needs to apply
He needs to apply a force to stop the ball. a force to make the stationary ball move.
Also, as his boot is in contact with the ball,
the force applied deforms the ball.

Hazuri
Ben F
O

2
R

CHAPTER
M
Sani
4

2
Sani's header produces a force on the ball to change its direction.

F4/2/47a
(b) Baseball

3 2 1
If the ball is not hit, it is The force from the bat deforms the ball A pitcher applies a force when
stopped by the catcher. and reverses the direction of the ball. he throws the baseball.
F4/2/47b

Relationship between Acceleration, Mass and Force

1 Consider two identical speedboats A and B at thrust is applied, speedboat P accelerates


rest on the sea. Different engine thrusts are slower than Q.
applied to accelerate the boats. However, 3 Thus, the three quantities: acceleration, mass
speedboat A which applies more engine thrust and force, are related.
increases the velocity at higher rate. 4 The relationship between the three quantities
2 Consider two identical speedboats, P and Q can be determined in the following experiments.
with P heavily loaded. When the same engine

81 Forces and Motion


2.2

Relationship between acceleration and force applied on a constant mass


Situation elastic cord attached to the trolley and stretched to a

fixed length represents one unit of force acting on the
Figure 2.45(a) shows car A and car B of the same
trolley.
mass at the same starting line. Car B is a sports car.
The engine capacity of sports car B is much bigger Apparatus/Materials
than car A. (A car with a bigger engine capacity can
provide greater engine thrust.) Trolley, 3 identical elastic cords, runway, ticker-
Figure 2.45(b) shows that sports car B has built up a timer, carbonised ticker tape, cellophane tape, 12V
higher velocity than car A after 3 seconds. a.c. power supply and a wooden block.
F
O Arrangement of apparatus
2

R B (greater engine capacity)


B
CHAPTER

M
ticker-timer stretched elastic cord (1 unitgreater engine
of force)
ticker tape thrust (force)
4 A
A friction-compensated
a.c. power runway
supply
trolley

wooden block
(a) t = 0 F
(a)
O F4/2/48
2

B
R greater
CHAPTER

M engine two stretched elastic cords (2 units of force)


thrust [same extension as (a)]
A (force)
4

(b) t = 3 s
Figure 2.45
(b)
Can you make an inference about this situation?
Figure 2.46
Inference
The acceleration of an object depends on the force Procedure
which acts on it. 1 A friction-compensated inclined runway is
Hypothesis prepared.
2 The apparatus is then set up as shown in Figure
When the mass of an object is constant, the greater 2.46(a).
the force applied, the greater its acceleration. 3 The ticker-timer is switched on and the trolley is
Aim pulled down the runway by an elastic cord

attached to the hind post of the trolley.
To investigate the relationship between acceleration
4 The elastic cord is stretched until the other end
and the force applied on a constant mass.
is level with the front end of the trolley. The
Variables length is maintained as the trolley runs down the

(a) Manipulated : force applied, F runway.
(b) Responding : acceleration of trolley, a 5 The ticker tape obtained is cut into strips of
Experiment 2.2

(c) Fixed : mass of trolley, m, and the 10-tick. A tape chart is constructed and the
stretching force of an elastic cord acceleration, a, is determined.
6 The experiment is repeated with 2, and 3 elastic
Notes cords to double and triple the pulling force to the
The force in this experiment is the stretching force in same constant extension as when one elastic cord
an elastic cord used to pull the trolley. A length of is stretched.

Forces and Motion 82


Results
Ticker tape chart:
Velocity (cm per tentick) Velocity (cm per tentick) Velocity (cm per tentick)

the gradient
is tripled
the gradient
is doubled

(a) One elastic cord (b) Two elastic cords (c) Three elastic cords
(a) (1 unit of force) (b) (2 units of force) (c) (3 units of force)
F
Figure 2.47 O

2
R
1 The gradient of the line passing through the dot at the top of each successive strip increases with the number

CHAPTER
M
of cords.
2 This indicates an increase in acceleration when the force is increased. 4
Tabulation of data Graph
Table 2.8 1 The graph of a against F is a straight line passing
through the origin as shown in Figure 2.48.
Units of force Acceleration, This shows that: a F (if m is constant).
applied, F a (m s2) acceleration, a
1 a1
2 a2
3 a3
force, F
O
Figure 2.48
Conclusion
The acceleration of an object is directly proportional to the force applied if the mass is constant.
The hypothesis is valid.

2.3 acceleratio

Relationship between acceleration and the mass of an object under a constant force
Situation

Figure 2.49(a) shows two similar lorries, A and B in front of a traffic light. When the light turns green, both drivers
step on the accelerator simultaneously with the same pressure to provide the same engine thrust, F. O
Figure 2.49(b) shows that within 3 seconds, the empty lorry has built up a higher velocity than the heavy one.
Experiments 2.2 & 2.3

B
B

full-loaded F B
lorry
A
A
empty
lorry F A


(a)
t = 0 (b) t = 3 s
Figure 2.49

F4/2/49
83 Forces and Motion
Inference 5 The elastic cord is stretched until the other end

is level with the front end of the trolley. The
The acceleration of an object depends on its mass. length is maintained as the trolley runs down the
Hypothesis runway.
When the force applied on an object is constant, the 6 The ticker tape obtained is cut into strips of
greater the mass of the object, the smaller its 10-tick. A tape chart is constructed and the
acceleration. acceleration, a, is determined.
Aim 7 The experiment is repeated using 2 trolleys (with
a second trolley stacked on the first trolley) and
To investigate the relationship between acceleration 3 trolleys. The elastic cord is stretched to the
and the mass of an object under a constant force. same fixed length as in the first experiment.
Variables Results
F

(a) Manipulated : mass of trolley, m Ticker tape chart:
O
(b) Responding : acceleration of trolley, a
2

R Velocity (cm per tentick)


(c) Fixed : force applied by an elastic cord, F
CHAPTER

M
Notes
4
(a) The mass in this experiment is represented by the
number of identical trolleys used.
(b) The constant force is applied by stretching the
elastic cord with the same extension for each (a) One trolley (1 unit of mass)
repetition of the experiment. F Velocity (cm per tentick)
O
Apparatus/Materials
2

R
CHAPTER

Ticker-timer, 12 V a.c. power Msupply, 3 trolleys, the gradient


is halved
elastic cord, runway, wooden block, ticker tape and
4
cellophane tape.
Arrangement of apparatus
(b) Two trolleys (2 units of mass)
ticker-timer stretched elastic cord
ticker tape Velocity (cm per tentick)

friction-compensated runway
a.c. power
supply the gradient is
trolley one third of (a)
wooden block

(a)
stretched elastic cord
(c) Three trolleys (3 units of mass)
F [same extension as in (a)]
Figure 2.51

2 units of mass
The gradients of the lines joining the uppermost dots
for successive strips decrease as the number of
trolleys used increases. This indicates that
acceleration decreases as mass increases.
(b)
Tabulation of data
Figure 2.50
Table 2.9
Procedure
Mass of Inverse
1 A friction-compensated inclined runway is Acceleration,
trolley, m of mass,
prepared. 1 a
Experiment 2.3

(Number of
2 The apparatus is then set up as shown in Figure 2.50. m (m s2)
trolleys)
3 A ticker tape is attached to the trolley and passed
through the ticker-timer. 1 1.00 a1
4 The ticker-timer is switched on and the trolley is 2 0.50 a2
pulled down the inclined runway by an elastic 3 0.33 a3
cord attached to the hind post of the trolley.

Forces and Motion 84


Graph 1
The graph of a against shows a straight line
m
acceleration, a passing through the origin as shown in Figure 2.52.
1
This shows that: a (if F is constant).
m

Conclusion
The acceleration of an object is inversely
O
proportional to its mass when the force acting
Figure 2.52 on it is constant.
The hypothesis is valid.
F
O

2
R

CHAPTER
M

Newton's Second Law of Motion 22

1 From Experiment 2.2: a F A force of 10 N acts on an object of mass 5 kg on a


smooth floor. Find its acceleration.
1
From Experiment 2.3: a a=?
m
10 N
The two results are combined.
F
a
m
k is a constant. Figure 2.54
orF ma
F = kma Solution
2 The unit of force is Newton, N. F = 10 N, m = 5 kg
3 In order to make the formula as simple as possible, F = ma
we make k = 1 by defining a force of 1N as: 10 = 5a
a = 2 m s2
1 N is the force which gives a mass of 1 kg
an acceleration of 1 m s2.

1 kg a = 1 m s-2
1N Fnet = ma is the mathematical expression of Newton's
second law of motion.
This law states:
Figure 2.53 The net force on an object is proportional to the rate of
change of momentum.
Change in momentum
F = kma Force
(net)
Time
1 N = k 1 kg 1 m s2 mv mu The net force on an
Experiment 2.3

k = 1 Fnet object causes the


t object with a mass
F = ma (v u) of m to change its
Fnet m velocity from u to v.
t

Newton's second Fnet ma vu


a=
law of motion Fnet = kma t

85 Forces and Motion


23
152
A car of mass 1200 kg travelling at 15 m s1 comes a = 2 30
to rest over a distance of 30 m. Find Retardation is
deceleration.
(a) the average retardation, and a = 3.75 m s2
(b) the average braking force. Retardation = 3.75 m s2
Solution (b) F = ma Negative sign indicates
= 1200 (3.75) retardation force or
u = 15 m s1, v = 0 m s1, s = 30 m opposing force.
= 4500 N
(a) Using the formula v2 = u2 + 2as:
0 = 152 + 2a(30) Average braking force = 4500 N

F
O
2

SPM SPM SPM SPM


R Balanced Forces and Unbalanced Forces 03/P1 05/P1 07/P2(C) 08/P1
CHAPTER

M
1 In general, there may be several forces acting on the object (whether a single force or several
4
on the mass, whether parallel or anti-parallel, forces are acting on it).
or in different directions.
2 Thus, the force, F, must be replaced with the Balanced Forces
net or resultant force when there are several
forces acting on the mass. 1 When the forces acting on an object are
F
O
balanced, they cancel each other out (that is,
2

R to use F = ma
Some prefer
net force = 0).
Fnet = ma 2 The object then behaves as if there is no force
CHAPTER

where FMis the sum of the


force vectors. acting on it.
4
3 Since Fnet = 0, the acceleration of the object,
where a is in the direction of the net or a = 0. Thus, the object remains at rest or moves
resultant force. at constant velocity when there is no net force
3 However, for simplicity, F = ma is always used, acting on it. This is Newtons first law of motion.
bearing in mind that F is the net force acting 4 Examples of balanced forces:

(a) Balanced forces on a stationary gymnast (b) Balanced forces on a car moving at a constant velocity
R

air resistance, G
thrust, T

T = G + Fr
friction, Fr
weight, W W=R
W
W=R

reaction force, R
from beam There are 3 horizontal forces acting on a car moving
at a constant velocity. The forward thrust, T,
provided by the car engine is balanced by the
frictional force on the wheels and the air resistance.
The weight of the gymnast, W, is balanced by the
F4/2/143
reaction force, R, from the beam. T = G + Fr
The two forces are of equal magnitude but opposite
in direction. The weight of the car, W, is vertically balanced by
Without the beam (that is, no reaction force), the the reaction force, R, from the road.
gymnast will fall to the ground because of her
weight. W=R

Forces and Motion 86


Balanced forces (Fnet = 0, a = 0)

F1 F2
F net = 0
(as no force
acting on it)
F1 = F2 F4/2/53a
FromFnet = ma:
0 = ma
a = 0 (since mass, m cannot be zero)

Object in motion F
Object at rest (v 0, and the object is moving O
(v = 0 m s1)

2
at constant velocity) R

CHAPTER
M

For example, For example,

5N
2 cm s1
PHYSICS
200 N Ffriction

Ffriction
Linda pushes a book on a table with a force of 5 N.
F4/2/53c
Zamhari pushes a heavy cupboard with a force of
F4/2/53b The book moves with a uniform velocity of 2 cm s1.
200N, but the cupboard does not move. Find the frictional force acting on the book.
Find the frictional force acting on the cupboard. Solution
Solution Using Fnet = ma: because the book
Using Fnet = ma: because the But Fnet = 0 since a = 0 moves with a
cupboard does uniform velocity
But Fnet = 0 since a = 0 5 Ffriction = 0
not move
200 Ffriction = 0 Ffriction = 5 N
Ffriction = 200 N (The frictional force here is known as dynamic
(The frictional force here is known as static friction) friction)

Note: In both situations, the frictional forces are equal to the forces applied.

SPM
Effect of Balanced Forces and Unbalanced Forces on an Object 08/P2(A)

Balanced forces (Fnet = 0, a = 0)

lift from wings, L 1 Balanced forces on an aircraft allow it to


move at constant velocity at a constant
altitude.
drag or air resistance, G
2 The engine thrust is balanced by the drag due to
engine thrust, T
air resistance while the weight of the aircraft is
T = G
weight, W
W = L
balanced by a lift from the wings. The lift from
the wings is discussed in Chapter 3.

87 Forces and Motion


Unbalanced forces (Fnet 0, a 0)
lift from wings, L

engine thrust, T

drag or air
weight, W resistance, G

1 When the forces acting on an object do not cancel out each other, a net force known as unbalanced force is
acting on the object.
2 Unbalanced forces produce an acceleration to the mass on which the forces are acting.
F 3 However, the object will accelerate in the direction of the net force.
O 4 When an airplane is moving at a constant velocity, if the pilot increases the engine thrust, the forces acting
2

R
horizontally are no longer balanced. There is a net force forwards and the plane will accelerate in the
CHAPTER

M
forward direction.
4

24
5000 N
Figure 2.55 shows Solution
upward
a small rocket of F
force from Since the upward force is greater than the downward
mass 300kg at the O
engine
force by 2000 N, the rocket accelerates upwards.
2

R 2000 N
point of take-off. Neglecting the mass of the exhaust gases, the upward
CHAPTER

M resultant
Find its initial mass
upward launching acceleration is given by:
300 kg
acceleration. 4 force Fnet = 5000 N 3000 N
weight = 2000 N
Fnet = ma
3000 N F 2000
a = mnet =
300
Figure 2.55 = 6.7 m s2
F4/2/55b

25
John pushes a 12 kg carton with a force of 50 N. When a floor is smooth, frictional force is nil.

2 m s2 F E velocity increasing
IL (a  0)
AG
FR
F1 = 50 N
Ffriction = 0 smooth surface
Fnet = 50 Ffriction

When an object on a rough floor moves at uniform


frictional force, Ffriction
velocity with a horizontal force acting on it, friction
is equal in magnitude to the applied force (but
Figure 2.56 acts in the opposite direction).
F4/2/55a
If the carton moves with an acceleration of 2 m s2,
what is the frictional force acting on the carton? constant velocity
F (a = 0)
E
Solution GIL
FRA
Fnet = ma
Ffriction rough surface
50 Ffriction = 12 2
F = Ffriction
F=F
Ffriction = 50 24 = 26 N friction

Forces and Motion 88


26 5 SPM
Clone
05

A shopper pushes a trolley with a force of 20N. The Two forces F1 and F2 act on a wooden block which is
trolley with a mass of 5 kg, moves at a uniform placed on a table. The friction between the table and
velocity of 1 m s1. He then increases his force to the block is 3 N.
accelerate the trolley. What force should he apply in
order to give the trolley an acceleration of 2ms2? F2 F1

Solution
table

20 N F = ?
Which pair of forces F1 and F2 will accelerate the block?
1m s1
a=2m s2 F
F1 (N) F2 (N) O

2
R
A 4 7

CHAPTER
(a) (b) M
F4/2/57 B 8 5
For (a): Ffriction = 20 N 4
C 6 4
For (b): Fnet = ma The trolley is moving D 9 5
F Ffriction = ma with uniform velocity,
F 20 = 5 2 i.e. the force applied
equals friction acting Comments
F = 30 N on the trolley.
The block will move with acceleration if the forces
acting on it are unbalanced.
If the difference between the forces F1 and F2 is
27 greater than 3 N, then there is an acceleration.
Mr Brown whose mass is 70 kg, performs as a human AnswerD
cannonball at a circus. He is propelled from a 1.6m
long cannon. He is in the barrel of the cannon for 1.2s.
6 SPM
Clone
07

The figure below shows a car of mass 1200 kg


moving at an acceleration of 2 m s2.

acceleration
Figure 2.57
Find the average net force exerted on him.
Solution If the frictional force acting on the car is 750 N, find
First, we need to find the acceleration of Mr Brown its engine thrust.
in the barrel. Then, apply F = ma to find the net force A 750 N C 2400 N
on him. B 1350 N D 3150 N
u = 0 m s1, s = 1.6m, t = 1.2 s Solution
1
Using s = ut + at 2: Use Fnet= ma since two forces are acting on the car.
2
Let T be the engine thrust of the car.
1 T Friction = ma
1.6 = 0 + a (1.2)2
2 T 750 = 1200 2
a = 1.6 2 = 2.22 m s2 T = 2400 + 750
1.44 = 3150 N
Fnet = ma = 70 22.2 = 1554 N AnswerD

89 Forces and Motion


Newtons Third Law of Motion
Newtons third law of motion states that:

If object A exerts a force, F on object B, then object B will exert an equal but opposite force, F on
object A. In other words:
To every action there is an equal but opposite reaction.

Everyday phenomena that are governed by Newtons (b) When a man paddles with a backward force, +F
third law of motion: Physics
Blog (action), the reaction force, F, pushes the boat
(a) When a boy presses on the wall with a force, F, the forwards.
wall presses on his hands with a normal reaction
F
force, F.
O
2

R
reactions, F
CHAPTER

M
action,+F
reactions,
4
F action, +F

F4/2/60b
(c) The principle used in rockets and jet engines can also
be explained by Newtons third Iaw of motion. The
F action that pushes the exhaust gases out through the
Note:The action and reaction forces of Newtons third
F4/2/60a O
law act on different objects whereas two nozzle results in a forward force (reaction force) that
2

R
balanced forces act on the same propels the rocket or jet engine forwards.
M object.
CHAPTER

2.5

1 What force is needed so that an object with a mass of 6 (a) When a 2 kg block is pushed with a force of
3kg has an acceleration of 2 m s2? 12N to the right, it accelerates with an
2 A wooden box of mass 2 kg is placed on a smooth acceleration of 3m s2. In which direction does
plane. If a force of F is applied to the box, it moves at friction act? Find its magnitude.
an acceleration of 3 m s2.
a = 3 m s2
Find the acceleration of another box with mass of
6kg if the same force is exerted on it.
12 N
3 Puan Zaitun pushes a trolley of mass m kg with a force 2 kg
of 30 N. The trolley moves with a uniform velocity of
1 m s1. When she doubles her force, the trolley
accelerates at 2 m s2. Find the mass of the trolley.
F4/2/62
4 Faizal rides a bicycle at a constant speed of 16ms1. (b) An additional force of 20 N to the left then acts
He stops pedalling and the bicycle stops completely on the block. In which direction does friction
after 6 s. Given the total mass of Faizal and his now act? Find the new acceleration of the
bicycle is 72 kg, find the average opposing force on block.
Faizal and his bicycle.
5 A race car of mass 1200 kg accelerates from rest to a
1 12 N 20 N
Clone velocity of 72 km h in 8 s.
SPM
2 kg
07
(a) Find the acceleration of the car.
(b) Find the net force acting on the car.

F4/2/63

Forces and Motion 90


2.6 Analysing Impulse and Impulsive Force

Impulse and Impulsive Force

When a player kicks a F4/2/64a


football, his boot is in
contact with the ball for a time t. F
O
During the time t, an average force F acts on the When a tennis racket hits an oncoming tennis ball,

2
F4/2/64b R
ball which makes the ball fly off with a the tennis racket delivers a large force that acts on

CHAPTER
M
momentum. the ball for a short time t. The ball bounces off in
Thus, the force F, acting for a period of time t, the opposite direction. Again, there is a change in 4
produces a change in momentum to the ball, momentum.
since the ball with a mass of m acquires velocity v The change in momentum is due to the force F
after the time t. acting on the object for a time t.

F = ma
F=m
( )vu
t
substitute a=
vu
t

SPM
Ft = mv mu mv mu 07/P1
F = t

Impulse = Change in momentum


Change in momentum
Impulsive force =
Time taken
unit = N s or unit = kg m s1
unit = N
Impulse is defined as the product of a
force, F and the time interval, t during which Impulsive force is defined as the rate of change
the force acts. of momentum during a collision or explosion.
Inpulse = Force Time
=Ft Impulsive force is the large force produced in a
Impulse is a vector quantity and has the same collision or explosion that happens in a short
direction as the force that causes the change in time of impact.
momentum.

Both formulae can be used to solve numerical problems.


Both are vector quantities.

91 Forces and Motion


Rebound and Impulse

1 An object might rebound from a wall, or stick to it without rebounding after striking it.
2 In which situation will the wall exert a greater impulse? Look at Example 28.

28
A tennis ball and a piece of mud with the same mass Solution
(0.060 kg) which are moving at 9 m s1 strike a wall. Take the direction to the right as positive.
The mud sticks to the wall while the ball rebounds at
6 m s1. Find the impulse on each object. For the mud: For the tennis ball:
Impulse Impulse
9 m s-1
F = Change in momentum = Change in momentum
O mud = mv mu = mv mu
2

R 9 m s-1
= 0 0.06 (9) = 0.06 6 0.06 (9)
CHAPTER

M
tennis ball = 0 + 0.54 = 0.36 + 0.54
4 6 m s-1 = 0.54 N s = 0.90 N s
Figure 2.58

3 Thus, a greater impulse is exerted


F on an object if it rebounds after a collision.
O
2

29 R
CHAPTER

M
Figure 2.59(a) shows a 2 kg wooden block initially at If the force acts for 3 s,
4
rest on a smooth surface. A force of 8 N is applied on (a) what is the impulse on the block?
the wooden block. (b) what is the velocity of the wooden block after 3 s?
u=0 v Solution
8N 8N
2 kg 2 kg
(a) Impulse = F t = 8 3 = 24 N s
t=0s t=3s (b) Impulse = Change in momentum
Ft = mv mu
(a) (b)
24 = 2v 0
Figure 2.59
v = 12 m s1

30
A player spikes an oncoming volleyball moving Solution
towards him at a speed of 5 m s1 to reverse its direction (a) Take the direction away from the hand as positive.
at a speed of 20 m s1. The mass of the ball is 0.36 kg. Impulse = Change in momentum
5 m s-1 20 m s-1
= mv mu
= 0.36 20 0.36 (5)
= 0.36 20 + 0.36 5
F
= 9Ns
mv mu
(b) F =
t
9
Figure 2.60 300 =
t
(a) Find the impulse on F4/2/164
the ball. 9
(b) If the average force acting on the ball is 300 N, t =
300
how long is the time of contact between the hand
and the ball? = 0.03 s

Forces and Motion 92


Effects of Time on Impulsive Force SPM
09/P1
SPM
09/P2/(A)

1 From F = mv mu 1
t F
t
= Change in momentum t small, F large
Time of impact
t large, F small
If the change in momentum is constant, then:
From F 1
t
2 A student throws a raw egg at a high speed at a wall, and another egg
against a towel held by his friends. In which case will the egg break? F
O

2
towel
R

CHAPTER
egg
egg M

egg
egg 4
pile of towel

The egg is stopped by the wall in a very short time The egg thrown against a towel falls on a few towel
interval, resulting in a large force which causes it towels stacked below and does not break.
to shatter. The movement of the egg is stopped in a longer
time interval, resulting in a small force.

In both cases, the eggs are stopped. Thus, the change in momentum is the same. pile of towe

3 To understand the effect of time in a collision, look at the following example.

Save your knees!


Joginder (mass, m = 50 kg) jumps down from a wall. He lands on a cement ground at a velocity of 6 m s1.

Joginder bends his knees upon landing. Joginder didnt bend his knees upon landing.
The time taken to stop his motion is 1.0s. The time taken to stop his motion is 0.05 s.

u = 6 m s1
u = 6 m s-1
v = 0 m s1
v = 0 m s-1
Solution
Take the downward direction as positive. Solution
m = 50 kg, u = 6 m s1, v = 0 m s1, t = 1.0 s Take the downward direction as positive.
mv mu m = 50 kg, u = 6 m s1, v = 0 m s1, t = 0.05 s
F =
t The negative sign indicates the mv mu 50 0 50 6
50 0 50 6 F= =
= force acts as an opposing force t 0.05
1 that reduces the momentum
of the object. = 6000 N
= 300 N
The impulsive force acting on Joginders leg is 6000N!
This force is great enough to break Joginders legs.
The impulsive force that acts on Joginders leg is 300N.

This example clearly shows that an effective way to reduce the impulsive force is to lengthen the collision time.
u = 6 m s1

v = 0 m s1
93 Forces and Motion
Increasing the Impulsive Force by Reducing the Time of Impact

In all the sports shown below, the time of impact with


It should be noted that
the ball is very small and the impulsive force
impulsive forces always
produced is large.
exist in pairs. In the figure
on the right, the change in
momentum of the tennis
ball produces a large
impulsive force on the
racket which reacts to give
rise to an equal but
F2 F1
opposite impulsive force to
FF
OO A footballer kicking A golfer driving a the ball (this is in
2

RR a football golfball with a club accordance with Newtons


M third law of motion).
CHAPTER

M
Both the racket and the
44 tennis ball are deformed
temporarily due to the
large force being exerted on
A batter hitting a A tennis player hitting a each other.
baseball tennis ball

F
O
2

R
A massive hammer head
CHAPTER

M
moving at a fast speed is
brought 4to rest upon hitting
the nail. The large change in
momentum within a short
time interval produces a
large impulsive force which
drives the nail into the wood.

pile driver

In
pestle
pile construction,
tion
the pile driver
is raised to a
certain height
before it is
An expert in karate can released. The
mortar
split a thick wooden slab momentum
with his bare hand which acquired by Food such as chillies and
is moving at a very fast the massive onions can be pounded using a
speed. The momentary pile driver is greatest just before it mortar and pestle (both made
contact between the hits the pile. The subsequent from stone). The pestle is
fast-moving hand and change in momentum that brought downwards at a fast
the wooden slab occurs in a short time interval speed and stopped by the
produces a large results in a large impulsive mortar in a very short time.
impulsive force which force which drives the pile into This produces a large
splits the wooden slab. the ground. impulsive force which crushes
the food.

Forces and Motion 94


Reducing the Impulsive Force by Increasing the
Time of Impact In sports, the effects of
impulsive forces are reduced
to prevent injuries to
athletes. Thick mattresses
Polystyrene and cardboard egg with soft surfaces are used
containers are stiff but in events such as the high
compressible. They will jump and pole-vaulting so
absorb and reduce impulsive that the time of impact
force by lengthening the on landing is extended,
time of impact. thus reducing the
resultant impulsive force.
polystyrene F F
O O

2
R R
The use of padding in certain sports equipment M

CHAPTER
M
like baseball gloves, goalkeeping mitts, pole-
cardboard egg carton
vaulting pits, boxing gloves, and gymnastic mats 4 4
is to prevent injuries to players by reducing the
impulsive force.

The effects of (a) Baseball glove (b) Boxing glove


time on the
magnitude of
the impulsive In baseball, a player must catch
the ball in the direction of the
force motion
m
mo of the ball. If the ball is
caught
c
cau by stopping it in its path,
the
t impulsive force acting on the
hand
h
ha will be considerable.
Moving
M
Mo his hand backwards
when
wh catching the ball
prolongs
pr the time for the
momentum change to occur so
mo
the impulsive force is reduced.

Playgrounds are covered with a coarse


fabric material which prolongs thee time When a boxer sees that
of impact when the children fall,l, thus his opponents fist is
reducing the impulsive force. going to hit his head, he
will move his head
backwards or duck.
This will increase the
stopping time, hence
reducing the average
force on his head since
the momentum change
coarse fabric
material will be longer.

95 Forces and Motion


31
A tennis player hits an oncoming 0.060 kg tennis ball Students are always tempted to find the acceleration, a
with a velocity of 60 m s1. The ball bounces off in the and then use F = ma to find the impulsive force.
opposite direction at 90ms1. mv mu
By using F = , you save the step of finding a,
t
90 m s-1
which is not necessary. Try using the latter formula if the
125 N question mentions the impulsive force, and
acceleration is not stated.
60 m s-1

F
7 SPM
Clone
07
O
2

R Figure 2.61 The front and rear sections of a car are designed to
CHAPTER

M crumple easily. What is the reason?


Find the time of impact between the racket and the A To increase the impact time
4 ball if the impulsive force acting on the ball is 125 N. B To increase the impulse
Solution C To decrease the momentum
D To increase the frictional force
Take the direction to the right as positive.
F = 125 N, m = 0.060 kg, v = 90 m s1, u = 60 m s1 Comments
F
mv mu Change in momentum
F = O Impulsive force =
2

t R Time
0.06 90 0.06 (60)
CHAPTER

M
When the time of impact is prolonged, the impulsive
125 =
t force is reduced.
4
5.4 + 3.6
t = AnswerA
125
= 7.2 102 s

8 SPM
Clone

32 09

Tiger Woods hits a golf ball of mass 0.045 kg at a An athlete bends his legs upon landing in a long
velocity of 30 m s1. jump event as shown in the Figure 2.63.

m = 0.045 kg

30 m s1

Figure 2.62

If the time of impact is 0.005 s, what is the average


impulsive force applied on the ball by the club?
Solution Figure 2.63
m = 0.045 kg, u = 0, v = 30 m s1, t = 5 103 s
The purpose of bending his legs is to reduce the
Impulsive force, F = mv mu A impulse on his feet
t
B impulsive force on his legs
(0.045 30) (0.045 0)
= C velocity before landing
5 103 D time of impact between his legs and the sand.
= 270 N

Forces and Motion 96


Comments landing to lengthen the time of impact; reducing the
The impulsive force is inversely proportional to the impulsive force.
time of impact. The athlete bends his legs upon
AnswerB

2.6

1 A force F acts on a 6 kg object at rest on a smooth 6 A baseball of mass 0.14 kg moving at 40 m s1 is


surface. If the velocity of the object increases to struck by a bat and rebounds at 60 m s1.
2ms1 in 6 s, what is the value of F ? F
2 An object of mass 2 kg is acted on by a force which O

2
R
causes the velocity of the object to increase from 40 m s-1 60 m s-1

CHAPTER
M
1ms1 to 9 m s1. What is the impulse on the object?
3 The figure shows a helicopter dropping a box of mass 4
60kg which touches a sandy ground at a velocity of
12ms1. The box takes 2 seconds to stop after it (a) (b)
touches the ground.
Given that the time of contact is 5 102s, find the
force exerted on the ball.

7 In a crash test, a car of mass 1500 kg crashes into a


wall at 15 m s1. It rebounds at 2 m s1. 40 m s-1

15.0 m s1

(a) Before collision

2 m s-1

What is the magnitude of the impulsive force on the


box? F4/2/77

4 Beckham kicks a ball with a force of 1500 N. The (b) After collision
time of contact of his boot with the ball is 0.008 s. If the collision time is 0.18s, find the force exerted
What is the impulse delivered to the ball? If the mass on the car.
of the ball is 0.5 kg, what is the velocity of the ball?
5 A 0.045 kg golf ball strikes a wall at a speed of 8 A pole-vaulter (mass, m = 50 kg) falls onto a foam
30ms1 and rebounds at a speed of 20 m s1. mattress which exerts a force of 250 N on him over a2 m s-1
time interval of 2.0 s.
30 m s-1

u=?
foam 250 N
20 m s-1 mattress

What is the impulse on the ball? If the force on the


ball is 500 N, find the contact time of the ball with
the wall. fo
Find his velocity just before landing on the mattress. ru

97 Forces and Motion


F

4
R
O

M
CHAPTER 2

2.7 Being Aware of the Need for Safety Features in Vehicles

lmportance of Safety Features in Vehicles SPM


07/P1

Forces and Motion


1 It is important to increase the time interval
of collision to reduce the impulsive force in Safety seat belts Headrest
Prevents the passenger from being To push the driver's head so that it moves
an accident. This concept was explained in thrown forward or out of the car. Slows together with the rest of his body when a
Section 2.6. down the forward movement of the stationary car is knocked from behind and
2 Figure below explains the safety features in passenger when the car stops abruptly. pushed forward suddenly. This will prevent
a severe whiplash injury to the neck.
the design of vehicles.

M
FF

44
RR
OO

M
CHAPTER
CHAPTER 22
Shatter-proof windscreen
Prevents the windscreen
from shattering and reduce
injuries of passengers caused
by the shards of glass.
Tyres with tread
Drains away water in

98
Automatic airbag its grooves when the
Acts as a cushion car is on a slippery
for the head and road. The car has
body in an more contact on
accident and thus the road.
prevents injuries to
the driver and front
passenger. Strong steel struts
Prevents the collapse of the front and back doors
of the car into the passenger compartment.
Also gives good protection from a side-on collision.

Anti-lock braking system (ABS)


Bumper Prevents the wheels from locking when the
Absorbs the impact brakes are applied suddenly.
in minor accidents, Allows a car to stop quickly in slippery road
thus preventing conditions and prevents the car from skidding.
damage to the car.
Crumple zone
Padded dashboard Increases the time interval of impact so that the resultant impulsive
Increases the time interval of collision, force is reduced. Therefore, only little energy is transferred to the
thereby reducing the impulsive force inside of the car, and the passengers sustain minimal bodily injuries.
produced during impact. The car is less likely to rebound upon impact. This reduces the
momentum change or impulse.
2.8 Understanding Gravity
SPM SPM SPM A stroboscopic photograph is
Acceleration Due to Gravity 05/P1 08/P1 09/P1
a photograph that shows the
images of an object in motion.
1 Figure 2.64 shows a stroboscopic photograph of two steel spheres The images are taken at
(of different sizes) falling under gravity. The two spheres are dropped regular time intervals.
simultaneously from the same height.
2 The time intervals between two successive images are the same.
3 Two inferences can be made from the photograph.

Inference 2 F
Inference 1 s1 O

2
The two spheres are falling R

CHAPTER
M
The two spheres are falling with the same acceleration.
s2
with an acceleration. The two spheres are at the 4
The distance between two same level at all times.
successive images of each s3 Thus, a heavy object and a
sphere increases, showing light object fall with the
that the two spheres are same gravitational
falling with increasing acceleration.
s4
velocity, i.e., falling with an In other words, gravitational
acceleration. acceleration is independent
of mass.

Figure 2.64

Free-falling Object 3 Practically, a free fall can only take place in a


vacuum.
1 A free-falling object is an object falling under 4 The demonstration in Table 2.10 shows the
the force of gravity only. difference between a fall in air and a free fall in
2 A free-falling object does not encounter other a vacuum of a coin and a feather. Both the coin
forces like air resistance or friction that would and the feather are released simultaneously
oppose its motion. from the same height.
Table 2.10
In air In vacuum

to vacuum
pump
coin tap
feather


The air resistance to the feather is greater than the In a vacuum, both the coin and the feather would
air resistance to the coin. The feather falls with a fall with the same acceleration regardless of their
smaller acceleration and takes a longer time to reach shapes or masses.
the ground.

99 Forces and Motion


Acceleration of Gravity

1 Figure 2.65 shows that all objects are pulled towards the centre of the
Earth by the force of gravity.
2 The objects will fall with an acceleration of 9.8 m s2 due to the pull of
this gravitational force.
3 Since this acceleration is due to the force of gravity, it is called the
gravitational acceleration.
4 The value of gravitational acceleration is important in physics and is Figure 2.65Gravitational
F4/2/80 force
specially denoted by the symbol, g. is always acting
5 However, the value of g varies slightly according to location. towards the centre
6 The value of g depends on: of the Earth
F
O (a) Latitude
2

R Generally, the value of g increases with latitude.


CHAPTER

M
g = 9.832 m s 2
4
North Pole (90 N)
rp = distance from the North Pole to
rp < re the centre of the Earth
re = distance from the Equator to the
re F centre of the Earth
g = 9.780 m s 2
Equator (0) O
2

R
CHAPTER

M

4 Figure 2.66(a)
F4/2/81a
The elliptical shape of the Earth causes the gravitational acceleration to vary with latitude.
The value of g is smaller at the equator than at the poles because the surface of the Earth is further
from the centre of the Earth at the equator than at the poles.
(b) Height above sea level
The greater the height above the sea level is, the smaller is the value of g.
g1 < g2
g2
Mount Everest

New Delhi (same latitude as


Mount Everest but nearer to
the centre of the earth)

Figure 2.66(b)
F4/2/81b

1 Air resistance depends on the shape or cross-


sectional area of the falling object.
The figure shows that a paper, repeatedly folded and
sealed tightly with a cellophane tape, falls faster than
crumpled
a loosely crumpled piece of paper (together with paper
cellophane tape) of the same mass. folded
2 Air resistance also increases with the velocity of paper
Physics
the falling object. An objectBlogmoving at a higher
velocity encounters greater air resistance than a
similar object moving at a lower velocity.
F4/2/82

Forces and Motion 100


Activ To determine the value of gravitational acceleration, g
ity 2.5
Apparatus/Materials
Ticker-timer with carbonised tape, power supply, retort stand, slotted weights with holder, G-clamp, cellophane
tape and soft board.
Arrangement of apparatus
G-clamp

ticker-timer
ticker
tape
F
O

2
12 V a.c. R
power supply ticker tape

CHAPTER
M
stool folded and
stapled
weight here 4

G-clamp
soft board

Figure 2.67
Procedure 2 Calculation of the value of g:
Initial velocity, u
1 A ticker-timer is clamped to a retort stand with a
x cm x
G-clamp and placed on a tabletop as shown in = = cm s1
2 0.02 s 0.04
Figure 2.67.
2 One end of a carbonised ticker tape Final velocity, v
(approximately 1.5m in length) is attached to a y cm y
= = cm s1
weight holder with a total mass of 200 g. 2 0.02 s 0.04
3 The other end of the ticker tape is passed Time taken, t = (6 1) strips 0.04 = 0.20 s
through the ticker-timer. Gravitational acceleration, g is calculated by
4 A soft board is placed on the floor below the substituting the values of u, v and t into the
weight to stop its fall. vu
5 The ticker-timer is switched on and the weight formula g = .
t
is released so that it falls squarely onto the soft
board. Discussion
6 Six strips are cut off from the middle section of 1 If the activity is repeated with weights of mass
the ticker tape with each strip containing 2 dot- 250 g, 300 g, etc, the calculated values of g
spaces. would be the same. This shows that gravitational
7 A tape chart is constructed. From the chart, the acceleration, g is not affected by the mass of the
acceleration of gravity is calculated. object.
Results 2 It is found that the value of the gravitational
acceleration, g obtained from this activity is less
1 Ticker tape chart:
than the standard value of 9.8 m s2. This is
because the weight (object) is not actually free
falling. It is affected by the following:
y cm
(a) Air resistance during its fall.
(b) There is friction between the ticker tape and
Activity 2.5

x cm the ticker-timer.
Conclusion
t A free-falling object falls with gravitational
Figure 2.68 acceleration, g of 9.8m s2.

101 Forces and Motion


Weight and Gravitational Acceleration SPM
09/P1

1 The weight of an object is defined as the force


of gravity which is exerted on it by Earth.
2 From the formula: Since acceleration is proportional to the force acting,
a student may think that a sphere with a weight 10
F = force and times the weight of a smaller sphere would fall with
F = ma
a = acceleration an acceleration 10 times that of the lighter one.
Since weight, W, is the force of gravity acting
on an object of mass, m that makes it fall with It is true that the acceleration of an object is directly
an acceleration, g, therefore, using the proportional to the force acting on it. However, at the
corresponding terms, we get: same time, acceleration is also inversely proportional
F to mass. Therefore, the effect of greater force is
O F = m a cancelled by the larger mass of the heavy sphere.
2

R
So all objects in free fall have the same acceleration

CHAPTER

M
regardless of their masses.
W = m g Unit is the same
4
as force, N.
Weight of object
= Mass of object Acceleration due to gravity

FF
Gravitational Acceleration Oand
O Gravitational Field Strength
22

RR
1 A gravitational field is theMMregion in which an object experiences a
CHAPTER
CHAPTER

force due to gravitational attraction.


44
2 The gravitational field strength is defined as the ratio of the weight
to the mass of the object or weight per unit mass. 10 spring balance
20
measures the
Weight W gravitational
30

40

3 Gravitational field strength = = Unit = N kg1 50


attraction
Mass m
60

70 (weight) on
the object
Rearranging the formula,

W = m Gravitational field strength m

Compare this with the formula W = m g: mg

W=m Gravitational field strength Earth


W = m g

Gravitational field strength = g Figure 2.69

33
g=Gravitational
An object of 8 kg weighs 120 N on a planet. acceleration
Find the gravitational field strength on the planet. g=Gravitational field
strength
Solution
g = 10 m s2
W = mg
= 10 N kg1
120 = 8g
120 N W = mg holds true when g
g =
8 kg is given in ms2 as well as
N kg1.
= 15 N kg1

Forces and Motion 102


34
Calculate the weight of an astronaut with a mass of 100kg
(a) on Earth,
1
(b) on the Moon where the gravitational acceleration is of that on Earth,
6
(c) in outer space.
[Take g = 10 m s2]
Solution
Situation (a) Surface of the Earth (b) Surface of the Moon (c) Outer space
F
O

2
R

CHAPTER
M

4
Mass, m 100 kg 100 kg 100 kg
Weight, W
W=mg Gravitational acceleration, g In the outer space, there is
=100 10 1 no gravitational pull on the
= g
=1000 N 6 astronaut.
1 g = 0
= 10 W= mg
6
= 100 0
= 1.67 m s2
= 0 N (no weight)
W=mg
=100 1.67
=167 N

Mass and Weight


Notice that in Example 34, the weight, W varies but the mass, m of an object remains the same.
The differences between mass, m and weight, W:

Mass, m Weight, W

The mass of an object is the amount of The weight of an object is the force of
matter in the object. Definition gravity on the object.
The weight of an object varies with the
The mass of an object is constant
Changing of value magnitude of gravitational field strength, g
everywhere.
of the location.

A scalar quantity Physical quantity A vector quantity

A base quantity Type of quantity A derived quantity

kilogram (kg) newton (N)


SI unit

103 Forces and Motion


SPM SPM
03/P1 07/P1

Remember our equations of motion with constant Some recommend changing the acceleration, a in the
acceleration: equations to g, but this is not necessary.
Simply bear in mind that while solving problems involving
v = u + at
free fall, the acceleration, a in the equations has a value of:
1 a = 10 m s2 (for downward motion)
s = ut + at 2
2 Since the object is accelerating downwards.
v2 = u2 + 2as a = 10 m s2 (for upward motion)
Since the object is moving with deceleration.

F 36 SPM
04/P1
O
Since W = mg,
2

R A coconut takes 1.5 seconds to fall to the ground.


the weight of 25 000 km
CHAPTER

M
an object What is
4 depends on mass weight (a) its speed when it strikes the ground?
20 000 km
the 1 kg g 1 N
1
(b) the height of the coconut tree?
gravitational
mass weight Solution
field strength, 15 000 km 1 kg 2N
g which g2 up = 0 Coconut falls from rest.
decreases as F ap = 10 m s2
the distance of 10 000 km O
mass weight
2

the object 1 Rkg 10 N Coconut accelerating


CHAPTER

from the M downwards.


5 000 km g3
centre of the
4
Earth
increases.
Earth h
g1 < g2 < g3

35 t = 1.5 s
v=?
Wei Bo is a basketball player. His vertical leap is
( a) v = u + at
0.75m. What is his take-off speed?
= 0 + 10 1.5
Solution = 15 m s1
1
(b) s = ut + at2
2
1
h = 0 + 10 1.5 1.5 = 11.3 m
2

0.75 m 37
After winning a game, a pitcher throws a baseball
s = 0.75 m vertically up with a velocity of 30 m s1.
a = 10 m s2 Wei Bo is moving upwards. (a) What is the time taken for the ball to reach the
v = 0 maximum height?
v2 = u2 + 2as At the highest point, (b) What is the speed of the baseball when it returns
0 = u2 + 2 (10) 0.75 Wei Bo is at rest to his hands?
momentarily.
u2 = 15 (c) How long is the ball in the air before it comes
u = 3.87 m s1 back to his hands?

Forces and Motion 104


Solution v2 = 900
(a) Consider the motion B v = 30 m s1+ve
from A to B (motion Since the ball is moving downwards (direction of

motion is opposite to its initial direction),
up):
u = 30 m s1 v = 30 m s1 ve
a = 10 m s2 The speed of the ball is 30 m s1.
At the (c) When the ball returns to his hands, the
v = 0 highest A C
v = u + at point, displacement, s, is zero.
0 = 30 + (10)t v = 0 u = 30 m s1, a = 10 m s2, s = 0
t = 3 s 1
s = ut + at2
2
The ball takes 3 s 1
0 = 30t + (10)t2 when the ball
to reach the 2 is thrown. F
maximum height. O

2
5t2 30t = 0 R
(b)
When the ball returns to his hands, the

CHAPTER
t(t 6) = 0 M
displacement, s, is zero. t = 0 when the ball returns 4
Consider direction upwards as positive. or t = 6 to his hands.
u = 30 m s1, a = l0 m s2, s = 0
v2 = u2 + 2as The time taken for the ball to return to his
= 302 + 2(10)(0) hands is 6 s.

Problems Involving F = ma and W = mg


(a) From Example 37, by neglecting air resistance, a ball
thrown at 30 m s1 takes 3 seconds to move up. Lift
This is true as the deceleration is 10 m s2. The
velocity decreases by 10 m s1 for each second.
So after 3 seconds, the velocity is zero. reading on the scale shows
the normal reaction force
30 m s1 20 m s1 10 m s1 0 m s1 0
W = true weight = mg
1 s 1 s 1s W R = normal reaction force
exerted on the girl by the
(b) The velocity when the ball returns to its original platform of the scale
position is of the same magnitude as the initial R
velocity of the ball, but in the opposite direction.
weighing machine
(c) The time to descend to the original position is the
same as the time to ascend. Therefore, an
alternative method to find the total time in the air is Figure 2.70
F4/2/85
shown as follows. 1 When a girl stands on the platform of a weighing
Time to reach the highest point = 3 s scale, there are two forces acting on her:
The total time in the air = 2 3 = 6 s (a) the girls weight, W (= mg) acting down
at rest wards, and
t=3s 0 m s-1 0 m s-1 t=3s (b) the upward normal reaction force, R exerted
on her feet by the platform of the scale.
t=2s 10 m s-1 10 m s-1 t=4s 2 The reading of the scale gives the value of the
normal reaction force, R.
3 Table 2.11 shows a different situation in the lift.
t=1s 20 m s-1 20 m s-1 t=5s [Please take note that when the lift is
accelerating upwards or downwards, the
reading on the scale gives the apparent weight,
t=0s 30 m s-1 30 m s-1 t=6s
which is equal to the normal reaction force on
the feet of the girl.]

F4/2/84 105 Forces and Motion


Table 2.11
Situation Explanation As force vector

(a)Lift at rest or moves up or down at a constant Velocity is zero or constant R


velocity a=0

cable Applying Fnet = ma,


R
R mg = ma
reading shows the
normal reaction R mg = 0
force, R R = mg
0
acceleration, a = 0 mg
F
O
F net = 0 Reading on the weighing
The girl feels normal.
2

R scale gives the true weight.


F4/2/86b
CHAPTER

4 weighing machine
mg

F4/2/86a

FF
(b) OO of a m s2
Lift moves up at an acceleration Lift accelerates upwards R1
22

RR Resultant force, Fnet on the girl


CHAPTER

M
CHAPTER

M
R1 cable
is upwards
44 R1 > mg
acceleration
=a
Applying Fnet = ma,
0 R1 mg = ma
R1 = mg + ma

Reading on the weighing scale mg


is increased by ma.
weighing machine The girl feels heavier.
F4/2/87b
mg

F4/2/87a

(c) Lift moves down at an acceleration of a m s2 Lift accelerates downwards R2


Resultant force, Fnet on the girl
cable
is downwards
mg > R2

Applying Fnet = ma,


R2 0 mg R2 = ma mg
R2 = mg ma
The girl feels lighter.
acceleration
=a Reading on the weighing scale F4/2/88b
is decreased by ma.
weighing machine
mg

F4/2/88a

Forces and Motion 106


SPM
38 06/P1

A box of mass 1.6 kg is suspended from a spring Solution


balance hanging from the ceiling of a lift.
The reading on the spring balance
= The tension in the spring, T
10

(a) If the lift is stationary:


20

30

40

T=mg
50

60

70

=1.6 10
T =16 N
(b) When the lift is accelerating upwards: F
O
T1 mg = ma

2
R
T1 =1.6 10 + 1.6 2 Apparent weight (the

CHAPTER
W = mg M
=19.2 N reading on the scale)
Figure 2.71 increases when object
accelerates upwards. 4
F4/2/203
What is the reading on the spring balance if
(a) the lift is stationary? (c) When the lift is accelerating downwards:
(b) the lift moves upwards at an acceleration of mgT2 =ma
2ms2? T2 =1.6 10 1.6 3 Apparent weight (the
=11.2 N reading on the scale)
(c) the lift moves downwards at an acceleration of decreases when object
3ms2? accelerates downwards.
[Take g = 10 m s2]

Help! I am floating!
What is the apparent weight of a girl on a weighing scale Solution
in a lift if the cable of the lift suddenly breaks? The girl, the weighing scale and the lift will free-fall if the
cable suddenly breaks.

a = g
R = mg ma
= mg mg
0
= 0

The apparent weight is zero and the girl experiences


weightlessness. She feels as if she is floating.

Pulley System

1 A frictionless pulley serves to change the direction of a force.


2 The tension, T that results from pulling at the ends of the string or rope
has the same magnitude along its entire length.

(A) A force pulling a mass over a pulley


In this situation, the tension T, is equal to the pulling force F, even if
the rope is slanting.

107 Forces and Motion


 boy is pulling a bucket filled with sand. The mass of the bucket with the sand is
A
3 kg. Find the tension in the rope if the bucket is
i(i) stationary, or
(ii) moving up with a constant velocity of 2 m s1.
Solution
Whether the bucket is stationary or moving up with a constant velocity,
the acceleration, a = 0.
Applying Fnet = ma,
T mg = ma
T (3 10) = 0
T = 30 N
F
O  he boy increases his force to move the bucket upwards with an acceleration of
T
2

R
2 m s2. Find the applied force, F.
CHAPTER

M
Solution
4
Since the bucket is moving upwards, T > mg.
Applying Fnet = ma,
T mg = ma
T (3 10) = 3 2
T =
F 6 + 30
O
= 36 N
2

R
Applied force, F = 36 N
CHAPTER

(B) A pulley with two masses 4


1 The heavier mass will accelerate downwards while the lighter
one will accelerate upwards with the same magnitude.
2 The tension is not equal to the weight of either mass.

39
Two masses of 5 kg and 3 kg are connected to a rope Applying Fnet = ma,
which passes over a frictionless pulley. Mg T = Ma
50 T = 5a (i)

For the 3 kg mass, it moves upwards (T> weight mg).


Applying Fnet = ma,
T mg = ma
T 30 = 3a (ii)
(i) + (ii): 50 30 = 8a
8a = 20
a = 2.5 m s2
Acceleration, a = 2.5 m s2
Figure 2.72
Substitute the value of a into equation (ii).
Find the tension in the rope and the acceleration of T 30 = 3 2.5
the 3 kg mass when the 5 kg mass is released. T = 30 + 7.5
Solution = 37.5 N
When the 5 kg mass is released, it moves downwards
Tension in the rope, T = 37.5 N
at an acceleration of a (weight Mg > T).

Forces and Motion 108


9 SPM
Clone
07
For Example 39, if you need to find the acceleration
only, you can solve the problem by considering the two A stone falls from a height of 20m. What is the time
masses together. taken for the stone to reach the ground?
The weight of 50 N overcomes the weight of 30 N. A 1.4 s C 2.8 s
Applying Fnet = ma, B 2.0 s D 3.6 s
50 30 = (5 + 3)a Since both the
20 = 8a masses are Solution
a = 2.5 m s2 moving together. u = 0 m s1, a = 10 m s2, s = 20 m
1
Applying s = ut + at 2,
2
40 1
20 = 0 + (10)t 2
F
2 O

2
A 2 kg weight is connected by a rope to a 3 kg R
5t2 = 20

CHAPTER
wooden block. The rope passes over a smooth pulley M
t 2 = 4
as shown in Figure 2.73.
t = 4 4
= 2 s
AnswerB

10 SPM
Clone
09

Figure 2.73 The diagram shows a


coconut falling from a
The weight is then released. Find the tension in the tree.
rope if a friction of 5 N acts against the wooden block. Which of the following
Solution graphs correctly shows
the relationship between
the gravitational force, F
and the mass, m, of the
coconut?

A
F C F

For the 2 kg weight, motion is downwards. m m


Applying Fnet = ma,
20 T = 2a (i) B
F D F
F4/2/209
For the wooden block, motion is to the right.
Applying Fnet = ma,
T 5 = 3a (ii)
m m
(i) + (ii): 20 5 = 5a
5a = 15 Comments
a = 3 m s2
Substitute a into equation (ii). Gravitational force (weight), w = mg (i.e. F = mg).
T 5 = 3 3 Since g is constant, so F m.
T = 9 + 5 The corresponding graph is a straight line passing
= 14 N through the origin.
Tension in the rope = 14 N Answer C

109 Forces and Motion


2.8

1 An object weighs 60 N on the surface of the Earth If the distance between the water surface and his
where the gravitational field strength is 10 N kg1. hands is 9 m, what is
What is the weight of the object on a planet where (a) the time, and
the gravitational field strength is 18 N kg1? (b) the velocity
2 Mustafa steps off a 3.0 m high diving board and when the stone strikes the water surface?
drops to the water below. Find his velocity when his 5 A ball is thrown vertically upwards with a velocity of
leg touches the water. 20m s1. At what height will the ball be when its
3 Alvin tosses a coin up. The coin takes 1.2 s to return velocity reduces to half?
to his hand.
6 A student of mass 50 kg stands on a weighing scale
F in a lift. If the reading on the scale becomes 550 N,
O is the lift accelerating upwards or downwards?
2

R Find the magnitude of the acceleration of the lift.


CHAPTER

M
7 Find the tension in the rope and the acceleration of
4 the 2 kg weight in each of the following figures.
(a) (b)

Find F
(a) the initial velocity of the coin, O
2

(b) the maximum height of the coin. R


CHAPTER

M
4 A man releases a stone into a well.
4

8 A steel sphere is dropped from a height of 9 m on


SPM
the surface of the Moon. Given that the gravitational
Clone
03 acceleration on the Moon equals
1
of Earths
6
gravitational acceleration, find the time taken by the
sphere to reach the surface of the Moon.

2.9 Analysing Forces in Equilibrium


SPM SPM
Forces In Equilibrium 07/P1 08/P1

1 Forces in equilibrium or balanced forces have


been discussed in Section 2.5. air resistance, G
2 When forces are in equilibrium, the net
force, Fnet or resultant force, FR is zero. The
object will either be W=G

(a) at rest, or
(b) in motion with constant velocity.
3 Examples of two forces in equilibrium are constant
(a) a skydiver falling at a constant velocity, velocity
weight, W
(b) a book resting on a table.
(a)A skydiver falling at constant velocity

Forces and Motion 110


normal reaction force, R
Addition of Forces and Resultant Force
R=W
1 In Figure 2.76(a), the two forces of 3 N and
4 N, pushing a wooden block of mass 2 kg on
a smooth surface in the same direction cause
weight, W = mg the block to accelerate at 3.5 m s2.
(a) a = 3.5 m s-2

(b) Book on a table


Figure 2.74

4 In this section, we shall study more about the


F
equilibrium of forces, including an object at O
rest on an inclined plane where three forces

2
R
are in equilibrium. The resultant force on the a = 3.5 m s-2

CHAPTER
M
object is zero.
4
normal reaction force, R (exerted by
surface on the object)
smooth surface
n

tio
rough fric (b)Same acceleration (effect) when a force of 7 N
surface
replaces the two forces 3 N and 4 N
Figure 2.76
weight, W = mg 2 Figure 2.76(b) shows that if the two forces are
replaced with a single force of 7 N, it will still
Figure 2.75 accelerate at 3.5 m s2.
3 Thus, the effect of pushing the block with 7 N
5 A tilted surface is called an inclined plane. is the same as pushing it with 3 N and 4 N in
6 To understand better how three forces work in the same direction, i.e., the block accelerates at
equilibrium, we need to understand 3.5m s2.
(a) the resultant force of two forces, and 4 For the example discussed, 7 N is the resultant
(b) the resolution of a force. force of the combined forces of 3 N and 4 N.
5 The resultant force is defined as a single
force that will produce the same effect as the
two or more combined forces that it replaces.
To Find the Resultant Force
SPM
(I) Two Parallel Forces 09/P1

The resultant force is obtained by simple arithmetic.

Forces acting in the same direction Forces acting in opposite directions

= =

Subtract the magnitude of the smaller force by the


larger one (to find the difference in magnitude
Add the magnitudes of the two forces.
between the two forces).
The resultant force is in the same direction as the
The resultant force is in the direction of the larger
two forces.
force.
Resultant force, F=F1 + F2
Resultant force, F=F2 F1

111 Forces and Motion


(II) Two Non-parallel Forces (forces at an angle to each other acting at a point) Resultant force, FR
the tail of F1 to the

1 Simple arithmetic cannot be applied to find the resultant force of two


non-parallel forces.
For example, how to determine the resultant force of the two forces F1
and F2 acting at an angle to each other as shown in Figure 2.77?
2 Instead, we can determine the resultant force by drawing scaled Figure 2.77
diagrams using the two methods below.
SPM
Method (I) 07/P1

The Triangle Method (Tail-to-Tip Method)


F Figure 2.78 shows a scaled diagram of the triangle of forces constructed to determine the resultant force of the
O two forces, F1 and F2, acting at an angle to each other (Figure 2.77).
2

R First, choose a suitable scale for the two forces, for example, 1 cm : 20 kN.
CHAPTER

M Then, follow the steps below to determine the resultant force. Complete the triangle by
connecting the tail of F1 to
4 Draw F2 according to the the tip of F2.
Draw the force F1 according scale, with its tail starting B B
4
to the scale. from the tip of F1.
1 3 FR
F2 F2

O A F O O
F1 F1 A F1 A
O 2 5
2

R With the help of a protractor, To find the resultant force,


CHAPTER

M measure at A. measure the length OB and


the angle AOB.
4
Figure 2.78
F4/2/93

Method (II)

The Parallelogram Rule (Parallelogram of Forces)


Figure 2.79 shows a scaled diagram of the parallelogram of forces constructed to determine the resultant of two
forces, F1 and F2 acting at an angle to each other (Figure 2.77).
First, choose a suitable scale for the two forces, for example, 1 cm : 50 N.
Then, follow the steps below to determine the resultant force.
With the help of a protractor,
Complete the parallelogram Draw the diagonal of the
measure , then draw F 2 so
using a pair of compasses. parallelogram.
that the tails of both forces F 1
and F 2 start at O. 3 4
2 B
C B C
C

F2 FR
F2 F2


A O A
O O A F1
F1 F1
5
1 This diagonal represents the
Draw the force, F 1 according magnitude and direction of the
to the chosen scale. resultant force, F R.

Figure 2.79
Note that the tails of both forces F1 and F2, and the tail of the resultant force, FR are all at the same point, O and
FR is in between F1 and F2.
F4/2/94
[Note: More students prefer Method (II) to Method (I).]

Forces and Motion 112


41
An ocean liner is towed into harbour by two tugs, A and B, exerting forces of 12 kN and 10 kN
respectively and the tow-ropes making an angle of 68. Find the resultant force acting on the liner.

12 kN

68

10 kN B
F
O

2
Figure 2.80 R

CHAPTER
M

Solution 4

Use a scale of 1 cm : 2 kN (if a scale of l cm : 1 kN is used, the diagram would be too large).

4
Complete the parallelogram by
using a pair of compasses.

Arc with a radius of 5 cm


C B and the centre of circle at C.

Arc with a radius of 6 cm


and the centre of circle
at A.

3
Draw a line, OC, of 5
6 cm to represent Draw the diagonal OB.
the 12 kN force 6 cm FR
from point O.
F2

6
Measure the angle after
drawing diagonal OB.

2 68
Measure 68 from OA
38 F1
using a protractor.
O 5 cm A
1
Draw a line, OA, of 5 cm to
represent the 10 kN force.

The resultant force, FR, is represented in both magnitude and direction by the diagonal OB.
Length of OB = 9.2 cm
Magnitude of the resultant force, FR = 9.2 2 kN
= 18.4 kN
AOB = 38
The ocean liner is pulled forward by a resultant force of 18.4 kN at an angle of 38 to the 10 kN force.

Note: There is no need to draw the scaled diagram following the original directions of F1 and F2 as long as the
angle between them is correctly measured.
[Note:If you prefer to solve the problem by calculation, please refer to Example 48 in page 117.]

113 Forces and Motion


(III) Two Perpendicular Forces Figure 2.81 shows the two forces (F1 and F2)
together with the resultant force, FR using the
This is a special situation for two non-parallel forces parallelogram rule.
acting on an object at a right angle to each other. The resultant force is obtained using Pythagoras
theorem.

Resultant force, FR = F12 + F22


FR
F2 F2 F
tan = 2
F1


F1 Note: You can also solve the problem by drawing a scale
F diagram as discussed in section (II).
O Figure 2.81
2

R
CHAPTER

4 42
Samy and Heng Gee pull a crate with forces of 70N Solution
and 90 N respectively. A sketch of the forces is drawn.
Heng Gee
F
O
2

R 70 N FR
CHAPTER

M
F2 = 70 N
4
90 N

F1 = 90 N Resultant force, FR = 702 + 902


= 114 N
70
tan = = 0.7778
Samy The incorrect
90 way to determine the resultant force
using the parallelogram rule:
Figure 2.82 tan = 37.9
D force is 114N C
The resultant which acts in a direction
Find the resultant force on the crate due to these two 37.9 from the original 90 N force.
forces.
F2 FR
BD represents the
resultant force.

A F1 B

The incorrect way to determine the resultant force Remember:


using the parallelogram rule: To find the resultant force using the parallelogram
rule, the tails of the three forces FR, F1 and F2 must
D C
originate from the same point.
D C
F2 FR
BD represents the
resultant force. F2 FR
AC represents the
resultant force.
A F1 B

A F1 B

Remember:
Forces and Motion 114
To find the resultant force using the parallelogram
rule, the tails of the three forces FR, F1 and F2 must
originate from the same point.
Resolution of Forces SPM 5 The magnitudes of the vertical and horizontal
03/P2
components can be determined with
1 Since two forces can combine to give a single knowledge of simple trigonometry. We need to
resultant force, therefore by reversing the know the magnitudes only since the direction
process, a single force can also be resolved or is already known.
broken up into two components. Refer to Figure 2.83(b).
2 This reverse process is known as the
resolution of forces. For OAB: For OBC:
3 Usually, a force is resolved into components Fx Fy
cos = sin =
which are perpendicular to each other. F F
4 Figure 2.83(a) shows a single force, F. Applying
the parallelogram rule, the force F is resolved F = F cos F = F sin
x y
F
into a pair of perpendicular components: the
O
vertical force, Fy and the horizontal force, Fx.

2
R

CHAPTER
M
C B

4
Fy = F sin
F Fy F Horizontal component, Fx = F cos , and vertical
component, Fy = F sin , are not fixed formulae.
It depends on the position of .
O Fx A Fx = F cos Generally, you can take the component at the given
angle as F cos and the other component as F sin .
(a) (b) (c) Please study Example 43 carefully.
Figure 2.83

43
Find the horizontal and vertical components for each To find the vertical component:
of the following forces. Fy = F sin
= 80 sin 60
120 N = 80 0.866
60
80 N = 69.3 N upwards
(b)
60 120 sin 60

(a) (b) 120 N


60
Figure 2.84
120 cos 60
Solution
(a)
is 60 from vertical line.
80 N
80 sin 60 To find the horizontal component:
60
Fx = F sin
= 120 sin 60
80 cos 60
= 120 0.866
= 103.9 N to the left
is 60 from horizontal line.
To find the horizontal component: To find the vertical component:
Fx = F cos Fy = F cos
= 80 cos 60 = 120 cos 60
= 80 0.5 = 120 0.5
= 40 N to the right = 60 N downwards
* Please take note of the position of the angle .

115 Forces and Motion


44
A boy is pulling a toy car with a force of 6 N as shown Solution
in Figure 2.85.
6 sin 30
6N

30

6 cos 30

The horizontal component, Fx = 6 cos 30


F Figure 2.85 = 6 0.866 = 5.2 N
O Since the toy car is in dynamic equilibrium (a = 0), the
Find the horizontal component of the force. If the toy
2

R horizontal force on the toy car is balanced by the friction.


car is moving with constant velocity, what is the
CHAPTER

M Friction = 5.2 N
friction against the toy car?
4

45 SPM
03/P1

Santhiran pulls a 5 kg crate on the Ffloor with a force of 35 sin 25


O
35 N. The friction against the box is 8N. 35 N
2

R
25
CHAPTER

M
8N 35 cos 25
4 8N
35N

25
8N
Since the crate moves along the floor, only the
horizontal forces need to be considered.
Figure 2.86
Applying Fnet = ma,
What is the acceleration ofF4/2/97
the crate? 35 cos 25 8 = 5a
Solution 23.7 = 5a
a = 4.7 m s2
First, resolve the pulling force into its horizontal and
vertical components.

46
A gardener pushes a (c) If the gardener pulls the handle with the same
250 kg concrete roller force in a reverse direction, what is the total
with a force of 200 N downward force on the ground then?
directed from the handle. 33 (d) In which direction, pushing or pulling, would the
200 N
The handle is at an roller be more effective in levelling the grass?
angle of 33 to
Solution
the horizontal.
Figure 2.87 Resolve the force into horizontal and vertical components.
F4/2/98
(a) Find the horizontal and vertical components of the
force.
(b) What is the weight of the concrete roller?
What is the total downward force on the ground?

Forces and Motion 116


(a) To find the horizontal component: Resultant vertical force on the ground
Fx = F cos = Weight + Vertical component
= 200 cos 33 = 2500 + 109 = 2609 N
= 200 0.8387 (c) When the gardener pulls the roller:
= 168 N to the right The vertical component of the force
To find the vertical component: = 109 N upwards
Fy = F sin Resultant vertical force on the ground
= 200 sin 33 = Weight Vertical component
= 200 0.5446 = 2500 109 = 2391 N
= 109 N downwards (d) When the gardener pushes the roller, the net
(b) Weight, W= mg = 250 10 downward force is greater. This makes it more
= 2500 N effective in levelling the grass.
F
O

2
47 R

CHAPTER
M
Eric and Darren are pulling a trolley on which their What is
sister, Tina is sitting. Both Eric and Darren pull the (a) the total of the forces towards the left? 4

rope with a force of 50 N, which makes an angle of (b) the acceleration of Tina and the trolley?
30 with the direction of motion.
Solution
Resolving the tension forces into components.
a
50 N 50 sin 30
50 cos 30
30 40 N
40 N
30
50 cos 30
50 N 50 sin 30

(a) We need only consider the horizontal components


of the forces.
Total forces to the left
= 2 50 cos 30 40
Figure 2.88 = 2 50 0.866 40 = 46.6 N
(b) Fnet = ma
The total mass of Tina and the trolley is 30 kg and the 46.6 = 30a
friction between the trolley and the floor is 40 N. a = 1.6 m s2

To Find the Resultant Force of Two Forces at an Angle to Each Other by Calculation

48
Find the resultant force 8N
First resolve the 8 N force into its vertical and
of the two forces as 120 horizontal components.
shown in Figure 2.89. Total vertical force = 8 sin 60 = 6.93 N
12 N
Total horizontal force to the right = 12 8 cos 60
Figure 2.89 = 12 4 = 8 N
Solution F4/2/239 FR = 82 + 6.932 = 10.6 N
8N 8 sin 60 FR 6.93
tan = ____
6.93 N
60 120
8
12 N 8 cos 60 12 N 8N
= 41
The resultant force is 10.6 N at an angle of 41 to
12N.
F4/2/240

117 Forces and Motion


The Effective Components of a Weight on an Inclined Plane

1 Consider a box resting on an inclined plane at an angle to the horizontal.


2 The weight of the box is W = mg. The weight can be resolved into two perpendicular
components:
(a) the component vertical or perpendicular to the plane = mg cos ,
(b) the component down or parallel to the plane = mg sin .

F
O
2

R W = mg
CHAPTER

F s
sin mg co
mOg
2

R W = mg

CHAPTER

4
Figure 2.90

3 To solve problems involving inclined planes, weight is replaced with its two perpendicular
components. F4/2/99

Acceleration of an object on a smooth


Object in equilibrium on a rough inclined plane
inclined plane

The figure below shows a box at rest on an The figure below shows a box on a smooth
inclined plane. inclined plane.
normal reaction force, R
For an object on a smooth inclined plane, the
vertical Rcomponents of the forces are balanced.
g
n vin n
oHowever, the force downne the plane is not balanced.
tm a pla
tio
F fric
n, jec ith n th
ict
io ob wn w ratio oo
fr do cele sm R
in ac a
gs sin vin
g
m mg mo h an ne
mg cos ctmg t
i cosn pla
je th
ob wn w ratio oo
do cele sm
ac a
Since the box is at rest: sin
mg
The net force perpendicular to the plane = 0 mg cos

R mg cos = 0
Applying Fnet = ma,
The net force parallel to the plane = 0
mg sin = ma
Ffriction mg sin = 0
a = g sin
Therefore: F4/2/100
If = 30, If = 60,
R = mg cos a = 10 sin 30 a = 10 sin 60
Ffriction = mg sin = 5 m s2 = 8.66 m s2
The greater the inclined angle, the greater its
acceleration.

Forces and Motion 118


49
A carton of mass 5 kg is at rest on an inclined plane making an angle of 15 with the horizontal.
Find the frictional force and the normal force acting on the carton.
Solution
l
ma
F nor FR
g
5k
t ion
fric

15
15 50 cos 15
s in 50
15
W = mg 50 15
F
O

2
Friction, FR= mg sin Fnormal= mg cos R
= 5 10 sin 15

CHAPTER
= 5 10 cos 15 M
= 50 0.2588 = 50 0.9659 4
= 12.9 N = 48.3 N

50
Figure 2.91 shows a horizontal force, F acts on a wooden block of weight 8 N placed on a smooth plane inclined at
30 to the horizontal.
ne
pla
o th
F smo

30 W=8N

Figure 2.91

What must the magnitude of F be to prevent the block from sliding down?
Solution
Resolving the force, F and W(= mg) parallel to the plane.
(The vertical component does not play a role because when the block is in motion, it will only move along the
plane.)

30
F os
Fc
30 30
in
30 30 m gs
os sin
F c mg 30 W = mg 30
Forces along the plane

F cos = mg sin
F cos 30 = 8 sin 30
8 0.5
F =
0.866
= 4.6 N

119 Forces and Motion


Three Forces in Equilibrium

Problems involving three forces in equilibrium can be solved either by:


(a) Method A: Resolution of forces
(b) Method B: Drawing a closed triangle of forces

51
A 6 kg weight is suspended from a hook in the ceiling
as shown in Figure 2.92. A horizontal string pulls the T1
F supporting string so that the latter makes an angle of
O 50 T2
50 with the horizontal. Find the tensions in both
2

R
strings.
CHAPTER

4
6 kg

60 N

Figure 2.92
F
Solution
O
2

Method A: Resolution of forcesR Method B: By drawing a closed triangle of forces


CHAPTER

M
The slanting force, T1 is resolved as shown. Starting with any one of the three forces, a triangle
4 of forces is drawn. Lets say you start with OA
which represents the weight, 60 N.
T1 sin 50
T1 a Draw the line OA with an arrow, to represent
T2 T1 cos 50 T2 60N.
50
b Starting from A, draw line AB which represents
60 N 60 N T2.
c Then, starting from B, draw the line BO which
represents T1 to complete the triangle. The
directions of the three forces follow an
Since the weight is in equilibrium, therefore:
anticlockwise direction. However, if you draw
Vertically:
in sequence 60N, T1 and T2, youll get a triangle
of forces that show a clockwise direction.
Total force upwards = Total force downwards O O O
a b c
T1 sin 50 = 60
60 T1
T1 = 60 N 60 N
sin 50 60 N
50
60 A A B A B
= T2 T2
0.7660
After drawing the triangle, use simple trigonometry
= 78.3 N
to solve the problem.
Horizontally: 60 60
= sin 50 = tan 50
T1 T
2
Total force to the left = Total force to the right 60 60
T1 = T2 =
T1 cos 50 = T2 sin 50 50
tan
T2 = 78.3 0.6428 60 60
= = 78.3 N = = 50.3 N
= 50.3 N 0.7660 1.192

Forces and Motion 120


52
Joan and her sister lift a pail of water of total mass
5.6kg. Each of them exerts a force of F at 25 to the F cos 25 F cos 25
25 25
vertical. Find the magnitude of F. F F F sin 25 F sin 25

56 N 56 N
F F
25
25
We need to consider the vertical components of the
forces only.
F
Since the pail of water is in equilibrium:
O
Total force upwards = Total force downwards

2
R
2F cos 25 = 56

CHAPTER
M
W = mg 56 W = mg
F = 4
2 cos 25 = 5.6 10
Figure 2.93
F4/2/102 = 56 N
= 30.9 N
Solution Note:Method B is not suitable for this question as
Resolve F into its vertical and horizontal components. there is no right angle involved.

53
A lamp of weight 25 N is supported by two ropes as shown
in Figure 2.94. F
25
F
Given that the tension in rope A is 20 N, find 25
(a) the angle ,
(b) the tension in rope B.
Solution
Resolve the forces into their components.
W = mg

T F4/2/102
T cos 20 cos 30 Figure 2.94
30 20 N

T sin 20 sin 30

25 N 25 N

(a) For horizontal component: (b) Substitute = 52.5 into equation (i).
T sin = 20 sin 30 F4/2/253
T sin 52.5 = 10
T sin = 10 (i) 10
T =
sin 52.5
For vertical component:
T cos + 20 cos 30 = 25 = 12.6 N
T cos = 25 20 cos 30 Tension in rope B = 12.6 N
T cos = 7.68 (ii)
T sin 10
(i) (ii): T cos = 7.68

tan = 1.302
= 52.5

121 Forces and Motion


Each elephant in the diagram below is pulling a rope Vertical component:
attached to a mass of 10 kg. The elephants pull with a 2 5000 cos = 100
force of 5000 N. 100
cos = 2 5000

= 0.01
5000 N
= 89.4
5000 N The angle between the rope, 2 = 2 89.4 = 178.8

It is impossible to make the rope horizontal.


10 kg
See the explanation below.
F Let T = tension in this rope.
O Find the angle between the ropes. 2T cos = 100
2

R Can the ropes be pulled to become horizontal? Supposing = 90 The rope is


horizontal.
CHAPTER

M 100
Solution T =
2 cos
4 Resolve the forces into their components:
50
5000 cos 5000 cos =
cos 90
Value cannot be

50 determined.
5000 N 5000 N =
0
5000 sin 5000 sin
F
Note:You may try to use = 89.99999 and see what is
O
the value of tension in the rope, T.
2

100 N 100 N
R
CHAPTER

F4/2/103 4

11 SPM
Clone
04

The figure below shows a poster hanging on a wall. Comments


nail First, indicate the directions of tension forces along
tension,T1
the string. Since the poster is in equilibrium, the
tension,T2 three forces should then form a closed triangle of
forces.

T1
T2

T1

weight, W
W
T2
Which of the following triangles of forces shows the
forces, T1, T2 and W acting on the poster? W

A B C D
Remember:
Draw the second force from the tip of the first force.
F4/2/259
Then, draw the third force from the tip of the second
force to complete the triangle.
Answer A

Forces and Motion 122


12 SPM
Clone
06

A mirror of weight 12 N is hung on the wall using a Solution


string as shown below. 3
A
Measure 30, same
reasoning as 2 . Then
30 draw line AC to intersect
1 with BC.
Draw a vertical
T T line of 6 cm (to T
represent the
weight of 12 N).
60 60
6 cm
(12 N) C
F
O

2
R

CHAPTER
T M
2
30 Measure 30 using a 4
protractor (since the string
Draw a scale drawing of a triangle of forces to makes 30 to the vertical).
Draw a line BC.
determine the tension, T in the string. B
(Use the scale 1 cm : F4/2/104a
2 N)
Measure the length of BC.
BC = 3.4 cm F4/2/104b
Tension, T = 3.4 2 N = 6.8 N

2.9

1 Two forces of 60 N and 80 N act on an object. 4 In a circus, a monkey is pulling his chain with a
Find forward force of 20 N as
(a) the greatest and the smallest forces that can be shown in the figure.
exerted,
(b) the resultant force if the force 80 N is acting to
the right and is at right angles to the 60 N force,
(c) the resultant force if the force 80 N is acting to
the right and is at 120 to the 60 N force.
2 Two astronauts push a
800 kg satellite with 45 N

forces of 35 N and 45 N What force does the clown need to apply to the
which make an angle of chain to prevent the monkey from moving forward?
35 N
60 with each other. 5 A footballer is injured and requires traction on his
Find the resultant force foot as shown in the figure below.
and the acceleration
acquired by the satellite.

3 Aunty Anne is
ironing with a
force of 6 N as
shown in the
figure.

What is the effective force that pushes the iron If the force of the traction is 125 N, what is the mass,
forwards? m required to pull the rope?

123 Forces and Motion


6 A boy exerts a force of F to pull a box of mass 2 kg 8 A wooden block is placed on top of a smooth
up an inclined plane which makes an angle of 30 runway of length 2 m. The angle of inclination with
with the floor. the floor is 30. Find
(a) the time and
(b) the velocity of the block
when it reaches the bottom of the runway.

F
9 A bird of mass 0.6 kg stands on the midpoint of a
tion horizontal rope. The rope sags and an angle of 150
Fric N is formed.
= 3
30

Given that the friction acting on the box is 3 N, find 150


F
O (a) the normal reaction force, R acting on the box,
2

R (b) the component of the weight down the plane,


CHAPTER

M (c) the value of F if the box is moving up the plane


with an acceleration of I m s2.
4 Find the tension in the rope.

7 A 3.5 kg flowerpot is supported by two ropes as


shown in the figure. The rope attached to the wall is 10 A traffic light of mass 15 kg is suspended from
horizontal and the rope attached to the ceiling two cables as shown in the figure.
makes an angle of 30 with the horizontal.
F
40 50
O
40
2

R 50
CHAPTER

M T2
T1

m = 15 kg

Find the tension in each rope. Find the tension in each cable.

2.10 Understanding Work, Energy, Power and Efficiency


2.10 Understanding Work, Energy, Power and Efficiency
SPM
Work 03/P1

1 Work is done when a force causes an object to move in the direction of the force.

F F
s
F

s

(a) A manF4/2/105a
pushing a bicycle (b) A weightlifter lifting a barbell
Figure 2.95 F4/2/105b

Forces and Motion 124


2 The work done, W is defined as the product 4 One joule is the work done when a force of
of the force, F and the displacement, s in the one newton moves an object over a distance of
direction of the force. one metre, in the direction of the force.
Displacement in the 1m
Work done=Force 1N
direction of the force

W=F s
Figure 2.96
F4/2/106a

3 The SI unit of work is the joule, J. 1 J = 1 N 1 m = 1 N m


Work is a scalar quantity.

Work = Force Displacement (in the direction of the force) F


O

2
R

CHAPTER
M
1 Force, F in the same direction as the 2 Force, F at an angle to the displacement, s
displacement, s 4
F sin
F F
F F F cos

s s

The object moves over F4/2/106


a distance
b of s, in the The object does notF4/2/106
move in c the direction of F.
direction of the force. Instead, the horizontal component of the force,
F cos , moves the object on the surface of the floor.
W=F s Work done, W
Horizontal Displacement in the
= component direction of the force

= F cos s

W = Fs cos

54
55
A worker in a hypermarket pushes a trolley full of
goods with a horizontal force of 120 N for a Man Ling pulls a luggage with a force of 30 N that is
displacement of 15 m. What is the work done by him? applied along the handle at an angle of 40 to the
horizontal. How much work has she done after
Solution walking a distance of 15 m horizontally?
Solution
120 N 120 N

N N
30 30
15 m 40

The force and the displacement


F4/2/107 are in the same
15 m
direction.
Work done, W = F s
= 120 15 Work done, W = Fs cosF4/2/108

= 1800 J = 30 15 cos 40 = 345 J

125 Forces and Motion


SPM
No work is done when 08/P1

1 Force, F does not move 2 Force, F is at a right angle (perpendicular) to


(a) James stands still for the displacement, s
30 minutes holding some F = 20 N
A waiter walks a distance of 5 m holding a tray of
books of 20 N. What is the food that weighs 10 N. How much work has the
work done on the books? waiter done to the tray?
direction of motion
W = 20 N
F = 10 N

F
O (b) Nicholas pushes a concrete wall in his
2

R
classroom with a force of 20 N for 30 minutes.
CHAPTER

M weight = 10 N
How much work does he do on the wall?
4 wall
5m

20 N
Solution
The waiter exerts an upward force of 10 N while he
F
O is holding the tray. When he walks forward a
2

R distance of 5m, the tray is not displaced upwards or


CHAPTER

M downwards that is, the displacement in the direction


Solution of the force is zero.
4
Work done, W=F 0=0
Work done, W = F s
= 20 0 Alternative method
=0 Use the formula: W = Fs cos
Both James and Nicholas will get tired but there F = 10 N, s = 5 m, = 90
is no work done on the books or the wall as the W= Fs cos = 10 5 0 = 0
force does not move. This means that there is no work done to the tray.

Work Done against the Force of Gravity


56
1 An upward force must be applied to lift an object
Auntie Sarimah mops
of weight, mg newton, to a height of h metres.
the floor by using a
force of 9 N at an angle 4m
F (= mg)
of 60 to the floor. 60
How much work has
she done after mopping 9N
a total distance of 4 m?
h
weight
= mg

Figure 2.97
Figure 2.98
Solution 2 The work done is the same as the force
The angle between the force and displacement is 60. multiplied by the distance, in the direction of
Work done, W= Fs cos the forcethat is,
= 9 4 cos 60 where F is equal to and
Work done = F h opposite in direction to
= 18 J W = mg h the weight, mg.

Forces and Motion 126


57
A carton of weight, mg newton, is lifted to the top of a Solution
flight of stairs. (a) Work done
Displacement in the
=Force 
F direction of the force
W= mg h
= mgh
h

Reminder
Work done mg or mg b because the
mg
force, F (= mg) acts vertically over a height h. F
b
The work done is not dependent on the path O

2
Figure 2.99 taken i.e. or b, but on the vertical height R

CHAPTER
M
reached.
(a) What is the work done? F4/2/111
(b) If the mass of the carton is 2 kg and the height of 4
the stairs is 60 cm, calculate the work done by the (b) If m = 2 kg, g = 10 m s2, h = 60 cm = 0.6m
worker. [Take g = 10 m s2] The work done, W= mgh
F
=2 10 0.6
= 12 J
h


mg SPM
Energy 06/P1
b

1 Energy is defined as the capacity to do work.


2 The SI unit of energy is the joule, J. It has the same unit as work.
3 Energy is a scalar quantity. F4/2/111

Gravitational potential Elastic potential energy is


Heat energy is a type of
energy stored in an object Kinetic energy is the the energy stored in a
energy associated with
due to its position. energy that a body has due compressed/stretched spring
temperature.
to its motion. or elastic material.

FORMS OF ENERGY

Chemical energy is a stored


Sound energy is the energy which is released Nuclear energy is the Electrical energy is the
energy created when an during a chemical reaction. energy released by a energy of electric charges
object vibrates. Batteries and food store nuclear reaction. or electric currents.
chemical energy.

127 Forces and Motion


Energy Transfer and Work Done

1 Energy is transferred from one object to another when work is done.

A weightlifter displaces a barbell from the ground to a A pitcher throws a baseball towards a batter.
height above his head. Work is done to lift the barbell.

F
O Chemical energy Kinetic energy
2

R Chemical energy Potential energy


The work done transfers the chemical energy in the
CHAPTER

M
The chemical energy (stored in food) in the F4/2/113
pitchers body to the baseball. The chemical energy
4 weightlifter is transferred to the barbell which is converted to kinetic energy due to the motion of
possesses energy due to its position, that is, the the ball.
gravitational potential energy.


F
2 Work done is the medium forO transferring energy from one object to another.
2

R
CHAPTER

M
Potential Energy
4
1 The potential energy of an object is defined as the energy stored in the object because of its
position or its state.
2 There are two types of potential energy.

1 Gravitational potential energy 2 Elastic potential energy


Example: A ball raised above the floor Example: A wound coil-spring in a toy car

final position

(a) Before winding (a) After winding


ball
Elastic potential energy will be discussed in
mass = m kg initial position Section 2.12.

SPM
Gravitational Potential Energy 04/P1 F

1 A boy is lifting a box of mass m kg at a


constant velocity to a height, h metres above
the Earths surface.
2 Since the box is moving with constant
velocity, its acceleration is zero. mg
3 Thus, the net force acting on the box is zero. h
This means that the upward force, F is equal to
the weight of the box (= mg).
F = mg (in magnitude)
Figure 2.100
F4/2/114
Forces and Motion 128
4 Since the force moves a displacement of h to
lift the box, work is done by the force. 58
Displacement A pole-vaulter of mass 55 kg clears the bar at 6.0m.
Work done, W = Force in the direction What is his potential energy at the highest point?

of force [Take g = 10 m s2]
= F s Solution
= mg h
= mgh

5 What has happened to the work done?


It has changed into one form of energy which, 6.0 m
in this case, is the gravitational potential F
energy, Ep stored in the box because of its O

2
vertical position above the ground. R

CHAPTER
M
W Ep
Ep =W 4
= mgh Potential energy, Ep= mgh
=55 10 6
Ep =mgh = 3300 J

6 Figure 2.101 shows a ball of mass m displaced


from position B to C. The h represents the Kinetic Energy
vertical height.
1 Kinetic energy, Ek, is the energy possessed by
an object due to its motion.
m kg
2 Consider a force F acting on a trolley of mass
mkg on a frictionless plane over a distance of s.
C The force accelerates the trolley from an initial
velocity, u (= 0 m s1) to a velocity, v m s1.
F
s
h

B A Figure 2.102

Figure 2.101 3 What happens to the work done, W = F s?


Since
m kg the trolley possesses kinetic energy, Ek
(because of its motion), which is acquired
When the ball is displaced from position B to because of the work done, we can reasonably
C
C, work is done. conclude that the kinetic energy of the trolley
Work done, W = F s cos F s is transferred from the work done.
=Fh h

= mgh Ek = Work done


=Fs F = ma
is the angle betweenBthe force A= mas
vector and the displacement vector. v 2 = u2+ 2as
Vertical height, h = s cos (v 2 u2) (v 2 u2)
=m as =
2 2
1 2
= mv u=0
7 Therefore, an object raised to a height, h 2
possesses a gravitational potential energy, 1 2
Ep = mgh, which is independent of the path Ek = mv
2
taken by the object.

129 Forces and Motion


59
A pitcher throws a baseball of mass 145 g with a
m m
speed of 20 m s1. What is its kinetic energy?
v v' = 2v

Solution
Momentum p = mv p = mv
Kinetic energy, Ek = 2 mv
= 2p
1
= mv 2 u=0 Kinetic Ek = 1 mv2
E = 1 m(v)2
2 energy
2 2
1 2
= 2 m(2 v)
1
= 0.145 20 20 145 g = 0.145 kg
F
O
2 = 4 1 mv2

2 ( )
2

R = 29 J = 4 Ek
CHAPTER

F
O
2

Work is equivalent to energy as they are


R interchangeable.
(a) Work done = Energy acquired, and
CHAPTER

M
(b) Energy = Work done
4

Principle of Conservation of Energy

1 Figure 2.103 shows a coconut of mass, m kg 3 As the coconut falls to the ground, the
falling from a height of h metres to the gravitational potential energy, Ep decreases
ground. while the kinetic energy, Ek increases due to its
increasing velocity.
4 However, the sum of kinetic and potential
energy remains constant as the coconut falls.
5 When the coconut reaches the ground, all its
100% Ep
initial potential energy has been changed into
kinetic energy.
6 This is a typical example of the conservation
of energy.
h 50% Ep 7 The Principle of conservation of energy
50% Ek states that:

Energy cannot be created or


destroyed. It can be transformed
100% Ek
from one form to another, but the
total energy in a system is constant.
Figure 2.103

2
Before falling, the coconut possesses 8 Figure 2.104 shows another example of the
gravitational potential energy, EP = mgh. Since conservation of energy by an athlete during a
the coconut is at rest, its kinetic energy, Ek is zero. pole-vaulting event.

Forces and Motion 130


N
bar Ep(gravitational)

M
P

v Ep(elastic)

Ek
Ek(kinetic) L
Q
J K

F
O
Ek of running athlete Elastic, EP of the bending pole Gravitational potential energy, EP Ek of athlete

2
R

CHAPTER
Figure 2.104 Transformation of energy during a pole-vaulting event M

Solution 4
60
1 1
(a) Kinetic energy, Ek = mu2 = 2 102
A durian falls from a height of 6 m. What is the 2 2
velocity of the durian just before it strikes the = 100 J
ground? [Take g = 10 m s2] (b) The kinetic energy is used to overcome the
Solution frictional force.
Initial kinetic Work done to
According to the Principle of conservation of energy: =
energy overcome friction
Kinetic energy gained = Potential energy lost
Ek = F s
1 F = F rictional force
mv 2 = mgh 100 = F 5
2 m is the
mass of F = 20 N
1
v 2 = 10 6 the durian. (c) Heat energy gained = Kinetic energy lost = 100 J
2
v = 120
= 10.95 m s1
62
The velocity of the durian before it strikes the ground
is 10.95 m s1. Figure 2.106 shows a v=0

gymnast bouncing on a
trampoline. He moves
upwards at a velocity
of 8ms1. h=?
61 How high will he reach? u = 8 m s1
A moving 2 kg object has an initial velocity of [Take g = 10 m s2]
10ms1. It comes to a stop on a rough surface after
travelling a distance of 5 m.
10 m s1
stop
Figure 2.106
2 kg

frictional force
rough surface Solution
5m
Potential energy gained = Kinetic energy lost
Figure 2.105 1
mgh = mv 2
Calculate 2
(a) the kinetic energy of the object, 1
10 h = 8 8
(b) the frictional force acting on the object, 2
(c) the heat energy produced. h = 3.2 m

131 Forces and Motion


63
At a childrens playground, Calvin of 30 kg mass Solution
climbs up a concrete slide of 2.3 m height. He slides (a) Change in potential energy
down the slope that has a length of 5 m. At the end of = mgh
the slope, which is 0.3 m above the ground, his = 30 10 (2.3 0.3)
velocity is 1 m s1. = 600 J
1
(b) Kinetic energy, Ek = mv 2
2
1
= 30 1 1
2
F = 15 J
O (c) Let F be the frictional force. According to the
2

R
Principle of conservation of energy:
CHAPTER

M
Potential Kinetic Work done
+
4
energy lost = energy gained against friction
Figure 2.107
600 = 15 + (F 5)
(a) What is his change in potential energy? 5F = 600 15
(b) What is his kinetic energy at the end of the slope? F = 585
(c) Find the average frictional F force against his 5 F = F rictional force
motion along the slope. [Take Og = 10 m s2] = 117 N
2

R
CHAPTER

To pull or to lift?
The figure (a) shows a worker pulling a load of mass (c) Explain why the work done in (a) is greater than the
120kg along a 3 m inclined wooden plane onto a lorry. work done in (b) even though the gain in gravitational
The tension in the rope is 600N. potential energy of the load is the same.

Solution
worker lorry
(a) Pulling:
rope
Work done, Wa= F s
600 N
load = 600 3
= 1800 J
3 m 0.9 m (b) Lifting vertically:
road
Work done, Wb= mg h
(a) = 120 10 0.9
= 1080 J
(c) The work done in situation (a) is greater than that in
situation (b) because extra work is required to
overcome the friction between the load and the
0.9 m
inclined plane. The extra work done changes into
heat energy which dissipates to the surroundings.
(b) Extra info
(a) What is the work done by the worker to pull the load Then, why does the worker choose method (a)?
onto the lorry? Obviously, this is because he only needs to apply a lesser
(b) Instead of pulling, if the worker lifts the load vertically force to pull the load (600 N < 1200 N).
up a height of 0.9 m onto the lorry as shown in figure It is easier to raise a heavy load by pulling it along an
(b), how much work does he need to do? inclined surface than lifting it vertically. This is the function
[Take g = 10 m s1] of an inclined plane.

Forces and Motion 132


Activ To show the principle of conservation of energy
ity 2.6
Apparatus/Materials Final velocity of the trolley and the weight just
before the weight hits the soft board, v
Ticker-timer with tape and power supply, thread, x
trolley, slotted weights with holder, smooth pulley =
5 0.02 s
on a clamp, metre rule, triple beam balance, trolley
x
runway, soft board, and cellophane tape. = cm s1
0.1
2 Calculation:
Arrangement of apparatus Mass of trolley = M
Mass of weight = m F
ticker-timer Height of weight before its release = h O

2
trolley thread Final velocity of trolley and weight = v R

CHAPTER
pulley M
ticker (a) Loss of potential energy of the weight
tape
= mgh 4

weight (b) Gain in kinetic energy of the trolley and


1
the weight = (M + m) v2
soft board 2
Figure 2.108 It is found that:
F4/2/293 1
(M + m) v2 = mgh
2
Procedure
1 The mass of the trolley, M is determined using Discussion
the triple beam balance. 1 The loss of potential energy of the weight is not
2 A 150 g weight (m) is connected by a thread, converted to kinetic energy of the weight only.
which passes over a smooth pulley, to the front Part of it is converted to kinetic energy of the
of the trolley. trolley.
3 A friction-compensated runway is prepared and
the apparatus is set up as shown in Figure 2.108. 2 As the trolley and the weight are moving with
4 The height of the weight from the soft board, h the same speed, the total kinetic energy gained
is measured with a metre rule. 1
= (M + m) v 2.
5 The ticker-timer is switched on and the trolley is 2
released.
6 From the ticker tape obtained, the velocity of Conclusion
the trolley and the weight just before the weight
When the weight falls, the loss of its gravitational
hits the soft board is determined.
potential energy is equal to the gain in kinetic
energy of the trolley and the weight. Energy is
Results therefore conserved. The principle of conservation
of energy is thus verified.
1 Analysis of the ticker tape:

after the weight hits before hitting Note


the soft board, the the soft board, the In practice, you may find that
trolley moves at a trolley moves with
constant velocity increasing velocity 1
(M + m) v2 < mgh.
2
Activity 2.6

direction
of This is because part of the potential energy lost is
motion
converted to sound energy and does work against
air resistance. The principle of conservation is still
Figure 2.109 true if these are included.

133 Forces and Motion


Power SPM 3 The larger units are the kilowatt (kW) and the
04/P1
megawatt (MW).
1 Power, P is the rate at which work is done, or 1 kW = 1000 W or 103 W
the rate at which energy is transformed. 1 MW = 1 000 000 W = 106 W
4 Power is a scalar quantity as both the work
done and energy are scalar quantities.
Power, P = Work done
Time taken 5 Another unit of power is the horsepower (hp),
Energy transformed which is commonly used in electrical appliances
or P= such as air conditioners.
Time taken
1 hp = 746 W
2 The SI unit of power is the watt (W). 1 hp 3 kW
4
F 1 W = 1 J = 1 J s1
O 1 s
Power depends on 2 factors:
2

R Work or energy
P= W = E
CHAPTER

M Time
t t
4
1 If t is constant: 2 If W or E is constant:
PW 1
P t
Work done , Power
Time taken , Power
For example, F
Two librarians A and B are transferring
O books from For example,
2

the floor to a tabletop. Librarian A carries


R one book Two cars A and B of the same mass accelerate until
CHAPTER

to the table while librarian B carriesMtwo books. 80 km h1. As both cars have the same mass, both
cars do the same amount of work to accelerate to
4
the same velocity.
t=5s
v = 80 km h1
A
A

B B

t=0 t = 10 s
stationary v = 80 km h1

Car A takes 5 s to reach the velocity while car B


In 10 s, librarian B does more work (lifting 20 takes 10 s to do so.
books as compared to 10 books by librarian A). Car A takes a shorter time to complete the work.
Librarian B generates more power. Car A is more powerful.

Activ To measure the rate at which work is done by a person


ity 2.7
Apparatus/Materials
Weighing scales, stopwatch and ruler.
Arrangement of apparatus
Activity 2.7

(a) (b) (c)


Figure 2.110

Forces and Motion 134


Procedure Work done by the person in climbing the flight of stairs
= Weight Vertical height of stairs
1 The mass of the person, m is determined using a
= mgh
weighing scale.
= mgnx
2 The time, t taken by the person to climb a flight
Work done
of stairs is measured with a stopwatch. Power generated by the person =
Time taken
3 The number of steps, n is counted and the height
of each step, x (in metre) is measured using a mgnx
=
ruler. t
Conclusion
Calculation The above method can be used to measure or
compare the power outputs of two or more persons.
Mass of person = m kg F
Note:
Number of steps = n O
The above method measures only the initial power

2
R
Height of each step = x m of the person. In the process of climbing up a flight

CHAPTER
M
of stairs (say, 200 steps) without stopping, the time
Time taken = t s
taken to climb every 10 steps increases as the 4
Vertical height of stairs, h = nx m person becomes tired and hence his/her power
Weight of person = mg N, where g = 10 N kg1 progressively decreases.

64
A student who is undergoing National Service Work done, W = mgh
training has a mass of 45 kg and takes 6s to climb a = 45 10 5.76
flight of stairs of 36 steps. If each step is 16cm high, = 2592 J
how much power is generated by the student?
[Take g = 10 m s2] Power generated by the student, P
Work done
Solution =
Time taken
First, find the vertical height, h of the stairs. 2592
h = nx =
6
= 36 0.16
n = number of stairs = 432 W
= 5.76 m

Power and Velocity


65
A stuntman (mass 60 kg) in a movie climbs up onto Power, P can also be expressed in terms of force,
the roof of a building of 56 m high. If the power F and velocity, v.
generated by the stuntman is 2.1 kW, how long does Power, P
it take him to complete this task? [Take g = 10 m s2] Work done
= Work done = Force Displacement
Solution Time taken

Power generated by the stuntman = Work done = Force Displacement


Time taken Time
2100 = mg h = Force Displacement
t Time
= 60 10 56 = Force Velocity
Displacement
Change the unit of t Velocity =
Time
kilowatt to watt. 33 600 P=Fv
2.1 kW = 2100 W t=
2100
t = 16 s

135 Forces and Motion


66
A car is moving at a constant velocity of 30 m s1. Solution
Since the car is moving at constant velocity,
Constant velocity
Fnet = ma = 0 a=0
engine
thrust

friction = 700 N Engine thrust, F = Total resistance force


= 700 N
Figure 2.111
Power required, P= F v
If the total resistance force (road friction and air = 700 30
F
O
resistance) acting on the car is 700 N, what is the = 21 000 W
power needed by the car to maintain this velocity?
2

R = 21 kW
CHAPTER

Efficiency of Various Devices

1 Heat engines such as a diesel or a petrol engine, and electric motors are
two common devices that convert F energy from one form into another,
usually mechanical energy. O
2

R
2 The pie chart in Table 2.12 shows the relationship between the total
CHAPTER

M
energy input and total energy output of an efficient device and an
inefficient device respectively.4

Table 2.12
Efficient device Inefficient device
wasted energy useful energy
output output

energy useful energy wasted


input energy output input energy output

input output input output



F4/2/115a F4/2/115b

Energy input= Total energy output


= Useful energy output + Wasted energy output

In any device, not all A portion of the energy output is


the energy input is converted into heat energy as a result
converted into useful of friction or air resistance. This heat is
energy output or lost to the surroundings and cannot
useful work done. be used to produce useful work.

Forces and Motion 136


3 Efficiency compares the useful energy output to the energy input.
The efficiency of a device:

Useful energy output The efficiency of a device can also be calculated in


Efficiency = 100% terms of power.
Energy input

Eo (useful) E (useful)
Efficiency = o 100%
= 100% Ei
Ei 
Eo The useful energy
output and the


t
= 100% energy input terms
Ei are each divided
t by time.
If efficiency = 100%, then:
Useful energy output = Energy input Useful power output F
Perfect device Efficiency = 100% O
Power input

2
R

P (useful)

CHAPTER
M
= o 100%
Pi
4
However, this perfect situation is not
achievable, that is, efficiency < 100%.

4 Table 2.13 compares the efficiency of a petrol engine and efficiency of an electric motor.
Table 2.13
Less efficient device (Petrol engine) Efficient device (Electric motor)
wasted energy
(thermal energy) wasted energy
(75 J) (20 J)

input input useful energy


energy useful energy
output energy output
100 J 100 J (80 J)
(25 J)
efficiency = 25% electric motor efficiency = 80%
petrol engine

A large portion of the heat produced at high The resultant heat which originates from friction or
Wasted energy
temperature in the engine is not converted into the heating of the coils in the motor is only a small
(Internal energy
kinetic energy of the car.
F4/2/116a portion of the energy input. of the car and
other energy)

Input energy Useful output energy


100 J (Kinetik energy of
the motion of the car

To increase the efficiency of a device, we need to reduce the wasted energy output.
F4/2/116b
Case Energy input = Useful energy output + Wasted energy output Efficiency
(I) 100 J 60 J 40 J 60
= 100%
100
= 60%

(II) 80 J 60 J 20 J 60
= 100%
80

= 75%

We need only 80 J to The more efficient


produce the same device is able to
useful energy output. Same useful reduce the wasted Efficiency
energy output energy output. improved.

137 Forces and Motion


67 13 SPM
Clone
10

A petrol engine has a work output of 96 kJ per The diagram shows a weight which is connected to
minute. What is the power input if the efficiency of a wooden block through a frictionless pulley with
the engine is 20%? an inelastic string.
Solution
Inelastic string
96 000 J Frictionless table
Frictionless
Power output =
60 s pulley

= 1600 W
P (useful)
F
Efficiency = o 100% Wooden block
Pi
O Weight
Floor
2

R 1600
20% = 100%
CHAPTER

M
P
i
4 160 000 What is the energy change in the system after the
Power input, Pi = weight is released?
20
A Potential energy of the wooden block
=8000 W
Kinetic Energy of the weight
B Potential energy of the weight Kinetic
F
O
energy of the wooden block
C Potential energy of the weight Kinetic
2

R
68 energy of the wooden block and weight
CHAPTER

M
D Potential energy of the weight and wooden
A crane lifts a load of 500 kg to a4height of 120 m in block Kinetic energy of wooden block
16 s.
Comment
When the weight moves downwards, its potential
energy is converted to the kinetic energy of both
the weight itself and the wooden block (since both
120 m are moving)
Answer C

Figure 2.112

If the power input is 45 000 W, what is the efficiency


of the motor used in the crane?
[Take g = 10 N kg1]
Solution
Useful energy output = mgh
= 500 10 120
= 600 000 J Power
Energy input = Power Time Energy
=
Time
= 45 000 16
= 720 000 J
E (useful)
Efficiency = o 100%
Ei

= 600 000 100%
720 000
= 83.3%

Forces and Motion 138


14 SPM
Clone
11

A ball bearing P is released at the top of a smooth plane as shown in the figure.
Smooth plane
Ball bearing P

0.6 m
Q

What is its velocity at Q? F


A 2.5 m s 1 O

2
B 3.5 m s 1 R

CHAPTER
C 4.9 m s 1 M

D 5.8 m s 1 4
Comments
The potential energy of the ball bearing at P is converted to the kinetic energy at Q.
Solution
Kinetic energy at Q = Potential energy at P
1 2
mv = mgh where m is the mass of the ball bearing
2
v = 2gh
= 2 10 0.6
= 3. 5 m s1

Answer B

2.10

1 The figure shows a weightlifter lifting a barbell of 80 kg.



SPM
Clone
What is the work done on the barbell?
06

1.8 m
1.7

2 A waiter carries a tray full of dishes weighing 30 N from the kitchen to a customer who is 5 m away.
What is the work done on the tray and dishes?
3 What is the work done by a workman carrying a bag of cement of mass 30 kg up a flight of stairs with
20 steps, if each step is 15 cm in height?

139 Forces and Motion


4 A nurse pushes a patient in a wheelchair with a force 9 Saidin, a fireman, slides down a pole when the fire
of 30 N for a distance of 25 m. alarm sounds. He reaches the ground at 5 m s1.
Given that his mass is 60 kg, find the friction that
acts against his motion as he slides down the pole.

30 N 30 N

25 m 3.0 m

What is the work done by the nurse?


F 5 The figure shows two trucks pulling a ship in a canal
O of length 1.6 km. The tension in each cable is
2

R 5000N.
CHAPTER

M
10 Simon rides his bicycle down a slope of 5 m vertical
4 height without pedalling his bicycle. The velocities of
the bicycle before and at the end of the slope are
2ms1 and 8ms1 respectively.
u = 2 m s1

F
O
2

R
v = 8 m s1
CHAPTER

Find the total work done on the M


ship.
5m
6 A fireman of a mass of 70 kg climbs
4
a 10-m ladder
to rescue a girl.

Given that the mass of Simon and his bicycle is


70kg, find
(a) the total mechanical energy before Simon
moves down the slope,
30 (b) the work done by Simon against friction along
the slope.
11 A sprinter runs at a steady speed of 8 m s1 against a
resistance force of 35 N. What is his power?
12 In a hypermarket, a machine can lift a crate of
500 kg up onto a shelf as shown in the figure. The
If the ladder is inclined at an angle of 30 to the wall
F4/2/305 time taken for the machine to do this work is
as shown in the figure, how much work has he done?
1minute.
7 A 7 kg bowling ball is rolling at 3 m s1. What is its
kinetic energy?
8 The figure shows a girl on a swing. She is released
from a height of 1.5 m.
2m
500 kg

initial
position
(a) What is the minimum power needed?
v=?
(b) If the machine F4/2/309
is operating at an efficiency of
1.5 m
80%, what is its power input?
13 A pump can force 300 kg of water to a height of
0.3 m
12m in 20s. Find its useful power. If the power
input is 2 kW, what is the efficiency of the pump?
Find her maximum velocity.

Forces and Motion 140


2.11 Appreciating the Importance of Maximising the Efficiency of Devices

2.11 Appreciating the Importance of Maximising the Efficiency


of Devices
1 It has been explained in Section 2.10 that the If the useful energy output (or work done) is
efficiency of diesel and petrol engines is low as constant, then:
a large portion of energy is lost as heat or 1
Efficiency
sound energy. Energy input
2 The efficiency of an electric motor is high but
the efficiency of an electrical generator at a Efficiency , Energy input
power station is low (about 30%).
3 The main sources of energy of an electrical 8 With higher efficiency, fuel-consuming
generator in a power station are fossil fuels devices will require less fuel to produce the F
such as coal, petroleum, and natural gas. same amount of work and hence, the O

2
4 Similarly, vehicles (for transportation) derive R
availability of existing sources of fossil fuels

CHAPTER
M
their energy from petrol and natural gas. Thus, will be prolonged.
fossil fuels are of utmost importance to us. 9 The same reasoning is applicable to other 4
5 It must, however, be noted that fossil fuels are devices. Since less energy input is required to
not renewable and these sources of energy will produce the same amount of work or useful
be depleted some day. energy output, the cost of operating the device
6 Apart from looking for alternative sources of is reduced (we need to pay for energy
energy or increasing the source of renewable consumption).
energy, efforts must be made to maximise the 10 Besides conserving resources, maximising the
efficiency of fuel-burning devices. efficiency can also reduce the heat output or
7 From the formula of efficiency: sound production which might be detrimental
Useful energy output to the environment.
Efficiency = 100%
Energy input

Ways of Increasing the Efficiency of


Devices
Electrical Devices life span is much longer since it does not
have a filament which can burn out when it
Light Fittings overheats.
1 Replace filament light bulbs with fluorescent 3 Use a lamp with a reflector so that the
lamps which have a higher efficiency, preferably illumination can be directed to specific areas.
compact fluorescent bulbs. In this way, the required illumination can be
obtained with a smaller number of light bulbs.

Air Conditioners
1 Choose a brand/model with a high efficiency.
2 Generally, a higher capacity unit has a higher
efficiency but the choice of an air conditioner
is determined by the users requirement and
Filament light bulb Compact fluorescent bulbs the size of the room. Central air conditioners
(Efficiency: 2%) (Effeciency: 79%) have high efficiencies.
Figure 2.113 3 Ensure that windows are closed and gaps
under doors are sealed when using an air
2 The illumination efficiency of a fluorescent conditioner so that the temperature in the
bulb is four times that of a filament bulb. Its room can be maintained.

141 Forces and Motion


Refrigerators 2 When less water is being used, the
1 Choose the capacity of a refrigerator according consumption of electricity to heat up the water
to the size of the family. is correspondingly reduced.
2 The refrigerator must be installed away from 3 A front-loading washing machine uses
sources of heat and direct sunlight. The less detergent compared to a top-loading
distance from the wall must be more than machine.
3cm to ensure there is good air flow.
3 The doors of the refrigerator must always be
shut tight.
4 Use a refrigerator with a manual defrost device
Operation of Electrical Devices
such a refrigerator can save as much as
1 The management of electrical devices includes
3050% electrical energy when compared to a
F
ensuring that the devices are in good operating
refrigerator with an automatic defrost device.
O condition.
However, defrosting must be carried out
2

R 2 If electrical devices function efficiently, it


periodically to maintain the efficiency of the
CHAPTER

M increases their life span.


refrigerator.
3 Devices must be inspected regularly and
4 5 In terms of energy efficiency, it is more
serviced.
economical to use a large capacity refrigerator
4 For example, the filter in an air conditioner
than a smaller capacity unit.
and the fins of the cooling coil of a refrigerator
Washing Machines must be periodically cleaned to ensure there
1 Use a front-loading machine,
F which is is good air flow and to maintain their efficiency.
more economical in waterO and electricity
2

consumption. R
CHAPTER

M
SPM
2.12 Understanding
4 Elasticity 08/P2(B)

2.12 Understanding Elasticity


Meaning of Elasticity

1 Figure 2.114 shows that the shape and the size of an object change
when an external force is applied on it.

when when
force is applied force is removed





when when
force is applied force is removed






Figure 2.114

2 When the external force is removed, the object returns to its original
shape and size.
3 The property of an object that enables it to return to its original
shape and dimensions (sizes) when an applied external force is
removed is called elasticity.
4 Rubber and plastic are examples of elastic materials.
5 Soap or plasticine is deformed when an external force is applied on
them. The deformation is permanent even after the external force is
removed that is, these materials are non-elastic.

Forces and Motion 142


force of
Forces between Atoms repulsion
force of repulsion
increases when
(positive)
x decreases
1 The property of elasticity is caused by the equilibrium distance
existence of forces of repulsion and attraction
between molecules in solid materials. distance of
separation between
2 For simplicity, we will discuss elasticity in xo
2 atoms, x
relation to metals only.
force of attraction
3 Forces of repulsion and attraction between decreases with distance, x
force of force of attraction
atoms are always present in metals in a solid attraction increases with
state. (negative) distance, x
4 In the absence of an applied external force Figure 2.115
on a metal rod, atoms vibrate about their
F
equilibrium positions. The atoms are acting between the two atoms is a force of O

2
separatedby a distance called the equilibrium repulsion. When the external force is R
distance, xo. At this equilibrium distance, the

CHAPTER
removed, the repulsive force pushes the atoms M
force of attraction is balanced by the force of back to their original position and the metal
4
repulsion the resultant force is therefore rod returns to its original length.
zero. 7 When a stretching force is applied to the two
5 The graph in Figure 2.115 shows the ends of the metal rod, its length increases. As
relationship between the resultant force and a result, the separation distance between the
the distance between two atoms, x. two atoms is increasedthat is, x > xo. The
6 When a compressive force is applied to the resultant force acting between two atoms is a
two ends of the rod, the length of the rod is force of attraction. When the external force is
reduced and hence the separation distance removed, the force of attraction pulls the
between the two atoms is reducedthat is, atoms back to their original position and the
x < xo. The resultant force (or effective force) metal rod returns to its original length.

Equilibrium
F F F F

MetalMetal rod under


Metal rod is compressed, length rod
no in equilibrium
external force Metal rod is stretched,
of rod decreases. condition. length of rod increases.
F4/2/120b
F4/2/120d

F4/2/120f

x < xo Arrangement of atoms x > xo

xo
F F F F

Metal rod under compression: Metal rod under tension:


Equilibrium condition:
F4/2/120c
1 Force of repulsion takes 1 Force of attraction takes
F4/2/120e 1 Force of repulsion F4/2/120g
effect. effect.
2 When the compressive force = Force of attraction
2 When the stretching force is
is removed, the force of 2 Resultant force = 0
removed, the force of attraction
repulsion between the atoms between the atoms pulls the
pushes the atoms back to atoms back to their equilibrium
their equilibrium positions. positions.

143 Forces and Motion


2.4

Relationship between force and extension of a spring


Situation
Azizi and Kamaruzzaman, who are 1 year old and
3 years old, are sleeping in their sarong cradles.
The springs of both cradles are identical and new.
Observe the babies and the springs in the Figure
2.116. What inference can you make?

F Inference
O
The extension of the spring depends on the weight or
2

R
force acting on it.
CHAPTER

M (a) Azizi (1 year old) (b) Kamaruzzaman


(3 years old)
4
Hypothesis
Figure 2.116
The greater the stretching force, the longer the
extension of the spring.

F
Aim O
2

To investigate the relationship between


R a stretching force (weight) and the extension of a spring.
CHAPTER

Variables 4
(a) Manipulated : weight of load, F
(b) Responding : spring extension, x
(c) Fixed : force constant of the spring, k

Apparatus/Materials
Spring, pin, slotted weight, weight holder, retort stand with clamp, metre rule, and plasticine.

Arrangement of apparatus

metre rule zero mark

clamp

spring

pin as pointer
retort
stand o o
extension, x
plasticine

Experiment 2.4

weight holder
weight

(a) (b)
Figure 2.117

Forces and Motion 144


Procedure
1 The metre rule is clamped vertically to the retort stand, with its 0 mark at the top.
2 The spring is suspended from the clamp.
3 The pin is attached to the bottom end of the spring with plasticine.
4 The initial position of the pin, o is noted.
5 A weight holder of mass 50 g is hooked onto the bottom of the spring. A slotted weight of mass 50g
is added to the weight holder. The total weight acting on the spring is noted and the new position of
the pin, is recorded.
6 The extension of the spring is calculated from the formula: x = o
7 Step 5 is repeated with the addition of 50 g each time until a maximum of 300 g.
All the respective readings are noted and recorded.
F
Tabulation of data O

2
Original position of the spring = o cm R

CHAPTER
M
Table 2.14
4
Load, Stretching Position Spring
m (kg) force, of pin, extension,
F = mg (cm) x = ( o )
(N) (cm)
0.10 1.0
0.15 1.5
0.20 2.0
0.25 2.5
0.30 3.0

Graph
The graph of spring extension, x against stretching force, F is drawn.
x (cm)

F (N)
O

Figure 2.118
Conclusion
The straight line in the x-F graph passes through the origin. This shows that the extension of the spring,
x is directly proportional to the stretching force, F.
The hypothesis is valid.

Hookes Law
Experiment 2.4

1 Hookes law states that the extension of a spring is directly


proportional to the stretching force acting on it provided the
elastic limit of the spring is not exceeded.
2 In Experiment 2.4, the spring returns to its original length when
the load is removed that is, the spring is elastic.

145 Forces and Motion


3 As the addition of weights continues (the stretching force increases), a situation will be reached
where there is permanent extension to the spring when the load is removed.
4 The elastic limit of a spring is the maximum stretching force which can be applied to the
spring before it ceases to be elastic, i.e. permanent extension or permanent stretching is
produced.

SPM SPM
04/P2 09/P1
Graph of Stretching Force, F against Spring Extension, x

1 Figure 2.119 shows a graph of stretching force, F against the extension of a spring, x.
F
Q P
E
F F = kx spring not
O spring obeying obeying Hookes law
Hookes law
2

R
CHAPTER

4
x
O R

Figure 2.119

2 Section OE of the graph is a straight


F line passing through the origin.
Thus, the stretching force is Odirectly proportional to the extension of the spring (that is, the
2

R
extension is directly proportional to the stretching force) and Hookes law is obeyed.
CHAPTER

M
3 Beyond point E (EQP), the graph takes the form of a curve that is, the stretching force does
not vary directly with the extension
4 and Hookes law is no longer applicable.
4 If the load is removed at point Q, the spring will contract but with a permanent extension,
represented by line OR.

Force Constant or Spring Constant

1 Figure 2.120 shows the linear portion of a graph of the stretching force against the extension of
a spring.
F

a x
O

Figure 2.120

2 The graph is a straight line passing through the origin. Hence:


F x
F=kx
where k is a constant known as the force constant or spring constant.
3 From the formula : F = kx
F (N)
k =
x (m)
The unit of k is N m .
1

Forces and Motion 146


4 From Hookes law: F = k x
Equation of straight line: y = m x, where m is the gradient.
k=m
k= b
a
= gradient of the F-x graph
5 The value of the force constant is a measure of the stiffness of the spring.

Force Constant and Stiffness of Spring

To understand the meaning of the force constant, look at the following examples: F
O

2
R
(a) A load of 2.5 N extends spring A by 20 cm. (b) A load of 15 N extends spring B by 20 cm.

CHAPTER
M
Applying F = kx, Applying F = kx,
F F 4
k = k =
x x
2.5 N 15 N
= =
0.2 m 0.2 m
= 12.5 N m1 = 75 N m1

F (N) Spring A requires 12.5 N to be


B
g extended by 1 metre whereas
in
75
spr
stiff spring spring B requires 75 N for the same
ng A extension.
spri
12.5 Spring B is stiffer than spring A.
soft spring

x (m)
O 1

Force constant, k = Gradient of graph


1 A larger value of k indicates a stiffer spring.
2 A steeper graph indicates a stiffer spring.

Force and compression of spring



Hookes law is also applicable in the case of the
compression of a spring where the compression of the
spring (a decrease in length) is directly proportional to
the compressive force.
F

x = 0 

Physics Blog
147 Forces and Motion

F4/2/121
69
A spring has an original length of 20 cm. When a load When 200 g is added, load = 300 + 200
of mass 300 g attached to it, the length of the spring is = 500 g
extended to 26 cm. Calculate the force constant, in = 0.5 kg
Nm1, of the spring. What is the length of the spring Stretching force= mg
when the load is increased by 200 g? =0.5 10
[Take g = 10 N kg1] = 5 N
Solution
From the formula: F = kx
m = 300 g = 0.3 kg
Stretching force= mg Extension, x = F
F k
O =0.3 10 = 3 N 5
=
2

R Extension, x= 26 20 50
= 6 cm = 0.06 m
CHAPTER

M
= 0.1 m
From the formula: F = kx
4 = 10 cm
F
k =
x
Length of the spring= 20 + 10
3
= = 30 cm
0.06
= 50 N m1F
O
2

R
CHAPTER

Application of Hookes Law on Springs


4 From Hookes Law, for a spring under two separate
forces F1 and F2 (or loads W1 and W2) with
Many questions do not need the calculation of the corresponding extensions x1 and x2:
force constant, k. F1 = kx1
The following method explained in these examples F2 = kx2 The stretching force is
is suggested. F1 x1 proportional to the
Dividing, = extension of the spring.
F2 x2

In the situations for loadextension:


W1 x1
=
0
W2 x2
1 2 m1 x1
Since W = mg, we obtain: =
m2 x2
x1 = 1 0
x2 = 2 0 Therefore:
F1 F1 x1 F1 1 0
= =
F2 F2 x2 F2
m1
2 0
m1 x1
m2 or = m1 1 0
m2 x2 or =
m2
2 0
Figure 2.121 Used when one
F4/2/122 extension or both Used only when
extensions are given, needed to find the
0 = the original length of a spring or easily determined original length or
1 = length of spring for F1 and m1 from the information reading on the scale
2 = length of spring for F2 and m2 given. when no load is
Used more often. attached (please refer
to Example 72).

Forces and Motion 148


70 71
The original length of a spring is 10 cm. With a load A 30 cm spring is compressed to 25 cm when a load
of mass 300 g, the length of the spring is extended to of 300 g is placed on it. What additional load is
16 cm. required to compress the spring to 22 cm?
(a) What is the length of the spring with a load of Solution
400g?
(b) What load is required to extend the spring to m1 x1
Applying = ,
19cm? m2 x2
Compression:
300 5 x1 = 30 25 = 5 cm
Solution = x2 = 30 22 = 8 cm
m2 8
m1 x1
(a) Applying = , 8 300 F
m2 x2 m2 = O
5

2
x1 = 16 10 R
300 6 = 6 cm
=

CHAPTER
= 480 g M
400 x2
Additional load= 480 300 = 180 g
6 400 4
x2 =
300
= 8 cm 72
Length of spring = 10 + 8
= 18 cm Figure 2.122 shows the two positions of a pin when a
spring is loaded with two different weights.
Alternative methods
ruler
m 1 0
(i) 1 =
m2
2 0 10 cm 10 cm
300 16 10
= 15 cm 15 cm
400 2 10

3 6 20 cm 20 cm
=
4 2 10 300 g
25 cm 25 cm
32 30 = 6 4
500 g
32 = 54
2 = 18 cm (a) (b)
Length of spring = 18 cm Figure 2.122
(ii) 300 g 6 cm Extension What is the reading of the pin when the spring is
= (16 10) cm without a load?
100 g 2 cm = 6 cm
400 g 4 2 cm Solution
= 8 cm
Let 0 = reading of the pin when the spring has no
Length of spring = 10 + 8
load.
= 18 cm
m1 1 0
Applying
m2 = 2 0 ,
m x
(b) Applying 1 = 1 ,

m2 x2
300 20
x1 = 16 10 = 0
300 6 500 25 0
= = 6 cm
m2 9 x2 = 19 10
= 9 cm 3 20
9 300 = 0
m2 = 5 25 0
6
75 30 = 100 50
= 450 g 20 = 25
A load of 450g is required. 0 = 12.5 cm

149 Forces and Motion


Factors Affecting the Stiffness of a Spring

Activ To investigate the factors that affect the elasticity of a spring


ity 2.8
Apparatus/Materials
A pair of steel springs, P and Q of different lengths, a pair of steel springs, R and S with wires of different
diameters (different thickness), a pair of steel springs, T and U with coils of different diameters, a steel spring, V
and a copper spring, W of the same dimension, pin, slotted weights, weight holder, retort stand with a clamp,
metre rule and plasticine.
F
O Procedure
2

R
 1 The apparatus is set up as shown in Figure 2.117 (Experiment 2.4).
CHAPTER

M
2 Steps 1 to 6 of Experiment 2.4 are repeated using the pair of springs of different lengths.
4 3 A force-extension (F-x) graph is plotted for each spring for comparison.
4 Steps 1 to 3 are repeated with springs of different thickness, springs with coils of different diameters and
springs of different type of materials. Please take note that in each case, all other physical factors remain
constant while one particular factor is being investigated.
F
Results and Inferences O
2

R
CHAPTER

M Table 2.15

4 Thickness of
Length of spring Coil diameter Type of material
spring wire
Steel springs Steel springs Steel springs Same dimensions

P Q R S T U V W
Factors
investigated

shorter smaller
larger
(thicker) smaller larger steel copper
longer spring spring

F (N) F (N) F (N) F (N) F (N) F (N) F (N)
spring P spring R spring R spring T spring T spring V spring T

spring Q
F-x graph spring S spring S spring U spring U spring W sprin

O x (cm) O
O x (cm) O O x (cm) x (cm) O x (cm) O
x (cm) x

The shorter spring, P The spring with the The spring with the
F4/2/123 The steel spring, V is
Conclusion is stiffer. wire of a larger coil of a smaller stiffer than the copper
Activity 2.8

diameter (thicker), diameter, T is stiffer. spring.


R is stiffer.

Forces and Motion 150


System of Springs
0
Identical Springs

Note: For each case below, when load = W, spring extension = x (applied to x

a spring alone). W
Note: o = length of original spring
Figure 2.123
Table 2.16

Series arrangement Parallel arrangement

F
0 0 0 0 O

2
R

CHAPTER
M
x x 1x 11
xx 1x
2 22 2
W W 4

0 0

x x

W W

The same load is applied to each spring. The load is shared equally among the springs.
Tension in each spring = W W
Tension in each spring =
Extension of each spring = x 2
Total extension = 2x x
Extension of each spring =
Length of spring system 2
= Total original length + Total extension x
Length of spring system = o +
= 2o + 2x 2
If n springs are used: If n springs are used:
The total extension = nx x
The total extension =
n

73
Figure 2.124 shows a Solution
series arrangement of First, find the extension of a single spring (very
three identical springs. important).
The original length of
each spring is 16 cm. Extension, x1 = 20 16 = 4 cm
p m1 x1
With a load of 200 g, Applying
m2 = x2 ,
the length of each
spring becomes 20 cm. 200 4
=
What is the length p 300 x2
of the system shown 4 300
300 g x2 = = 6 cm
in the figure? 200

Figure 2.124 Total extension = 3 6 cm = 18 cm
p = Total original length + Total extension
= (3 16 cm) + 18 cm = 66 cm

151 Forces and Motion


74
Figure 2.125 shows a system For spring P :
of 6 identical springs. m1 x1 x1 = 15 12
S T U = = 3cm
The original length of each m2 x2

spring is 12cm. When a load 200 3
of 200 g is added, the length =
y 300 x2
of each spring extends to Q R

15cm. 200x2 = 3 300
What is the length y of the x2 = 4.5 cm
spring system? For spring Q or R :
P
F x 4.5
Extension = 2 = 2
O
2

300 g
R
Extension = 2.25 cm
CHAPTER

M
Figure 2.125
For spring S, T or U :
4 x 4.5
Solution Extension = 3 = 3
First, find the extension of a single spring, x.
x Extension = 1.5 cm
For 2 springs in parallel, extension =
2
F
x
For 3 springs in parallel, extension
O =
3
2

R S T U 12 + 1.5
CHAPTER

4
extension = x
3
Q R 12 + 2.25

extension = x
2

P 12 + 4.5

extension = x
300 g
W

y= Total original length + Total extension


Length of spring system = 12 3 + (4.5 + 2.25 + 1.5)
= Total original length + Total extension = 36 + 8.25 = 44.25 cm

Non-identical Springs Arranged in Series

75
Springs A and B are two non-identical springs. Spring
10 cm
A
12 cm
B A has an original length of 10 cm which stretches to
15 cm 18 cm A 12cm with a load of 300 g. Spring B has an original
length of 15 cm and extends to 18 cm with a load of
300 g 500 g. What is the length y of the spring system with a
y
500 g B
load of 800 g?

800 g

Figure 2.126

Forces and Motion 152


Solution For spring B:
x1 = 18 15
m1 x1 500 3 = 3cm
=
Applying
m = x , 800 b
2 2
For spring A: Extension, b= 800 3 = 4.8 cm
300 2 x1 = 12 10 500
= = 2cm
y = Total original length + Total extension
800 a
= (10 + 15) + (5.3 + 4.8)
Extension, a= 800 2 = 5.3 cm = 35.1 cm
300

Elastic Potential Energy 5 Work done, W


= Average force Displacement, x
F
1 Elastic potential energy is the energy = 0 + F x O

2
stored in a spring when it is extended or 2 R

CHAPTER
M
compressed. = 1 Fx
2 The elastic potential energy is a result of the 2 4
work done to extend or compress the spring. =Area of the shaded triangle under the graph
3 Figure 2.127(a) shows a spring with an of force against extension
original length of o being extended and
Substitute F = kx into the formula.
compressed with a force that is increased from
zero to F. The extension and compression Work done, W = 1 kx x = 1 kx2
2 2
have the same value x. Figure 2.127(b) shows
the graph of force against the extension/ Elastic potential energy, Ep is given as:
compression of the spring.
EP=1 kx2 because
2 EP = W
o

x 76
spring
F
compressed
x A force is increased from 0 to 30 N and applied to a
spring, compressing it by 12cm as shown in Figure
2.128. force, F (N)

F
spring extended 30

(a)
force F4/2/329(a)
compression (cm)
12

F Figure 2.128

Area = 1 Fx
Find
2 (a) the force constant, k of the spring,
= Work done
= Elastic Potential Energy (b) the elastic potential energy stored.
Solution
Change the unit
O x extension/ compression (a) k = gradient of F-s graph of compression
30 N to metres.
(b) =
0.12 m = 250 N m
1
12 cm = 0.12 m
Figure 2.127
(b) Ep = Area under the F-s graph
4 The work done on the spring is not equal
1
to F x, because the force is not constant but is = 2 0.12 30 = 1.8 J
increasing from zero to F.

153 Forces and Motion


The graph in Figure 2.130 shows the relationship
77 between the applied force F, which increases non-
A spring with a 300 g sphere attached to it is uniformly with the extension, x of the elastic band of
extended by 5 cm. The force constant of the spring is a catapult. The mass of the stone is 25g.
200 N m1. (a) What is the work done to stretch the elastic band
of the catapult?
(b) What is the elastic potential energy stored in the
elastic band?
5 cm 300 g (c) If the stone is released, what is its velocity as it
leaves the catapult?

smooth surface
Solution
F (a) The number of squares covering the area under
Figure 2.129
O the Fx graph are estimated and multiplied by the
2

R value of each square. A partial square, whose


What is the elastic potential energy stored in the
CHAPTER

M
spring? What is the maximum velocity of the sphere area is greater than half the area of a full square,
4 when the stretching force is released? is counted as a complete square. Conversely, a
square that is less than half filled is not counted.
Solution
Number of squares = 28
Extension, x = 5 cm = 0.05 m
1 Value of 1 square = 0.5 N 0.02 m = 0.01 J
Elastic potential energy, Ep = kx2
2 F Work done, W = 0.01 J 28 = 0.28 J
1 O (b) Ep = Work done = 0.28 J
2

= R200 0.052 (c) Ek = Ep = 0.28 J


2 M
CHAPTER

1
= 0.25 J mv2 = 0.28
2
Maximum kinetic energy gained 4
= Elastic potential energy lost 1
0.025 v2 = 0.28
1 2 Velocity, v is 2
mv = 0.25 maximum when
2
the spring is not v2 = 0.28 2
1 under tension or 0.025
0.3 v2 = 0.25
2 compression, i.e., = 22.4
when the original
2 0.25 length is restored. v = 4.7 m s1
v2 =
0.3
v = 1.666
= 1.29 m s1 79
A spring is compressed with a force of 300 N.
If the potential energy stored is 6 J,
78 (a) what is the compression of the spring?
elastic band
(b) determine the force constant of the spring.
stone Solution
x 1
(a) Elastic potential energy, Ep = Fx
F 2
1
F (N)
6 = 300 x
2
3.0
2.5
Compression, x = 0.04 m = 4 cm
2.0 (b) Use the formula F = kx,
1.5 F
1.0 k =
x
0.5
x ( cm ) 300
2 4 6 8 10 12 14 16 = = 7500 Nm1
0.04
Figure 2.130

Forces and Motion 154


Uses of the Property of Elasticity

Weighing Spring shock Baby hammock Mattress and


apparatus absorbers cushion
A weighing apparatus Spring shock absorbers The spring attached to The springs in a
uses the elastic property are mounted on the the baby hammock mattress or cushion
of a spring. When a load wheels of cars and enables it to bounce undergo many cycles of
is being weighed, the motorcycles to absorb vertically when the compression during its
spring is either extended, impact and reduce hammock is displaced use yet the mattress or
or compressed, in direct vibrations resulting downwards slightly. cushion always returns
proportion to the load from uneven road The bounce helps babies to its original shape.
whose value is registered surfaces. to fall asleep. This is due to the
on the linear scale of the elasticity of the F
O
apparatus. If there is no springs.

2
R
load, the spring in the

CHAPTER
M
weighing apparatus
returns to its original 4
length which
corresponds to the zero
reading on the scale.

Uses of springs
Voltmeter,ammeter, in daily life Toys
galvanometer,spring
operated clocks and
watches The toys below move
using the power of
The potential energy springs. The work done
stored in the spiral in compressing the
springs is converted into spring is converted into
kinetic energy which Tyre pressure elastic potential energy,
Door closer gauge
turns the hands of the which is then converted
clock and the pointer of This device has a spring When the nozzle of the into the kinetic energy of
the meter until the system and is mounted gauge is fixed on to the the toys, thus making the
springs return to their on the door to close it valve of the tyre, the air toys move.
original state. automatically. When pressure in the tyre pushes
the door is open, the a piston which compresses
elastic potential energy a spring inside the gauge.
stored in the compressed This allows us to read the
spring closes the door. amount of pressure in the
tyre.
door closer

155 Forces and Motion


The material of trampoline has The elasticity of a springboard
The pole in a pole-vaulting event enables a diver to jump high in
uses the property of elasticity to an elastic property to help a
gymnast bounce upwards. the air before diving into the
F help the pole-vaulter to leap over pool.
O the bar.
2

R
CHAPTER

M
Other elastic material
4

F
O
2

R
As the bow bends, the elastic
CHAPTER

M
twine of the bow is stretched to The elasticity of the strings of a The double clips and clothes
store the elastic potential energy 4 tennis or a badminton racket pegs make use of the spring
which propels the arrow when it allows a ball or shuttlecock to between the clips to hold things
is released. bounce off them. tightly.

15 SPM
Clone
11

Figure (a) and (b) shows the set up to study the elasticity of spring X and Y and the corresponding force-length graphs.
Force
X

Length of
spring
(a) (b)

Which comparism about spring X and Y is correct? Comment Force


X
A X is stiffer than Y As shown in the diagram,
B To stretch 1 cm, work done needed on X is less for the same force
than on Y applied, spring X Y
C The original length of X is longer than Y extends less than spring
F
Y. Therefore, spring X
is stiffer than spring Y Length of
spring
Answer A

Forces and Motion 156


2.12

1 A spring of length 20 cm is extended to 26 cm


when pulled by a force of 10 N. What is the length
of the spring when it is compressed by a force of p
20N?
2 The figure below shows a spring which is compressed. 450 g

m 500 g
m
(b)

What is the length of the spring system in Figure (b)?


12 cm 10 cm All the springs are identical.
F
6 The figures show a spring system comprising O

2
identical springs. R
Calculate the value of m if the original length of the

CHAPTER
M
spring is 15 cm.
3 The figure shows the pointer reading of a spring. 4
When an additional load of 200 g is placed in the
pan, the pointer reads 25 cm.

20 cm
300 g 25 cm
600 g 300 g

What is the reading of the pointer when the total


load is removed? Each spring is of length 18 cm and extends to 22
cm when subjected to a load of 200 g. What is the
4 M and N are two non-identical springs each
length of the spring system in the figure?
measuring 12 cm. When subjected to loading, their
respective lengths are as shown in the figure (a) 7 Figure (a) shows a spring of length 20 cm
below. compressed to a length of 15 cm by a load of 250g.

15 cm N 250 g
M
18 cm m
y
300 g
15 cm 12 cm
500 g

600 g

(a) (b) (a)


What is the length y of the spring system if M and N
250 g
are arranged in series and the load applied is 600 g? m
5 Figure (a) shows two springs arranged in parallel,
each of length 9 cm, subjected to a load of 600 g.
15 cm 12 cm

12 cm
(b)

600 g
What is the value of m in Figure (b) if three identical
(a) springs are compressed to a length of 12cm?

157 Forces and Motion


8 The figure is a graph of extension, x against load, m 9 The figure shows a sphere of mass 20 g pushed
for a spring. against one end of a spring on a smooth surface.
The original length of the spring is 18 cm and its
x (cm)
force constant is 12.5 N m1.
a
12 cm 12 cm 20 g

compression
force
O m (g) smooth surface
600 800

(a) What is the elastic potential energy stored in the


(a) What is the value of the force constant, k? spring?
F (b) What is the value of a? (b) What is the maximum velocity reached by the
O (c) What is the potential energy stored when the sphere when the compressive force on the
2

R spring is extended by a cm? spring is removed?


CHAPTER

F
O
2

1. Speed is the rate of change of distance.


R the collision is less than the kinetic energy before
CHAPTER

M
Distance travelled the collision.
Speed =
Time taken (b) In an elastic collision, momentum, total energy
4 and kinetic energy are conserved.
2. Velocity is the rate of change of displacement.
Displacement 10. Acceleration is proportional to the applied force but
Velocity = inversely proportional to the mass.
Time taken
11. Impulse is the product of a force and the time
3. Acceleration is the rate of change of velocity.
it acts.
vu
a= Impulse = Ft
t
12. Impulse is equal to the change in momentum.
4. Equations of linear motion with constant acceleration: Ft = mv mu
v = u + at 13. The impulsive force is reduced by prolonging the
1 time of impact.
s = (u + v)t
2 14. When forces are in equilibrium, the object will be at
v 2 = u2 + 2as rest or move with constant velocity.
1 15. A force can be resolved into vertical and horizontal
s = ut + at 2
2 components.
5. On a displacementtime graph, the gradient of the 16. Work is the product of a force and the distance
graph represents the velocity of the object. moved in the direction of the force.
6. On a velocitytime graph, 17. Energy is the capacity to do work.
(a) the gradient of the graph is equal to the (a) Potential energy, Ep = mgh
1
acceleration of the object. (b) Kinetic energy, Ek = mv 2
2
(b) the area under the graph is numerically equal to
the distance travelled. 18. Power is the rate at which work is done, or the rate at
7. Inertia is the tendency for an object to maintain its which energy is transformed.
state of rest or uniform motion in a straight line. Work done
Power =
8. Momentum of a moving object is the product of its Time taken
mass and its velocity. Useful power output
9. The principle of conservation of momentum states 19. Efficiency = 100%
Power input
that the total momentum of a system is conserved,
20. Hookes Law, F = kx
if no external force acts on the system. 1
(a) For inelastic collision, momentum and total Elastic potential energy, Ep = kx 2
2
energy are conserved but the kinetic energy after

Forces and Motion 158


SPM Exam Practice 2

2
Multiple-choice Questions

2.1 Linear Motion


1 Puan Halimah pushes her trolley from P to U via Q, R, S, T and U as shown in 4 Which features of a graph is used
Diagram 1. to determine acceleration?
A The slope of a displacement F
time graph O

2
B The slope of a velocitytime R

CHAPTER
M
graph
C The area under a 4
displacementtime graph
D The area under a velocitytime
graph

2.3 Inertia
5 A body stays at rest, or continues
SPM
Clone to move with a constant velocity,
09 unless an external force acts on it.
1.1 Significant Figure Which of the Newtons laws is
related to the above statement?
Diagram 1 A Newtons first law of motion
B Newtons second law of
motion
Find her displacement from P.
2.2 Motion Graphs C Newtons third law of motion
A 10 m
B 17 m 3 In Diagram 3, graphs X and Y are 6 A loaded ocean tanker is more
C 23 m the velocity-time graphs for car V difficult to stop than an empty one
D 77 m and car W respectively. because it
A possesses more kinetic energy
2 Cars P and Q start from rest
B sinks more into the water
simultaneously and accelerates at
C possesses more inertia
acceleration a and 3 m s2
D encounters a smaller frictional
respectively.
force 1.1 S

2.4 Momentum and


Conservation of
Momentum
Diagram 3
7 A golfer swings a club to hit a golf
ball. The club continues to swing
Diagram 2
Which of the following areas while the golf ball flies off with a
After 2 seconds, P is ahead of Q represents the distance between 1.1 high velocity.
Signifi Which of the
cant Figure
by 6 m. Find a. the two cars at time t? following quantities is conserved?
A 4.5 m s2 A OPQ A Kinetic energy
B 5.0 m s2 B OQR B Potential energy
C 5.5 m s2 C OPQR C Momentum
D 6.0 m s2 D OPR D Impulse

159 Forces and Motion


8 Diagram 4 shows two trolleys Which of the following tape charts
2.6 Impulse and Impulsive
P (mass = 2 kg) and can be expected if the student
Q (mass = 1 kg) placed together. Force repeats the experiment using a
P and Q move in opposite 11 An egg falls on a thick mattress. It falling weight of 200 g?
directions when the release pin is does not break because the mattress A
struck. A reduces the time of impact.
B prolongs the time of impact.
C reduces the kinetic energy of
the egg.
D reduces the eggs impulse.
12 Diagram 6 shows Nazli catches an
incoming softball of mass 0.14 kg
Diagram 4
F by moving his hand backwards.
O Which of the following statements B
2

R is correct?
A Kinetic energy of P
CHAPTER

M
= Kinetic energy of Q
4 B Momentum of P
= 1 Momentum of Q 1.1 Significant Figure
2
C Speed of P
= 1 the speed of Q
2 F Diagram 6
O C
The velocity of the ball reaching
2

D Velocity of P R
= 2 times velocity of Q his glove is 10 m s1. If the time
CHAPTER

M
taken to stop the ball is 1.2 s, what
4 is the force exerted on his hand?
A 0.9 N C 2.0 N
2.5 The Effects of a Force B 1.2 N D 8.6 N
9 The graph in Diagram 5 shows the
velocity-time graph of a car.
2.7 Safety Features in Vehicles
13 Which of the following is not a
safety feature of a car?
15 The mass of an object in Kuala
A Safety belts
Lumpur is 20 N. Compare its
B Spacious compartments
mass and weight on Mount
C Air bags
Everest.
D Padded dashboard

Mass Weight
Diagram 5
2.8 A 2 kg 20 N
Which part of the graph indicates Gravity
B 2 kg 20 N
that the resultant force on the car 14 The tape chart shown in Diagram C 2 kg 20 N
is zero? 7 is obtained in an experiment D 2 kg 20 N
A PQ C RS carried out in a laboratory.
B QR D ST
1.1 The falling weight used is
Significant Figure
10 A force of 9 N pushes an object of 100 grams. 1
16 The weight of a student is the
mass 3 kg on a floor. If the object 3
moves at a uniform velocity of weight of his father. If the total
2 m s1, what is the magnitude of 1.1
weight
Signifiof theFigure
cant student and his
the friction, in N? father is 600 N, what is the mass,
in kg, of the student?
A 1
3 A 15
B 3 B 20
C 6 C 30
D 9 Diagram 7 D 150

Forces and Motion 160


1.1 Significant Figure
At which point of the path does
2.9 Forces in Equilibrium 2.12 Elasticity
the stone possess minimum
17 FR1 and FR2 are the resultant forces kinetic energy? 24 A spring with an original length of
of the forces shown in Diagrams A P 20 cm is stretched to 26 cm.
8(a) and (b) respectively. B Q The force is F N and the elastic
C R potential energy stored is Ee. The
D S spring is then compressed so that
its length is now 17 cm. What is
the force and the elastic potential
21 Diagram 11 shows a pulley
energy stored in the spring?
system with P and Q of masses
(a) (b)
2 kg and 1 kg respectively.
Force Elastic
Diagram 8 applied potential F
What can be said about FR1 and FR2? energy stored O

2
A FR1 FR2 1 1 R
A F E
4 4 e

CHAPTER
B FR1 = FR2 M
C FR1 FR2 1 1
B F E 4
2 4 e
18 The following situations show
equilibrium of forces except 1.1 Significant Figure 1 1 1.1 S
C F E
SPM
Clone A a coconut falling from a tree. 2 2 e
08 B a ship floating at rest in the sea. D F Ee
C a car moving up a hill at Diagram 11
constant speed.
D a lamp hanging at rest in a hall. What is the total kinetic energy of
P and Q when Q rises 2 m?
A 5 J 25 Diagram 12 shows three identical
B 10 J springs, P, Q and R, supporting a
2.10 Work, Energy, Power and C 20 J load of 600 g. The original length
D 40 J of each spring is 10 cm.
Efficiency
19 A motor takes 5 s to lift a load of
SPM
6 kg to a height of 0.8 m. 22 Pak Hashim with a mass of
Clone
04 50 kg climbs a coconut tree
motor to a height of 6 m to pick a
coconut in 1 minute. What
is his power?
load A 5 W
B 25 W
C 50 W
0.8 m D 300 W

Diagram 9
2.11 The Importance of
What is the power of the motor?
A 9.6 W C 240 W Maximising the Efficiency
1.1 of cant
Signifi Devices
Figure
B 96 W D 375 W
23 The efficiency of a device can be Diagram 12
20 A boy throws a stone in the
increased if
direction as shown in Diagram 10.
A the useful power output is What is the total length, y, of the
increased while the power spring system if P, Q and R are
input is maintained. arranged in series and a load of
B the useful power output is 300 g is attached to spring R?
maintained while the A 27 cm
power input is increased. B 39 cm
C the useful power output and C 48 cm
Diagram 10 power input are increased. D 57 cm

161 Forces and Motion

1.1 Significant Figure


Structured Questions
1 A sprinter runs along a 100-m track as shown in 3 (a) Bakri carries a pail of sand up a flight of stairs of
Diagram 1. 1.2 m as shown in Diagram 3. The weight of the
pail of sand is 100 N.
100 m

t=5s

24 m

F Diagram 1
O
(a) The sprinter reaches his highest velocity
2

R
5seconds after starting from rest. He covers a
CHAPTER

M
displacement of 24 m in the 5 seconds. What is Diagram 3
4 his average acceleration? [2 marks]
(b) What is the average net force that is needed to What
100 mis the work done on the pail of sand by

achieve the acceleration in (a) if the mass of the Bakri? [2 marks]


sprinter is 60 kg? [2 marks] t=5s (b) Two elephants pulled the same pail of sand to the
(c) Find the total time it takes the sprinter to same height using two ropes and two pulleys as
complete 100 m if he maintainsF his
24 m highest shown in Diagram 4.
velocity. O
2

[ R 2 marks]
CHAPTER

M
2 A boy scout of a mass of 45 kg runs at a speed
of 6ms1 before grabbing a 4rope and swinging F4/2/145
upward.

Diagram 4
(i) How much work did the two elephants do
on the pail of sand? [1 mark]
(ii) If the angle between the two ropes is 178,
determine the tension in each rope.
[2 marks]
(iii) Is it possible to pull the ropes such that they
are horizontal to the floor? Explain your
answer.
Diagram 2 [3 marks]

4 Diagram 5 shows two blocks, P and Q, of masses 2m


(a) What is the change in energy? [1 mark]
and 3m respectively tied with a string. The spring
(b) How high will the boy swing upward?  [2 marks]
which is attached to block P is compressed to 12 cm.
[Take g = 9.8 m s2]
(c) The boy releases his hold on the rope at the
highest point of the swing.
(i) How does the boy fall? Tick your answer.

Vertically
In a parabolic path Diagram 5

[1 mark] (a) The force constant, k, for the spring is 5000 N m1.
(ii) State his velocity on landing. [1 mark] Explain the meaning of 5000 N m1.[1 mark]
(iii) The boy bends his knees and takes 1.2s to (b) Given the original length of the spring is 20 cm,
stop his fall. Find the impulsive force on his find the elastic potential energy stored in the
legs. spring. [2 marks]

Forces and Motion 162


(c) The string is burnt and P and Q are pushed apart from each other.
(i) Name the principle of physics in this motion. [1 mark]
(ii) If Q moves at a speed of 6 m s1, determine the speed of P.[2 marks]
(d) Determine the value of m.[2 marks]

Essay Questions
5 Diagram 6 shows two eggs of the same mass 6 (a) What is meant by force constant of a spring?
released from the same height. One egg falls on a  [1 mark] F
concrete floor and breaks while the other one falls on (b) Diagram 8 shows an increasing force applied to O

2
a thick folded towel and remains unbroken. the right against two springs. R

CHAPTER
M

Diagram 6

(a) (i) Define momentum. [1 mark]


(ii) Relate the outcome for each material to the
change in momentum. Then, deduce a Diagram 8
relevant physical concept. [5 marks]
Draw a force-distance graph to show how the
(b) Explain the changes in energy of the unbroken
force varies with the distance moved. The force
egg from the moment it is released until it
starts from zero and increases as it moves to the
touches the towel. [4 marks]
right. [2 marks]

Explain the shape of your graph. [2 marks]


(c) Mr Arsenal applies a stretching force of 380 N to
pull a chest expander (consists of 2 springs) from
deck 30 cm to 40 cm.

truck wheel
Diagram 7

(c) Diagram 7 shows a boy in action on his


stakeboard. Diagram 9
You are asked to investigate and suggest how the
skater can improve his performance by (i) Find the force constant of each spring.
considering the following aspects: (ii) What is the stretching force needed to pull
Skaters attire another chest expander of three springs
Structure of the skateboard (deck and truck) [identical to the springs in (i)] to 52 cm?
Wheels of the skateboard (c) Table 1 shows 4 toy pistols with different
Bearings fitted to the wheel [10 marks] specifications. You are required to determine the
most suitable toy pistol that can shoot the dart the
furthest.

163 Forces and Motion


Table 1

k Characteristics of the dart:


Rough body
Density = 1.6 g cm3
P = 1.6 g cm3


(a) (b)
Characteristics of the dart:
Smooth body
Density = 0.8 g cm3
F
O Q
2

R
CHAPTER

M

4 (a) (b)
Characteristics of the dart:
Rough body
Density = 1.2 g cm3

R F
O
2

R

CHAPTER

M
(a) (b)
4 Characteristics of the dart:
Smooth body
Density = 2.0 g cm3
S


(a) (b)
Study the specifications of all four toy pistols from the following aspects:
The force constant of the spring, k The density of the plastic dart
Compression of the spring The body of the dart
Explain the suitability of each aspect.
Justify your choice. [10 marks]

Experiment
1 A student carries out an experiment to study the relationship between the speed, v, of a trolley and the distance of
compression, e, of a spring that will push the trolley down the track after the compression of the spring is released.
A friction-compensated runway and a ticker-timer with its accessories are set up as shown in Diagram 1. The trolley is
pushed back to compress the spring with a compression, e, of 1.0 cm. The trolley is released. It moves at a constant
velocity, v, down the runway, after it is pushed by the compressed spring.

a.c. power supply

Diagram 1

Forces and Motion 164


The procedure is repeated with compression values of e, 2.0 cm, 3.0 cm, 4.0 cm and 5.0 cm.
The ticker tapes are shown in Diagrams 3(a), (b), (c), (d) and (e) respectively.
Diagram 2 shows the parts of the actual size ticker tape obtained in the experiment.
The velocity, v, of each compression can be calculated by using the formula:
x
v= cm s1
0.2
in which x is the distance covered in an interval of 10 ticks. An example is shown in Diagram 2.

Diagram 2 Part of a ticker tape at a constant velocity


F
O

2
R

CHAPTER
M

4
e = 1.0 cm
(a)


e = 2.0 cm
(b)


e = 3.0 cm
(c)


e = 4.0 cm
(d)


e = 5.0 cm
(e)
Diagram 3
(a) In the experiment described above, identify
(i) the manipulated variable, [1 mark]
(ii) the responding variable, [1 mark]
(iii) the constant variable. [1 mark]
(b) (i) Using the method shown in Diagram 2, measure x of every ticker tape in Diagram 3 with a ruler.
x
(ii) Using the formula v = , calculate the value of v for every ticker tape.
0.2
Tabulate your results for e, x and v. [7 marks]
(c) On the graph paper provided, draw a graph of v against e. [5 marks]
(d) Based on your graph, state the relationship between v and e. [1 mark]

COMPANION WEBSITE 165 Forces and Motion


Online Tests

You might also like